Re: Consistency of Postulates of QM (CORRECTION)

2018-06-21 Thread 'scerir' via Everything List

“The idea that they [measurement outcomes] be not alternatives but *all* really 
happen simultaneously seems lunatic to him [to the quantum theorist], just 
*impossible*. He thinks that if the laws of nature took *this* form for, let me 
say, a quarter of an hour, we should find our surroundings rapidly turning into 
a quagmire, or sort of a featureless jelly or plasma, all contours becoming 
blurred, we ourselves probably becoming jelly fish. It is strange that he 
should believe this. For I understand he grants that unobserved nature does 
behave this way – namely according to the wave equation. The aforesaid 
*alternatives* come into play only when we make an observation - which need, of 
course, not be a scientific observation. Still it would seem that, according to 
the quantum theorist, nature is prevented from rapid jellification only by our 
perceiving or observing it. [] The compulsion to replace the 
"simultaneous* happenings, as indicated directly by the theory, by 
*alternatives*, of which the theory is supposed to indicate the respective 
*probabilities*, arises from the conviction that what we really observe are 
particles - that actual events always concern particles, not waves." -Erwin 
Schroedinger, The Interpretation of Quantum Mechanics. Dublin Seminars 
(1949-1955) and Other Unpublished Essays (Ox Bow Press, Woodbridge, 
Connecticut, 1995), pages 19-20.


AG:  As your original quotation indicates, this was written pre-Everett who 
published his thesis in 1957. Who first got the idea that every outcome that's 
possible, must occur.  This is the person who led us astray. Unlikely that such 
a dumb idea would take hold. 


S:  Schroedinger wrote that (see above) in 1952 (July), well before H. Everett 
III. But the question might be: Schroedinger did not believe in *particles*, 
only in waves. Frankly I do not think that Everett - at least in his original 
"relative state" interpretation - had problems with *particles*. He usually 
reasoned in terms of 'wavefunction of particle'. "However, it seems to us to be 
much easier to understand particle aspects from a wave picture (concentrated 
wave packets) than it is to understand wave aspects (diffraction, interference, 
etc.) from a particle picture." "Even though the apparatus does not indicate 
any definite system value (since there are no independent system or apparatus 
states), one can nevertheless look upon the total wave function  as a 
superposition of pairs of subsystem states, each element of which has a 
definite q value and a correspondingly displaced apparatus state."

-- 
You received this message because you are subscribed to the Google Groups 
"Everything List" group.
To unsubscribe from this group and stop receiving emails from it, send an email 
to everything-list+unsubscr...@googlegroups.com.
To post to this group, send email to everything-list@googlegroups.com.
Visit this group at https://groups.google.com/group/everything-list.
For more options, visit https://groups.google.com/d/optout.


Re: Consistency of Postulates of QM (CORRECTION)

2018-06-21 Thread agrayson2000


On Thursday, June 21, 2018 at 8:25:59 AM UTC, scerir wrote:
>
> Il 5 dicembre 2017 alle 10.25 scerir > ha 
> scritto: 
>
> Sometimes I read and re-read something Schroedinger seemed to have in 
> mind. 
>
> “The idea that [the alternate measurement outcomes] be not alternatives 
> but *all *really happening simultaneously seems lunatic to [the quantum 
> theorist], just *impossible. *He thinks that if the laws of nature took *this 
> *form for, let me say, a quarter of an hour, we should find our 
> surroundings rapidly turning into a quagmire, a sort of a featureless jelly 
> or plasma, all contours becoming blurred, we ourselves probably becoming 
> jelly fish. It is strange that he should believe this. For I understand he 
> grants that unobserved nature does behave this way – namely according to 
> the wave equation. . . . according to the quantum theorist, nature is 
> prevented from rapid jellification only by our perceiving or observing it.” 
> --Erwin Schroedinger, *The Interpretation of Quantum Mechanic*
> ...


As your original quotation indicates, this was written pre-Everett who 
published his thesis in 1957. Who first got the idea that every outcome 
that's possible, must occur.  This is the person who led us astray. 
Unlikely that such a dumb idea would take hold. AG

-- 
You received this message because you are subscribed to the Google Groups 
"Everything List" group.
To unsubscribe from this group and stop receiving emails from it, send an email 
to everything-list+unsubscr...@googlegroups.com.
To post to this group, send email to everything-list@googlegroups.com.
Visit this group at https://groups.google.com/group/everything-list.
For more options, visit https://groups.google.com/d/optout.


Re: Consistency of Postulates of QM (CORRECTION)

2018-06-21 Thread 'scerir' via Everything List
Il 5 dicembre 2017 alle 10.25 scerir  ha scritto:

Sometimes I read and re-read something Schroedinger seemed to have in mind.

“The idea that [the alternate measurement outcomes] be not alternatives but all 
really happening simultaneously seems lunatic to [the quantum theorist], just 
impossible. He thinks that if the laws of nature took this form for, let me 
say, a quarter of an hour, we should find our surroundings rapidly turning into 
a quagmire, a sort of a featureless jelly or plasma, all contours becoming 
blurred, we ourselves probably becoming jelly fish. It is strange that he 
should believe this. For I understand he grants that unobserved nature does 
behave this way – namely according to the wave equation. . . . according to the 
quantum theorist, nature is prevented from rapid jellification only by our 
perceiving or observing it.” --Erwin Schroedinger, The Interpretation of 
Quantum Mechanics. Dublin Seminars (1949-1955) and Other Unpublished Essays
 s.



The actual quote was a bit different and IMO much more interesting. Here the 
correct quotation.

“The idea that they [measurement outcomes] be not alternatives but *all* really 
happen simultaneously seems lunatic to him [to the quantum theorist], just 
*impossible*. He thinks that if the laws of nature took *this* form for, let me 
say, a quarter of an hour, we should find our surroundings rapidly turning into 
a quagmire, or sort of a featureless jelly or plasma, all contours becoming 
blurred, we ourselves probably becoming jelly fish. It is strange that he 
should believe this. For I understand he grants that unobserved nature does 
behave this way – namely according to the wave equation. The aforesaid 
*alternatives* come into play only when we make an observation - which need, of 
course, not be a scientific observation. Still it would seem that, according to 
the quantum theorist, nature is prevented from rapid jellification only by our 
perceiving or observing it. [] The compulsion to replace the 
"simultaneous* happenings, as indicated directly by the theory, by 
*alternatives*, of which the theory is supposed to indicate the respective 
*probabilities*, arises from the conviction that what we really observe are 
particles - that actual events always concern particles, not waves."

-Erwin Schroedinger, The Interpretation of Quantum Mechanics. Dublin Seminars 
(1949-1955) and Other Unpublished Essays (Ox Bow Press, Woodbridge, 
Connecticut, 1995), pages 19-20.


 

-- 
You received this message because you are subscribed to the Google Groups 
"Everything List" group.
To unsubscribe from this group and stop receiving emails from it, send an email 
to everything-list+unsubscr...@googlegroups.com.
To post to this group, send email to everything-list@googlegroups.com.
Visit this group at https://groups.google.com/group/everything-list.
For more options, visit https://groups.google.com/d/optout.


Re: Consistency of Postulates of QM

2017-12-26 Thread Bruno Marchal


On 24 Dec 2017, at 02:31, Lawrence Crowell wrote:

On Saturday, December 23, 2017 at 11:20:27 AM UTC-6, Bruno Marchal  
wrote:


On 22 Dec 2017, at 16:21, Lawrence Crowell wrote:



On Wednesday, December 20, 2017 at 6:36:39 AM UTC-6, Bruno Marchal  
wrote:


On 19 Dec 2017, at 20:08, agrays...@gmail.com wrote:



On Tuesday, December 19, 2017 at 4:48:48 PM UTC, Bruno Marchal wrote:

On 18 Dec 2017, at 00:34, agrays...@gmail.com wrote:

That makes things worst. Toy will make physical outcomes directly  
dependent of the consciousness/knowledge of the observer.


Why do you and Clark persist in this error? Collapse, if it occurs,  
does NOT depend on human consciousness.


OK but what you said made it depending of consciousness.  There is  
no collapse possible with the SWE, so what is the collapse? All  
theories introducing a collapse in between the consciousness of the  
observer and the observed objects seem highly speculative to me.


Measurement is just a specific instance of decoherence. There have  
been "effective measurements" for billions of years.


OK. But not just in the universal wave, also in arithmetic, and  
Mechanism forces them to be equivalent at some level, in a  
verifiable way/


What is proved is that in all theories rich enough to add and  
multiplied distinguishable inductive termes, like numbers, or  
combinators, universal numbers and universal combinators exists, and  
we have to retrieved physics from arithmetic using the self- 
referential abilities of the universal machines (the Löbian chatty  
one).




I would agree to the extent these objects connect with  
Grothendieck's categories or Motivic. Quantum mechanics and  
spacetime physics share properties of "magma" that is a category type.



You might elaborate on this. The point is that whatever the physical  
reality is, it has to be the unique one in the head of all universal  
machines/numbers, so if it shares properties of "magma" (whatever that  
is), we can and should verify it in arithmetic. Then the G/G*  
separation might separate the justifiable and non justifiable part of  
the magma involved.







Chert or flint carry cosmic ray tracks, which indicate particles  
(likely muons) had their wave functions reduced with the interaction  
of the rock material many millions of years ago.


Not at all. When *we* do a measurement w just localized ourselves in  
the universal wave/matrix (it works in all pictures). The wave never  
collapse. I like feynman when he describe the collapse as a  
collective hallucination. It is exactly what Mechanism ensures.


I tend to think the geologist looking at cosmic ray tracks in chert  
is performing a classical measurement, at least on the level of FAPP.


What is a classical measurement? You do seem to believe in the  
collapse (and in a irreducibly physical ontology). Ah, maybe you don't  
as I see well below.




The quantum phase of superposition or entanglement associated with  
the state reduction of particles interacting with the stone are long  
gone.


Then the SWE, Dirac, Feynman does not apply to the classical realm,  
but what is that realm, and how to test its existence? With the SWE  
the quantum superpositions never disappear.





They have gone the way of T-rex or what ever other creatures lived  
then. The phase still exists, for if we think unitarity if  
fundamental then overlap phases and entanglement phases are  
conserved, but in this case the overlap phases has long ago done the  
"Elvis has left the building." What ever quantum phases there are  
associated with the track are from more recent interactions.


OK. Thanks for reassuring me! But when searching the truth, we must  
admit the existence of things which we cannot directly observe (like  
quarks), and if you agree on the global unitarity, then, well, you get  
the "parallel histories", even if we cannot interact with them. To  
know the exact 10^120 decimals of the position of a coin (assuming  
that makes sense) might need the terms in the universal wave which  
involve the dinosaurs in the parallel histories, even for a Bohmian  
(which will just not allow the dinosaurs wave to have particles).


Bruno








LC






I have a hard time thinking that a geologist looking at this has  
performed a quantum measurement that reduces the wave function then  
and there. It does not do much to invoke life either, for I doubt  
that an unusually smart trilobite or cockroach in the Paleozoic  
epoch would perform a measurement with some conscious idea of QM.

...

--
You received this message because you are subscribed to the Google  
Groups "Everything List" group.
To unsubscribe from this group and stop receiving emails from it,  
send an email to everything-list+unsubscr...@googlegroups.com.

To post to this group, send email to everything-list@googlegroups.com.
Visit this group at https://groups.google.com/group/everything-list.
For more options, visit https://groups.google.com/d/optout.



Re: Consistency of Postulates of QM

2017-12-26 Thread Bruno Marchal


On 25 Dec 2017, at 21:41, agrayson2...@gmail.com wrote:




On Monday, December 25, 2017 at 8:28:30 PM UTC, Russell Standish  
wrote:

On Sun, Dec 24, 2017 at 07:11:25PM -0800, agrays...@gmail.com wrote:
>
> *OK. I was thinking of the time evolution operator, denoted by U,  
which I

> believe is linear in t. AG*

Yes, it is linear and unitary. Unitary operators are linear, but
linear operators are not necessarily unitary. That is what I meant
when I said "unitary is not linear". They are not synonyms.

>
>
> > Projection operators are linear.
>
>
>
>
>
> *IIUC, projection operators model the "collapse" of the  
superposition of
> states to a single state within the superposition. Since the  
measurement
> process is believed to be non linear, how can the projection  
operator be

> linear?

I've never heard of a nonlinear measurement process. Who believes
that? The only person who's done any work researching nonlinear QM was
Weinberg IIRC.

Spin measurements are irreversible in principle, not simply FAPP.  
Bruce
showed that on Avoid2 IIRC. I think this means the measurement  
process,
at least in this case, must be non-linear. If that's true, then how  
can a linear

process such as decoherence, model measurements? AG

von Neumann's proposal was to use projection operators, which are
linear, but non-unitary. Being non-unitary is a problem, given unitary
evolution is one of the foundational axioms of QM.

Unitary evolution does not extend to the measurement process;


In the theory which assumes a physical collapse. Without collapse, the  
projection is a first person indexical explainable by Mechanism.






only
evolution of the wf prior to measurement. AG


Which leads to a dualist (and unknown, controversial) theory of mind.

That is the main motivation for Everett: it solves the measurement  
problem, just by eliminating the collapse of the wave, and showing how  
the recover the appearance of collapse in the mind (or in the personal  
diaries) of the observers.


The extension of that idea to arithmetic seems promising to explain  
how the wave itself get selected in the mind of most universal beings.


Bruno







> This raises a question about decoherence. If the myriad of
> individual processes are linear, which I believe is what the model  
affirms,

> how can any  measurement be non linear as it presumably is for spin
> measurements. AG*
>

Indeed that would be a problem. A sum of unitary operators need not be
unitary, though.



--


Dr Russell StandishPhone 0425 253119 (mobile)
Principal, High Performance Coders
Visiting Senior Research Fellowhpc...@hpcoders.com.au
Economics, Kingston University http://www.hpcoders.com.au


--
You received this message because you are subscribed to the Google  
Groups "Everything List" group.
To unsubscribe from this group and stop receiving emails from it,  
send an email to everything-list+unsubscr...@googlegroups.com.

To post to this group, send email to everything-list@googlegroups.com.
Visit this group at https://groups.google.com/group/everything-list.
For more options, visit https://groups.google.com/d/optout.


http://iridia.ulb.ac.be/~marchal/



--
You received this message because you are subscribed to the Google Groups 
"Everything List" group.
To unsubscribe from this group and stop receiving emails from it, send an email 
to everything-list+unsubscr...@googlegroups.com.
To post to this group, send email to everything-list@googlegroups.com.
Visit this group at https://groups.google.com/group/everything-list.
For more options, visit https://groups.google.com/d/optout.


Re: Consistency of Postulates of QM

2017-12-25 Thread agrayson2000


On Monday, December 25, 2017 at 8:44:42 PM UTC, Russell Standish wrote:
>
> On Mon, Dec 25, 2017 at 12:32:26PM -0800, agrays...@gmail.com 
>  wrote: 
> > > 
> > > *Not linear in t, but also named "unitary operator", not to be 
> confused 
> > > with the operator by the same name that preserves inner products. AG* 
> > > 
> > 
> > 
> > *Another correction: the time evolution operator is a unitary operator 
> > since it preserves inner products, but it is NOT NAMED a unitary 
> operator. 
> > AG * 
> > 
>
> That doesn't make sense. The evolution operator is of the form 
> exp(-i/ℏ Ht), where the H, the Hamiltonian operator, is assumed to 
> be Hermitian. It is a relatively trivial exercise to prove that any 
> operator of the form exp(iA) is unitary,


I didn't write it isn't unitary. I just corrected my earlier error 
concerning
its NAME. Its name is the *time evolution operator.* It is unitary but
NOT linear in t. AG
 

> where A is Hermitian. Trivial 
> when you see how to do it, but nevertheless I had to seek help from my 
> college tutor when I first encountered this :). 
>
> Cheers 
> -- 
>
>  
>
> Dr Russell StandishPhone 0425 253119 (mobile) 
> Principal, High Performance Coders 
> Visiting Senior Research Fellowhpc...@hpcoders.com.au 
>  
> Economics, Kingston University http://www.hpcoders.com.au 
>  
>
>

-- 
You received this message because you are subscribed to the Google Groups 
"Everything List" group.
To unsubscribe from this group and stop receiving emails from it, send an email 
to everything-list+unsubscr...@googlegroups.com.
To post to this group, send email to everything-list@googlegroups.com.
Visit this group at https://groups.google.com/group/everything-list.
For more options, visit https://groups.google.com/d/optout.


Re: Consistency of Postulates of QM

2017-12-25 Thread Russell Standish
On Mon, Dec 25, 2017 at 12:41:35PM -0800, agrayson2...@gmail.com wrote:
> 
> 
> 
> *Spin measurements are irreversible in principle, not simply FAPP. Bruce 
> showed that on Avoid2 IIRC. I think this means the measurement process,at 
> least in this case, must be non-linear. If that's true, then how can a 
> linearprocess such as decoherence, model measurements? AG*  

Maybe Bruce can comment, but in general, for a reversible process, the
evolution operator needs to be unitary (ie probability preserving). A
non unitary (but linear) process will be irreversible.

> 
> >
> > von Neumann's proposal was to use projection operators, which are 
> > linear, but non-unitary. Being non-unitary is a problem, given unitary 
> > evolution is one of the foundational axioms of QM. 
> >
> 
> 
> 
> *Unitary evolution does not extend to the measurement process; 
> onlyevolution of the wf prior to measurement. AG *

Yes - that is exactly the measurement problem.



-- 


Dr Russell StandishPhone 0425 253119 (mobile)
Principal, High Performance Coders
Visiting Senior Research Fellowhpco...@hpcoders.com.au
Economics, Kingston University http://www.hpcoders.com.au


-- 
You received this message because you are subscribed to the Google Groups 
"Everything List" group.
To unsubscribe from this group and stop receiving emails from it, send an email 
to everything-list+unsubscr...@googlegroups.com.
To post to this group, send email to everything-list@googlegroups.com.
Visit this group at https://groups.google.com/group/everything-list.
For more options, visit https://groups.google.com/d/optout.


Re: Consistency of Postulates of QM

2017-12-25 Thread Russell Standish
On Mon, Dec 25, 2017 at 12:32:26PM -0800, agrayson2...@gmail.com wrote:
> >
> > *Not linear in t, but also named "unitary operator", not to be confused 
> > with the operator by the same name that preserves inner products. AG*
> >
> 
> 
> *Another correction: the time evolution operator is a unitary operator 
> since it preserves inner products, but it is NOT NAMED a unitary operator. 
> AG *
> 

That doesn't make sense. The evolution operator is of the form
exp(-i/ℏ Ht), where the H, the Hamiltonian operator, is assumed to
be Hermitian. It is a relatively trivial exercise to prove that any
operator of the form exp(iA) is unitary, where A is Hermitian. Trivial
when you see how to do it, but nevertheless I had to seek help from my
college tutor when I first encountered this :).

Cheers
-- 


Dr Russell StandishPhone 0425 253119 (mobile)
Principal, High Performance Coders
Visiting Senior Research Fellowhpco...@hpcoders.com.au
Economics, Kingston University http://www.hpcoders.com.au


-- 
You received this message because you are subscribed to the Google Groups 
"Everything List" group.
To unsubscribe from this group and stop receiving emails from it, send an email 
to everything-list+unsubscr...@googlegroups.com.
To post to this group, send email to everything-list@googlegroups.com.
Visit this group at https://groups.google.com/group/everything-list.
For more options, visit https://groups.google.com/d/optout.


Re: Consistency of Postulates of QM

2017-12-25 Thread agrayson2000


On Monday, December 25, 2017 at 8:28:30 PM UTC, Russell Standish wrote:
>
> On Sun, Dec 24, 2017 at 07:11:25PM -0800, agrays...@gmail.com 
>  wrote: 
> > 
> > *OK. I was thinking of the time evolution operator, denoted by U, which 
> I 
> > believe is linear in t. AG* 
>
> Yes, it is linear and unitary. Unitary operators are linear, but 
> linear operators are not necessarily unitary. That is what I meant 
> when I said "unitary is not linear". They are not synonyms. 
>
> >   
> > 
> > > Projection operators are linear. 
> > 
> > 
> > 
> > 
> > 
> > *IIUC, projection operators model the "collapse" of the superposition of 
> > states to a single state within the superposition. Since the measurement 
> > process is believed to be non linear, how can the projection operator be 
> > linear? 
>
> I've never heard of a nonlinear measurement process. Who believes 
> that? The only person who's done any work researching nonlinear QM was 
> Weinberg IIRC. 
>




*Spin measurements are irreversible in principle, not simply FAPP. Bruce 
showed that on Avoid2 IIRC. I think this means the measurement process,at 
least in this case, must be non-linear. If that's true, then how can a 
linearprocess such as decoherence, model measurements? AG*  

>
> von Neumann's proposal was to use projection operators, which are 
> linear, but non-unitary. Being non-unitary is a problem, given unitary 
> evolution is one of the foundational axioms of QM. 
>



*Unitary evolution does not extend to the measurement process; 
onlyevolution of the wf prior to measurement. AG *

>
> > This raises a question about decoherence. If the myriad of 
> > individual processes are linear, which I believe is what the model 
> affirms, 
> > how can any  measurement be non linear as it presumably is for spin 
> > measurements. AG* 
> > 
>
> Indeed that would be a problem. A sum of unitary operators need not be 
> unitary, though. 
>
>
>
> -- 
>
>  
>
> Dr Russell StandishPhone 0425 253119 (mobile) 
> Principal, High Performance Coders 
> Visiting Senior Research Fellowhpc...@hpcoders.com.au 
>  
> Economics, Kingston University http://www.hpcoders.com.au 
>  
>
>

-- 
You received this message because you are subscribed to the Google Groups 
"Everything List" group.
To unsubscribe from this group and stop receiving emails from it, send an email 
to everything-list+unsubscr...@googlegroups.com.
To post to this group, send email to everything-list@googlegroups.com.
Visit this group at https://groups.google.com/group/everything-list.
For more options, visit https://groups.google.com/d/optout.


Re: Consistency of Postulates of QM

2017-12-25 Thread agrayson2000


On Monday, December 25, 2017 at 5:49:34 AM UTC, agrays...@gmail.com wrote:
>
>
>
> On Monday, December 25, 2017 at 3:11:25 AM UTC, agrays...@gmail.com wrote:
>>
>>  
>> On Sunday, December 24, 2017 at 9:33:56 AM UTC, Russell Standish wrote:
>>>
>>> On Sat, Dec 23, 2017 at 02:10:44PM -0800, agrays...@gmail.com wrote: 
>>> > 
>>> > 
>>> > On Saturday, December 23, 2017 at 2:11:32 PM UTC-7, Russell Standish 
>>> wrote: 
>>> > > 
>>> > > On Sat, Dec 23, 2017 at 09:20:05AM -0800, agrays...@gmail.com 
>>> > >  wrote: 
>>> > > > 
>>> > > > My tentative solution to the wave collapse problem is to trash 
>>> wave 
>>> > > > mechanics (which is not Lorentz invariant) and use Heisenberg's 
>>> Matrix 
>>> > > > Mechanics. No waves, nothing to collapse. Is this a cop-out? AG 
>>> > > 
>>> > > In matrix mechanics, the wave function is replaced by a vector, and 
>>> > > collapse is replaced by a projection onto a basis vector. 
>>> > > 
>>> > > Projections are not unitary (except for the identity matrix), and 
>>> that 
>>> > > is the problem with any collapse type theory. 
>>> > > 
>>> > 
>>> > Thanks. That's clears enough. Collapse by another name. CMIIAW, but 
>>> even if 
>>> > it were a unitary process, it would in effect be a local hidden 
>>> variable, 
>>> > forbidden by results of Bell experiments. But let's talk about 
>>> "unitary" 
>>> > which I think is equivalent to "linear". Why is non unitary, that is 
>>> non 
>>> > linear bad? Because it means irreversible? I do believe that some 
>>> > measurement processes are in fact irreversible in principle, and not 
>>> simply 
>>> > in the statistical sense, that is, FAPP.  IIRC, Bruce proved that for 
>>> spin 
>>> > measurements on Avoid2, but it was not well received. AG 
>>> > 
>>> I 
>>> Unitary does not mean linear.
>>
>>
>> *OK. I was thinking of the time evolution operator, denoted by U, which I 
>> believe is linear in t. AG*
>>
>
> *Not linear in t, but also named "unitary operator", not to be confused 
> with the operator by the same name that preserves inner products. AG*
>


*Another correction: the time evolution operator is a unitary operator 
since it preserves inner products, but it is NOT NAMED a unitary operator. 
AG *

>
>  
>
>> Projection operators are linear. 
>>
>>
>>
>>
>>
>> *IIUC, projection operators model the "collapse" of the superposition of 
>> states to a single state within the superposition. Since the measurement 
>> process is believed to be non linear, how can the projection operator be 
>> linear? AGThis raises a question about decoherence. If the myriad of 
>> individual processes are linear, which I believe is what the model affirms, 
>> how can any  measurement be non linear as it presumably is for spin 
>> measurements. AG* 
>>
>>> Unitary 
>>> means that applying the operator to a group of vectors preserves their 
>>> lengths and the angles between them. Effectively just a coordinate 
>>> transformation. Another property conserved is overall probability. 
>>>
>>> By constrast projections (or as my maths lecturer was fond of saying 
>>> "elephant foot map") squash things. Like the cockroach under my shoe, 
>>> lengths and angles between components are not preserved. 
>>>
>>> Cheers 
>>>
>>> -- 
>>>
>>> 
>>>  
>>>
>>> Dr Russell StandishPhone 0425 253119 (mobile) 
>>> Principal, High Performance Coders 
>>> Visiting Senior Research Fellowhpc...@hpcoders.com.au 
>>> Economics, Kingston University http://www.hpcoders.com.au 
>>> 
>>>  
>>>
>>>
>>

-- 
You received this message because you are subscribed to the Google Groups 
"Everything List" group.
To unsubscribe from this group and stop receiving emails from it, send an email 
to everything-list+unsubscr...@googlegroups.com.
To post to this group, send email to everything-list@googlegroups.com.
Visit this group at https://groups.google.com/group/everything-list.
For more options, visit https://groups.google.com/d/optout.


Re: Consistency of Postulates of QM

2017-12-25 Thread Russell Standish
On Sun, Dec 24, 2017 at 07:11:25PM -0800, agrayson2...@gmail.com wrote:
> 
> *OK. I was thinking of the time evolution operator, denoted by U, which I 
> believe is linear in t. AG*

Yes, it is linear and unitary. Unitary operators are linear, but
linear operators are not necessarily unitary. That is what I meant
when I said "unitary is not linear". They are not synonyms.

>  
> 
> > Projection operators are linear. 
> 
> 
> 
> 
> 
> *IIUC, projection operators model the "collapse" of the superposition of 
> states to a single state within the superposition. Since the measurement 
> process is believed to be non linear, how can the projection operator be 
> linear?

I've never heard of a nonlinear measurement process. Who believes
that? The only person who's done any work researching nonlinear QM was
Weinberg IIRC.

von Neumann's proposal was to use projection operators, which are
linear, but non-unitary. Being non-unitary is a problem, given unitary
evolution is one of the foundational axioms of QM.

> AGThis raises a question about decoherence. If the myriad of 
> individual processes are linear, which I believe is what the model affirms, 
> how can any  measurement be non linear as it presumably is for spin 
> measurements. AG* 
>

Indeed that would be a problem. A sum of unitary operators need not be
unitary, though.



-- 


Dr Russell StandishPhone 0425 253119 (mobile)
Principal, High Performance Coders
Visiting Senior Research Fellowhpco...@hpcoders.com.au
Economics, Kingston University http://www.hpcoders.com.au


-- 
You received this message because you are subscribed to the Google Groups 
"Everything List" group.
To unsubscribe from this group and stop receiving emails from it, send an email 
to everything-list+unsubscr...@googlegroups.com.
To post to this group, send email to everything-list@googlegroups.com.
Visit this group at https://groups.google.com/group/everything-list.
For more options, visit https://groups.google.com/d/optout.


Re: Consistency of Postulates of QM

2017-12-24 Thread agrayson2000


On Monday, December 25, 2017 at 3:11:25 AM UTC, agrays...@gmail.com wrote:
>
>  
> On Sunday, December 24, 2017 at 9:33:56 AM UTC, Russell Standish wrote:
>>
>> On Sat, Dec 23, 2017 at 02:10:44PM -0800, agrays...@gmail.com wrote: 
>> > 
>> > 
>> > On Saturday, December 23, 2017 at 2:11:32 PM UTC-7, Russell Standish 
>> wrote: 
>> > > 
>> > > On Sat, Dec 23, 2017 at 09:20:05AM -0800, agrays...@gmail.com 
>> > >  wrote: 
>> > > > 
>> > > > My tentative solution to the wave collapse problem is to trash wave 
>> > > > mechanics (which is not Lorentz invariant) and use Heisenberg's 
>> Matrix 
>> > > > Mechanics. No waves, nothing to collapse. Is this a cop-out? AG 
>> > > 
>> > > In matrix mechanics, the wave function is replaced by a vector, and 
>> > > collapse is replaced by a projection onto a basis vector. 
>> > > 
>> > > Projections are not unitary (except for the identity matrix), and 
>> that 
>> > > is the problem with any collapse type theory. 
>> > > 
>> > 
>> > Thanks. That's clears enough. Collapse by another name. CMIIAW, but 
>> even if 
>> > it were a unitary process, it would in effect be a local hidden 
>> variable, 
>> > forbidden by results of Bell experiments. But let's talk about 
>> "unitary" 
>> > which I think is equivalent to "linear". Why is non unitary, that is 
>> non 
>> > linear bad? Because it means irreversible? I do believe that some 
>> > measurement processes are in fact irreversible in principle, and not 
>> simply 
>> > in the statistical sense, that is, FAPP.  IIRC, Bruce proved that for 
>> spin 
>> > measurements on Avoid2, but it was not well received. AG 
>> > 
>> I 
>> Unitary does not mean linear.
>
>
> *OK. I was thinking of the time evolution operator, denoted by U, which I 
> believe is linear in t. AG*
>

*Not linear in t, but also named "unitary operator", not to be confused 
with the operator by the same name that preserves inner products. AG*
 

> Projection operators are linear. 
>
>
>
>
>
> *IIUC, projection operators model the "collapse" of the superposition of 
> states to a single state within the superposition. Since the measurement 
> process is believed to be non linear, how can the projection operator be 
> linear? AGThis raises a question about decoherence. If the myriad of 
> individual processes are linear, which I believe is what the model affirms, 
> how can any  measurement be non linear as it presumably is for spin 
> measurements. AG* 
>
>> Unitary 
>> means that applying the operator to a group of vectors preserves their 
>> lengths and the angles between them. Effectively just a coordinate 
>> transformation. Another property conserved is overall probability. 
>>
>> By constrast projections (or as my maths lecturer was fond of saying 
>> "elephant foot map") squash things. Like the cockroach under my shoe, 
>> lengths and angles between components are not preserved. 
>>
>> Cheers 
>>
>> -- 
>>
>>  
>>
>> Dr Russell StandishPhone 0425 253119 (mobile) 
>> Principal, High Performance Coders 
>> Visiting Senior Research Fellowhpc...@hpcoders.com.au 
>> Economics, Kingston University http://www.hpcoders.com.au 
>>  
>>
>>
>

-- 
You received this message because you are subscribed to the Google Groups 
"Everything List" group.
To unsubscribe from this group and stop receiving emails from it, send an email 
to everything-list+unsubscr...@googlegroups.com.
To post to this group, send email to everything-list@googlegroups.com.
Visit this group at https://groups.google.com/group/everything-list.
For more options, visit https://groups.google.com/d/optout.


Re: Consistency of Postulates of QM

2017-12-24 Thread agrayson2000
 
On Sunday, December 24, 2017 at 9:33:56 AM UTC, Russell Standish wrote:
>
> On Sat, Dec 23, 2017 at 02:10:44PM -0800, agrays...@gmail.com 
>  wrote: 
> > 
> > 
> > On Saturday, December 23, 2017 at 2:11:32 PM UTC-7, Russell Standish 
> wrote: 
> > > 
> > > On Sat, Dec 23, 2017 at 09:20:05AM -0800, agrays...@gmail.com 
> > >  wrote: 
> > > > 
> > > > My tentative solution to the wave collapse problem is to trash wave 
> > > > mechanics (which is not Lorentz invariant) and use Heisenberg's 
> Matrix 
> > > > Mechanics. No waves, nothing to collapse. Is this a cop-out? AG 
> > > 
> > > In matrix mechanics, the wave function is replaced by a vector, and 
> > > collapse is replaced by a projection onto a basis vector. 
> > > 
> > > Projections are not unitary (except for the identity matrix), and that 
> > > is the problem with any collapse type theory. 
> > > 
> > 
> > Thanks. That's clears enough. Collapse by another name. CMIIAW, but even 
> if 
> > it were a unitary process, it would in effect be a local hidden 
> variable, 
> > forbidden by results of Bell experiments. But let's talk about "unitary" 
> > which I think is equivalent to "linear". Why is non unitary, that is non 
> > linear bad? Because it means irreversible? I do believe that some 
> > measurement processes are in fact irreversible in principle, and not 
> simply 
> > in the statistical sense, that is, FAPP.  IIRC, Bruce proved that for 
> spin 
> > measurements on Avoid2, but it was not well received. AG 
> > 
> I 
> Unitary does not mean linear.


*OK. I was thinking of the time evolution operator, denoted by U, which I 
believe is linear in t. AG*
 

> Projection operators are linear. 





*IIUC, projection operators model the "collapse" of the superposition of 
states to a single state within the superposition. Since the measurement 
process is believed to be non linear, how can the projection operator be 
linear? AGThis raises a question about decoherence. If the myriad of 
individual processes are linear, which I believe is what the model affirms, 
how can any  measurement be non linear as it presumably is for spin 
measurements. AG* 

> Unitary 
> means that applying the operator to a group of vectors preserves their 
> lengths and the angles between them. Effectively just a coordinate 
> transformation. Another property conserved is overall probability. 
>
> By constrast projections (or as my maths lecturer was fond of saying 
> "elephant foot map") squash things. Like the cockroach under my shoe, 
> lengths and angles between components are not preserved. 
>
> Cheers 
>
> -- 
>
>  
>
> Dr Russell StandishPhone 0425 253119 (mobile) 
> Principal, High Performance Coders 
> Visiting Senior Research Fellowhpc...@hpcoders.com.au 
>  
> Economics, Kingston University http://www.hpcoders.com.au 
>  
>
>

-- 
You received this message because you are subscribed to the Google Groups 
"Everything List" group.
To unsubscribe from this group and stop receiving emails from it, send an email 
to everything-list+unsubscr...@googlegroups.com.
To post to this group, send email to everything-list@googlegroups.com.
Visit this group at https://groups.google.com/group/everything-list.
For more options, visit https://groups.google.com/d/optout.


Re: Consistency of Postulates of QM

2017-12-24 Thread Russell Standish
On Sat, Dec 23, 2017 at 02:10:44PM -0800, agrayson2...@gmail.com wrote:
> 
> 
> On Saturday, December 23, 2017 at 2:11:32 PM UTC-7, Russell Standish wrote:
> >
> > On Sat, Dec 23, 2017 at 09:20:05AM -0800, agrays...@gmail.com 
> >  wrote: 
> > > 
> > > My tentative solution to the wave collapse problem is to trash wave 
> > > mechanics (which is not Lorentz invariant) and use Heisenberg's Matrix 
> > > Mechanics. No waves, nothing to collapse. Is this a cop-out? AG 
> >
> > In matrix mechanics, the wave function is replaced by a vector, and 
> > collapse is replaced by a projection onto a basis vector. 
> >
> > Projections are not unitary (except for the identity matrix), and that 
> > is the problem with any collapse type theory. 
> >
> 
> Thanks. That's clears enough. Collapse by another name. CMIIAW, but even if 
> it were a unitary process, it would in effect be a local hidden variable, 
> forbidden by results of Bell experiments. But let's talk about "unitary" 
> which I think is equivalent to "linear". Why is non unitary, that is non 
> linear bad? Because it means irreversible? I do believe that some 
> measurement processes are in fact irreversible in principle, and not simply 
> in the statistical sense, that is, FAPP.  IIRC, Bruce proved that for spin 
> measurements on Avoid2, but it was not well received. AG 
> 

Unitary does not mean linear. Projection operators are linear. Unitary
means that applying the operator to a group of vectors preserves their
lengths and the angles between them. Effectively just a coordinate
transformation. Another property conserved is overall probability.

By constrast projections (or as my maths lecturer was fond of saying
"elephant foot map") squash things. Like the cockroach under my shoe,
lengths and angles between components are not preserved.

Cheers

-- 


Dr Russell StandishPhone 0425 253119 (mobile)
Principal, High Performance Coders
Visiting Senior Research Fellowhpco...@hpcoders.com.au
Economics, Kingston University http://www.hpcoders.com.au


-- 
You received this message because you are subscribed to the Google Groups 
"Everything List" group.
To unsubscribe from this group and stop receiving emails from it, send an email 
to everything-list+unsubscr...@googlegroups.com.
To post to this group, send email to everything-list@googlegroups.com.
Visit this group at https://groups.google.com/group/everything-list.
For more options, visit https://groups.google.com/d/optout.


Re: Consistency of Postulates of QM

2017-12-23 Thread Lawrence Crowell
On Saturday, December 23, 2017 at 11:20:27 AM UTC-6, Bruno Marchal wrote:
>
>
> On 22 Dec 2017, at 16:21, Lawrence Crowell wrote:
>
>
>
> On Wednesday, December 20, 2017 at 6:36:39 AM UTC-6, Bruno Marchal wrote:
>
>
> On 19 Dec 2017, at 20:08, agrays...@gmail.com wrote:
>
>
>
> On Tuesday, December 19, 2017 at 4:48:48 PM UTC, Bruno Marchal wrote:
>
>
> On 18 Dec 2017, at 00:34, agrays...@gmail.com wrote:
>
>
> That makes things worst. Toy will make physical outcomes directly 
> dependent of the consciousness/knowledge of the observer.
>
>
> Why do you and Clark persist in this error? Collapse, if it occurs, does 
> NOT depend on human consciousness. 
>
>
> OK but what you said made it depending of consciousness.  There is no 
> collapse possible with the SWE, so what is the collapse? All theories 
> introducing a collapse in between the consciousness of the observer and the 
> observed objects seem highly speculative to me.
>
>
> Measurement is just a specific instance of decoherence. There have been 
> "effective measurements" for billions of years. 
>
>
> OK. But not just in the universal wave, also in arithmetic, and Mechanism 
> forces them to be equivalent at some level, in a verifiable way/
>
> What is proved is that in all theories rich enough to add and multiplied 
> distinguishable inductive termes, like numbers, or combinators, universal 
> numbers and universal combinators exists, and we have to retrieved physics 
> from arithmetic using the self-referential abilities of the universal 
> machines (the Löbian chatty one).
>
>
>
I would agree to the extent these objects connect with Grothendieck's 
categories or Motivic. Quantum mechanics and spacetime physics share 
properties of "magma" that is a category type. 
 

>
> Chert or flint carry cosmic ray tracks, which indicate particles (likely 
> muons) had their wave functions reduced with the interaction of the rock 
> material many millions of years ago.
>
>
> Not at all. When *we* do a measurement w just localized ourselves in the 
> universal wave/matrix (it works in all pictures). The wave never collapse. 
> I like feynman when he describe the collapse as a collective hallucination. 
> It is exactly what Mechanism ensures.
>

I tend to think the geologist looking at cosmic ray tracks in chert is 
performing a classical measurement, at least on the level of FAPP. The 
quantum phase of superposition or entanglement associated with the state 
reduction of particles interacting with the stone are long gone. They have 
gone the way of T-rex or what ever other creatures lived then. The phase 
still exists, for if we think unitarity if fundamental then overlap phases 
and entanglement phases are conserved, but in this case the overlap phases 
has long ago done the "Elvis has left the building." What ever quantum 
phases there are associated with the track are from more recent 
interactions. 

LC
 

>
>
>
>
>
> I have a hard time thinking that a geologist looking at this has performed 
> a quantum measurement that reduces the wave function then and there. It 
> does not do much to invoke life either, for I doubt that an unusually smart 
> trilobite or cockroach in the Paleozoic epoch would perform a measurement 
> with some conscious idea of QM. 
>
> ...

-- 
You received this message because you are subscribed to the Google Groups 
"Everything List" group.
To unsubscribe from this group and stop receiving emails from it, send an email 
to everything-list+unsubscr...@googlegroups.com.
To post to this group, send email to everything-list@googlegroups.com.
Visit this group at https://groups.google.com/group/everything-list.
For more options, visit https://groups.google.com/d/optout.


Re: Consistency of Postulates of QM

2017-12-23 Thread agrayson2000


On Saturday, December 23, 2017 at 2:11:32 PM UTC-7, Russell Standish wrote:
>
> On Sat, Dec 23, 2017 at 09:20:05AM -0800, agrays...@gmail.com 
>  wrote: 
> > 
> > My tentative solution to the wave collapse problem is to trash wave 
> > mechanics (which is not Lorentz invariant) and use Heisenberg's Matrix 
> > Mechanics. No waves, nothing to collapse. Is this a cop-out? AG 
>
> In matrix mechanics, the wave function is replaced by a vector, and 
> collapse is replaced by a projection onto a basis vector. 
>
> Projections are not unitary (except for the identity matrix), and that 
> is the problem with any collapse type theory. 
>

Thanks. That's clears enough. Collapse by another name. CMIIAW, but even if 
it were a unitary process, it would in effect be a local hidden variable, 
forbidden by results of Bell experiments. But let's talk about "unitary" 
which I think is equivalent to "linear". Why is non unitary, that is non 
linear bad? Because it means irreversible? I do believe that some 
measurement processes are in fact irreversible in principle, and not simply 
in the statistical sense, that is, FAPP.  IIRC, Bruce proved that for spin 
measurements on Avoid2, but it was not well received. AG 

-- 
You received this message because you are subscribed to the Google Groups 
"Everything List" group.
To unsubscribe from this group and stop receiving emails from it, send an email 
to everything-list+unsubscr...@googlegroups.com.
To post to this group, send email to everything-list@googlegroups.com.
Visit this group at https://groups.google.com/group/everything-list.
For more options, visit https://groups.google.com/d/optout.


Re: Consistency of Postulates of QM

2017-12-23 Thread Russell Standish
On Sat, Dec 23, 2017 at 09:20:05AM -0800, agrayson2...@gmail.com wrote:
> 
> My tentative solution to the wave collapse problem is to trash wave 
> mechanics (which is not Lorentz invariant) and use Heisenberg's Matrix 
> Mechanics. No waves, nothing to collapse. Is this a cop-out? AG 

In matrix mechanics, the wave function is replaced by a vector, and
collapse is replaced by a projection onto a basis vector.

Projections are not unitary (except for the identity matrix), and that
is the problem with any collapse type theory.



-- 


Dr Russell StandishPhone 0425 253119 (mobile)
Principal, High Performance Coders
Visiting Senior Research Fellowhpco...@hpcoders.com.au
Economics, Kingston University http://www.hpcoders.com.au


-- 
You received this message because you are subscribed to the Google Groups 
"Everything List" group.
To unsubscribe from this group and stop receiving emails from it, send an email 
to everything-list+unsubscr...@googlegroups.com.
To post to this group, send email to everything-list@googlegroups.com.
Visit this group at https://groups.google.com/group/everything-list.
For more options, visit https://groups.google.com/d/optout.


Re: Consistency of Postulates of QM

2017-12-23 Thread Bruno Marchal


On 22 Dec 2017, at 16:21, Lawrence Crowell wrote:




On Wednesday, December 20, 2017 at 6:36:39 AM UTC-6, Bruno Marchal  
wrote:


On 19 Dec 2017, at 20:08, agrays...@gmail.com wrote:



On Tuesday, December 19, 2017 at 4:48:48 PM UTC, Bruno Marchal wrote:

On 18 Dec 2017, at 00:34, agrays...@gmail.com wrote:

That makes things worst. Toy will make physical outcomes directly  
dependent of the consciousness/knowledge of the observer.


Why do you and Clark persist in this error? Collapse, if it occurs,  
does NOT depend on human consciousness.


OK but what you said made it depending of consciousness.  There is  
no collapse possible with the SWE, so what is the collapse? All  
theories introducing a collapse in between the consciousness of the  
observer and the observed objects seem highly speculative to me.


Measurement is just a specific instance of decoherence. There have  
been "effective measurements" for billions of years.


OK. But not just in the universal wave, also in arithmetic, and  
Mechanism forces them to be equivalent at some level, in a verifiable  
way/


What is proved is that in all theories rich enough to add and  
multiplied distinguishable inductive termes, like numbers, or  
combinators, universal numbers and universal combinators exists, and  
we have to retrieved physics from arithmetic using the self- 
referential abilities of the universal machines (the Löbian chatty one).




Chert or flint carry cosmic ray tracks, which indicate particles  
(likely muons) had their wave functions reduced with the interaction  
of the rock material many millions of years ago.


Not at all. When *we* do a measurement w just localized ourselves in  
the universal wave/matrix (it works in all pictures). The wave never  
collapse. I like feynman when he describe the collapse as a collective  
hallucination. It is exactly what Mechanism ensures.






I have a hard time thinking that a geologist looking at this has  
performed a quantum measurement that reduces the wave function then  
and there. It does not do much to invoke life either, for I doubt  
that an unusually smart trilobite or cockroach in the Paleozoic  
epoch would perform a measurement with some conscious idea of QM.


The measurement apparatus is a large number of quantum states which  
interact with a quantum system in some prepared state. This then  
results in the outcome of the needle or needle state. Ultimately I  
see this as a process whereby a set of qubits, thought of as quantum  
symbol strings or sets, encode quantum numbers. This then leads to  
an axiomatic incompleteness of QM, or the Schrodinger wave equation.  
We then have a breakdown of the quantum postulates as a sort of  
incompleteness of physical axioms to predict an actual outcome. One  
does not need to invoke consciousness to understand this.


If mechanism is true, the theology of the machine, including physics,  
is "theory" independent. You need only a theory capable of mimicking a  
(Turing) Universal machinery. Very elementary arithmetic is enough.  
The rest are the standard classical definition of belief, knowledge,  
and a notion of observabmle based on the mechanist thought experiences.


I often gives three explicit theories (Q, SK-combinator, and a  
Matiyazevic-Jones system of diophantine equations).


It is part of the mechanist mind-body problem to retrieve physics from  
any of those Turing complete theories. The advantage of using self- 
reference is that we get the difference between true, rationally  
communicable, rational knowledge, observable, etc, but also the logic  
of the non-rationally-justifiable, the non-ovservable, etc. Those are  
not empty due to the incompeleteness incarnated in the difference  
between two modal logics G and G* inherited by the intensional variant  
itself imposed by incompleteness.






Consciousness may however be some aspect of this self-reference.


You cannot limit it to anything "physical" or "empirical a priori",  
without cheating, and preventing the distinction between qualia and  
quanta.




It then could be possible that in some subtle way consciousness  
plays some role in the physical universe.


To be short: It creates it completely. NUMBER ==> CONSCIOUSNESS ===>  
PHYSICAL LAWS


Necessarily, when assuming a precise, weak, classical (in the logician  
sense) version of Mechanist hypothesis in the cognitive science (not  
in physics).




It might be in making measurements of physics in the earliest  
universe. This might serve to reduce quantum states appropriate for  
conscious life. Think of this as a sort of cosmic Wheeler Delayed  
Choice Experiment. There may be some ergodic principle at work as  
well, where an ensemble of IGUS/ET beings make measurements and the  
physical outcome is an mean of their measured outcomes.


I agree with many of what you say for quantum mechanics, except you  
seem to believe in some collapse of the wave, which I see only as an  
indexical memorable 

Re: Consistency of Postulates of QM

2017-12-23 Thread agrayson2000


On Friday, December 22, 2017 at 4:39:25 AM UTC-7, agrays...@gmail.com wrote:
>
>
>
> On Thursday, November 9, 2017 at 3:43:21 PM UTC, agrays...@gmail.com 
> wrote:
>>
>> If the measurement problem were solved in the sense being able to predict 
>> exact outcomes, thus making QM a deterministic theory, would that imply an 
>> INCONSISTENCY in the postulates of QM? TIA.
>>
>
> Does the Heisenberg or Feynman formulation of QM (Matrix Mechanics and 
> Path Integral respectively) have any issues analogous to the collapse 
> problem of Standard QM (aka the Copenhagen Interpretation)? TIA
>

My tentative solution to the wave collapse problem is to trash wave 
mechanics (which is not Lorentz invariant) and use Heisenberg's Matrix 
Mechanics. No waves, nothing to collapse. Is this a cop-out? AG 

-- 
You received this message because you are subscribed to the Google Groups 
"Everything List" group.
To unsubscribe from this group and stop receiving emails from it, send an email 
to everything-list+unsubscr...@googlegroups.com.
To post to this group, send email to everything-list@googlegroups.com.
Visit this group at https://groups.google.com/group/everything-list.
For more options, visit https://groups.google.com/d/optout.


Re: Consistency of Postulates of QM

2017-12-22 Thread Lawrence Crowell


On Wednesday, December 20, 2017 at 6:36:39 AM UTC-6, Bruno Marchal wrote:
>
>
> On 19 Dec 2017, at 20:08, agrays...@gmail.com  wrote:
>
>
>
> On Tuesday, December 19, 2017 at 4:48:48 PM UTC, Bruno Marchal wrote:
>
>
> On 18 Dec 2017, at 00:34, agrays...@gmail.com wrote:
>
>
> That makes things worst. Toy will make physical outcomes directly 
> dependent of the consciousness/knowledge of the observer.
>
>
> Why do you and Clark persist in this error? Collapse, if it occurs, does 
> NOT depend on human consciousness. 
>
>
> OK but what you said made it depending of consciousness.  There is no 
> collapse possible with the SWE, so what is the collapse? All theories 
> introducing a collapse in between the consciousness of the observer and the 
> observed objects seem highly speculative to me.
>
>
Measurement is just a specific instance of decoherence. There have been 
"effective measurements" for billions of years. Chert or flint carry cosmic 
ray tracks, which indicate particles (likely muons) had their wave 
functions reduced with the interaction of the rock material many millions 
of years ago. I have a hard time thinking that a geologist looking at this 
has performed a quantum measurement that reduces the wave function then and 
there. It does not do much to invoke life either, for I doubt that an 
unusually smart trilobite or cockroach in the Paleozoic epoch would perform 
a measurement with some conscious idea of QM. 

The measurement apparatus is a large number of quantum states which 
interact with a quantum system in some prepared state. This then results in 
the outcome of the needle or needle state. Ultimately I see this as a 
process whereby a set of qubits, thought of as quantum symbol strings or 
sets, encode quantum numbers. This then leads to an axiomatic 
incompleteness of QM, or the Schrodinger wave equation. We then have a 
breakdown of the quantum postulates as a sort of incompleteness of physical 
axioms to predict an actual outcome. One does not need to invoke 
consciousness to understand this.

Consciousness may however be some aspect of this self-reference. It then 
could be possible that in some subtle way consciousness plays some role in 
the physical universe. It might be in making measurements of physics in the 
earliest universe. This might serve to reduce quantum states appropriate 
for conscious life. Think of this as a sort of cosmic Wheeler Delayed 
Choice Experiment. There may be some ergodic principle at work as well, 
where an ensemble of IGUS/ET beings make measurements and the physical 
outcome is an mean of their measured outcomes. 

LC

-- 
You received this message because you are subscribed to the Google Groups 
"Everything List" group.
To unsubscribe from this group and stop receiving emails from it, send an email 
to everything-list+unsubscr...@googlegroups.com.
To post to this group, send email to everything-list@googlegroups.com.
Visit this group at https://groups.google.com/group/everything-list.
For more options, visit https://groups.google.com/d/optout.


Re: Consistency of Postulates of QM

2017-12-22 Thread agrayson2000


On Thursday, November 9, 2017 at 3:43:21 PM UTC, agrays...@gmail.com wrote:
>
> If the measurement problem were solved in the sense being able to predict 
> exact outcomes, thus making QM a deterministic theory, would that imply an 
> INCONSISTENCY in the postulates of QM? TIA.
>

Does the Heisenberg or Feynman formulation of QM (Matrix Mechanics and Path 
Integral respectively) have any issues analogous to the collapse problem of 
Standard QM (aka the Copenhagen Interpretation)? TIA

-- 
You received this message because you are subscribed to the Google Groups 
"Everything List" group.
To unsubscribe from this group and stop receiving emails from it, send an email 
to everything-list+unsubscr...@googlegroups.com.
To post to this group, send email to everything-list@googlegroups.com.
Visit this group at https://groups.google.com/group/everything-list.
For more options, visit https://groups.google.com/d/optout.


Re: Consistency of Postulates of QM

2017-12-21 Thread Bruno Marchal


On 20 Dec 2017, at 23:14, Brent Meeker wrote:




On 12/20/2017 5:09 AM, Bruno Marchal wrote:


On 19 Dec 2017, at 23:03, Brent Meeker wrote:




On 12/19/2017 9:47 AM, Bruno Marchal wrote:


On 18 Dec 2017, at 07:48, Brent Meeker wrote:




On 12/17/2017 8:06 AM, Bruno Marchal wrote:


On 15 Dec 2017, at 22:19, Brent Meeker wrote:




On 12/15/2017 9:24 AM, Bruno Marchal wrote:
that the statistics of the observable, in arithmetic from  
inside, have to "interfere" to make Digital Mechanism making  
sense in cognitive science, so MW-appearances is not bizarre  
at all: it has to be like that. Eventually, the "negative  
amplitude of probability" comes from the self-referential  
constraints (the logic of []p & <>p on p sigma_1, for those  
who have studied a little bit).


Can you explicate this.


Usually, notions like necessity, certainty, probability 1, etc.  
are assumed to obey []p -> p. This implies also []~p -> ~p, and  
thus p -> <>p, and so, if we have []p -> p, we have [] -> <>p  
(in classical normal modal logics).


Then provability, and even more "formal provability" was  
considered as as *the* closer notion to knowledge we could hope  
for,


Something a mathematician or logician might dream, but not a  
mistake any physicist would ever make. Knowledge is  
correspondence with reality, notdeducibility  
from axioms.


Which reality?

Since Gödel we do distinguish correspondence with the  
arithmetical reality and deducibilty from axioms. We know that  
*all* effective theories can only scratch the arithmetical truth.


You seem to identify reality with physical reality. That is a  
strong physicalist axiom. When doing metaphysics with the  
scientific method, especially on the mind-body problem, it is  
better to be more neutral.


I identify reality with what we can empirically agree on.


That is close to Aristotle metaphysics. It does not work with  
Mechanism.


But even without mechanism, I prefer to be metaphysically neutral,  
and identify reality with whatever is the reason why we can agree  
on an empirical reality. It does not need to be the empirical  
reality itself, as there is no evidence for that up to now.


That's the epitome of question begging:  "What you gonna take as  
evidence for empirical reality?  Your lyin' eyes or my beautiful  
theory?"


We must distinguish between the numbers that we measure, the relations  
that we infer, the metaphysical interpretation of those relations, etc.


We just need to be clear of what we assume at the start. You seem to  
assume an Aristotelian God (Primary Matter), and I assumed a  
Pythagorean God (elementary arithmetic).


Note that you assume also, at the meta level though, elementary  
arithmetic, to get predictions in the frame of your metaphysics. But  
you need an identiy thesis (à-la mind-brain) which does not work when  
assuming Mechanism (without adding Ptolemaic ad hoc metaphysical  
epicycles).







Would the logic S4Grz1, Z1* and X1* be different from quantum logic  
(soemthing we can measure) then we would have an evidence that the  
reality is the empirical reality, but up to now, it fits, and so  
there is no evidences that the physical empirical reality is the  
fundamental one.


Quantum mechanics is entirely based on empirical observation.   
Nobody proposed to derive it from meta-physics.


Yes, but with mechanism we have no choice, except for hiding the mind- 
body problem under the rug (a bad habit since a very long time).






















and so it came as a shock that no ("rich enough") theory can  
prove its own consistency. This means for example that neither  
ZF nor PA can prove ~[]f, that is []f -> f,


This seems to me incorrectly rely on []f->f  being equivalent to  
~f->~[]f and ~f=t.  I know that is standard first order logic,  
but in this case we're talking about the whole infinite set of  
expressible propositions.  It's not so clear to me that you can  
rely on the law of the excluded middle over this set.



We limit ourself to correct machine, by construction. It does not  
matter how they are implemented below their substitution level,  
and this is only what correct machine can prove on themselves at  
their correct substitution level, and any higher order correct 3p  
description.


That is all what we need to extract the "correct physics". No  
need to interview machines which believe they are Napoleons. I  
mean it is premature to invoke them in the fabric of the physical  
reality (despite it is unclear what is the part of possible lie  
at play here, cf Descarte's malin démons)








and so such machine cannot prove generally []p ->, and  
provability, for them, cannot works as a predicate for  
knowledge, and is at most a (hopefully correct) belief.


Now, this makes also possible to retrieve a classical notion of  
knowledge, by defining, for all arithmetical proposition p, the  
knowledge of p by []p & true(p).


I'm not impressed.


You should!

The 

Re: Consistency of Postulates of QM

2017-12-20 Thread Brent Meeker



On 12/20/2017 5:09 AM, Bruno Marchal wrote:


On 19 Dec 2017, at 23:03, Brent Meeker wrote:




On 12/19/2017 9:47 AM, Bruno Marchal wrote:


On 18 Dec 2017, at 07:48, Brent Meeker wrote:




On 12/17/2017 8:06 AM, Bruno Marchal wrote:


On 15 Dec 2017, at 22:19, Brent Meeker wrote:




On 12/15/2017 9:24 AM, Bruno Marchal wrote:
that the statistics of the observable, in arithmetic from 
inside, have to "interfere" to make Digital Mechanism making 
sense in cognitive science, so MW-appearances is not bizarre at 
all: it has to be like that. Eventually, the "negative amplitude 
of probability" comes from the self-referential constraints (the 
logic of []p & <>p on p sigma_1, for those who have studied a 
little bit).


Can you explicate this.


Usually, notions like necessity, certainty, probability 1, etc. 
are assumed to obey []p -> p. This implies also []~p -> ~p, and 
thus p -> <>p, and so, if we have []p -> p, we have [] -> <>p (in 
classical normal modal logics).


Then provability, and even more "formal provability" was 
considered as as *the* closer notion to knowledge we could hope for, 


Something a mathematician or logician might dream, but not a 
mistake any physicist would ever make. Knowledge is correspondence 
with reality, not deducibility from axioms.


Which reality?

Since Gödel we do distinguish correspondence with the arithmetical 
reality and deducibilty from axioms. We know that *all* effective 
theories can only scratch the arithmetical truth.


You seem to identify reality with physical reality. That is a strong 
physicalist axiom. When doing metaphysics with the scientific 
method, especially on the mind-body problem, it is better to be more 
neutral.


I identify reality with what we can empirically agree on.


That is close to Aristotle metaphysics. It does not work with Mechanism.

But even without mechanism, I prefer to be metaphysically neutral, and 
identify reality with whatever is the reason why we can agree on an 
empirical reality. It does not need to be the empirical reality 
itself, as there is no evidence for that up to now.


That's the epitome of question begging:  "What you gonna take as 
evidence for empirical reality?  Your lyin' eyes or my beautiful theory?"


Would the logic S4Grz1, Z1* and X1* be different from quantum logic 
(soemthing we can measure) then we would have an evidence that the 
reality is the empirical reality, but up to now, it fits, and so there 
is no evidences that the physical empirical reality is the fundamental 
one.


Quantum mechanics is entirely based on empirical observation. Nobody 
proposed to derive it from meta-physics.

















and so it came as a shock that no ("rich enough") theory can prove 
its own consistency. This means for example that neither ZF nor PA 
can prove ~[]f, that is []f -> f, 


This seems to me incorrectly rely on []f->f being equivalent to 
~f->~[]f and ~f=t.  I know that is standard first order logic, but 
in this case we're talking about the whole infinite set of 
expressible propositions.  It's not so clear to me that you can 
rely on the law of the excluded middle over this set.



We limit ourself to correct machine, by construction. It does not 
matter how they are implemented below their substitution level, and 
this is only what correct machine can prove on themselves at their 
correct substitution level, and any higher order correct 3p description.


That is all what we need to extract the "correct physics". No need 
to interview machines which believe they are Napoleons. I mean it is 
premature to invoke them in the fabric of the physical reality 
(despite it is unclear what is the part of possible lie at play 
here, cf Descarte's malin démons)








and so such machine cannot prove generally []p ->, and 
provability, for them, cannot works as a predicate for knowledge, 
and is at most a (hopefully correct) belief.


Now, this makes also possible to retrieve a classical notion of 
knowledge, by defining, for all arithmetical proposition p, the 
knowledge of p by []p & true(p).


I'm not impressed.


You should!

The beauty is that "Bp & p" leads to an explanation of why the 
machine get suck in infinities when trying to know who she is. from 
the machine's view, this looks quite like a soul, or subject of 
consciousness, which "of course" cannot justify any 3p account of 
him. from its point of view, the doppelganger is a construction 
which proves that he is not a machine, and that the doppelganger is 
an impostor! The beauty of "Bp & p" is that it says "no" to the 
doctor! The machine's elementary first insight is that she is no 
machines at all, and she is right from that points of view, as G* 
can justify.







Unfortunately, we cannot define true(p) in arithmetic (Tarski), 
nor can we define knowledge at all (Thomason, Scott-Montague). But 
for eaxh arithmetical p, we can still mimic knowledge by []p & p, 


Since you can't define knowledge, how can you say you can mimic it?



Re: Consistency of Postulates of QM

2017-12-20 Thread Bruno Marchal


On 19 Dec 2017, at 23:03, Brent Meeker wrote:




On 12/19/2017 9:47 AM, Bruno Marchal wrote:


On 18 Dec 2017, at 07:48, Brent Meeker wrote:




On 12/17/2017 8:06 AM, Bruno Marchal wrote:


On 15 Dec 2017, at 22:19, Brent Meeker wrote:




On 12/15/2017 9:24 AM, Bruno Marchal wrote:
that the statistics of the observable, in arithmetic from  
inside, have to "interfere" to make Digital Mechanism making  
sense in cognitive science, so MW-appearances is not bizarre at  
all: it has to be like that. Eventually, the "negative  
amplitude of probability" comes from the self-referential  
constraints (the logic of []p & <>p on p sigma_1, for those who  
have studied a little bit).


Can you explicate this.


Usually, notions like necessity, certainty, probability 1, etc.  
are assumed to obey []p -> p. This implies also []~p -> ~p, and  
thus p -> <>p, and so, if we have []p -> p, we have [] -> <>p (in  
classical normal modal logics).


Then provability, and even more "formal provability" was  
considered as as *the* closer notion to knowledge we could hope  
for,


Something a mathematician or logician might dream, but not a  
mistake any physicist would ever make. Knowledge is correspondence  
with reality, not deducibility from axioms.


Which reality?

Since Gödel we do distinguish correspondence with the arithmetical  
reality and deducibilty from axioms. We know that *all* effective  
theories can only scratch the arithmetical truth.


You seem to identify reality with physical reality. That is a  
strong physicalist axiom. When doing metaphysics with the  
scientific method, especially on the mind-body problem, it is  
better to be more neutral.


I identify reality with what we can empirically agree on.


That is close to Aristotle metaphysics. It does not work with Mechanism.

But even without mechanism, I prefer to be metaphysically neutral, and  
identify reality with whatever is the reason why we can agree on an  
empirical reality. It does not need to be the empirical reality  
itself, as there is no evidence for that up to now. Would the logic  
S4Grz1, Z1* and X1* be different from quantum logic (soemthing we can  
measure) then we would have an evidence that the reality is the  
empirical reality, but up to now, it fits, and so there is no  
evidences that the physical empirical reality is the fundamental one.















and so it came as a shock that no ("rich enough") theory can  
prove its own consistency. This means for example that neither ZF  
nor PA can prove ~[]f, that is []f -> f,


This seems to me incorrectly rely on []f->f  being equivalent to  
~f->~[]f and ~f=t.  I know that is standard first order logic, but  
in this case we're talking about the whole infinite set of  
expressible propositions.  It's not so clear to me that you can  
rely on the law of the excluded middle over this set.



We limit ourself to correct machine, by construction. It does not  
matter how they are implemented below their substitution level, and  
this is only what correct machine can prove on themselves at their  
correct substitution level, and any higher order correct 3p  
description.


That is all what we need to extract the "correct physics". No need  
to interview machines which believe they are Napoleons. I mean it  
is premature to invoke them in the fabric of the physical reality  
(despite it is unclear what is the part of possible lie at play  
here, cf Descarte's malin démons)








and so such machine cannot prove generally []p ->, and  
provability, for them, cannot works as a predicate for knowledge,  
and is at most a (hopefully correct) belief.


Now, this makes also possible to retrieve a classical notion of  
knowledge, by defining, for all arithmetical proposition p, the  
knowledge of p by []p & true(p).


I'm not impressed.


You should!

The beauty is that "Bp & p" leads to an explanation of why the  
machine get suck in infinities when trying to know who she is. from  
the machine's view, this looks quite like a soul, or subject of  
consciousness, which "of course" cannot justify any 3p account of  
him. from its point of view, the doppelganger is a construction  
which proves that he is not a machine, and that the doppelganger is  
an impostor! The beauty of "Bp & p" is that it says "no" to the  
doctor! The machine's elementary first insight is that she is no  
machines at all, and she is right from that points of view, as G*  
can justify.







Unfortunately, we cannot define true(p) in arithmetic (Tarski),  
nor can we define knowledge at all (Thomason, Scott-Montague).  
But for eaxh arithmetical p, we can still mimic knowledge by []p  
& p,


Since you can't define knowledge, how can you say you can mimic it?


All (serious) philosophers agree that knowledge is well axiomatized  
by the modal logic T and S4 (T + Bp -> BBp).


I've had Edmund Gettier over for dinner and he definitely does not  
agree with this idea of knowledge.  If I'm right in assuming that 

Re: Consistency of Postulates of QM

2017-12-20 Thread Bruno Marchal


On 19 Dec 2017, at 22:40, Brent Meeker wrote:




On 12/19/2017 9:11 AM, Bruno Marchal wrote:


On 18 Dec 2017, at 07:17, Brent Meeker wrote:




On 12/17/2017 7:32 AM, Bruno Marchal wrote:

  But in fact the box is not isolated.


Oh? Just isolate the whole universe. That should be easy.




The box too is interacting with the environment.  So it's like  
the Zeno effect.  Although there is a probability at each impact  
of producing a coherent tails component, those components don't  
sum to a finite component over a finite number of impacts.


The Zeno effect makes you "staying statistically" in the  
universe, like the non-isolation of the box makes you impossible  
to have access to the universe where the coin felt on the  
opposite side, but without collapse, the superposition can simply  
never disappear.


I can never disappear, but it cannot reach a significant  
probability for tails in several ages of the universe.


I am not good with unities. My last attempt to evaluate the time to  
get the six outcomes of a dice, in a lattice version of the problem  
lead to only few minutes. When I have the time and the courage, I  
will compute that again. Note that the wall was an irregular  
lattice, to ensure that the dices did not get trapped in a cycle.  
It did use some "quantum chaos" idea if I remember well. Of course,  
we do agree that there is no quantum chaos, and everything is  
reversible, but all explanations that this still plays a role FAPP,  
by partial tracing out + big numbers, can be translated into the  
"many-world" as explanation of how feeling the splits and keeping  
trace of the "other terms of the wave) is close to impossible.


I might be wrong on this. No problem. Shaking the box for several  
ages of the universe would astonished me (and force me to revise  
quantum chaos theory, if not the prime numbers!). Or it could be my  
poor handling of the scientific notation of real numbers, common  
for *some* "pure mathematician" (yellow grin).


Keep in mind that all object diffuse, so, even without shacking the  
dice, if we wait long enough, it diffuses so as we get in few  
shakings the six alternate worlds, but I guess that in this case,  
we do have to wait a very long time. The bouncing of the dies are  
crucial, as is the shape of the box.


But remember that you have to consider the classical processes as  
well, which compete with quantum diffusion.


Relatively to the mind of each multiple observers. OK. But that is  
what brings the superposed macro-differences.





You may say, "Well it's all quantum." but that's not to the point.   
The classical processes are different in that they can, in  
principle, be measured without changing them by making measurements  
on the environment.


OK. So our problem is only on the quantitative. We might tackle it one  
day.


Bruno





Brent

--
You received this message because you are subscribed to the Google  
Groups "Everything List" group.
To unsubscribe from this group and stop receiving emails from it,  
send an email to everything-list+unsubscr...@googlegroups.com.

To post to this group, send email to everything-list@googlegroups.com.
Visit this group at https://groups.google.com/group/everything-list.
For more options, visit https://groups.google.com/d/optout.


http://iridia.ulb.ac.be/~marchal/



--
You received this message because you are subscribed to the Google Groups 
"Everything List" group.
To unsubscribe from this group and stop receiving emails from it, send an email 
to everything-list+unsubscr...@googlegroups.com.
To post to this group, send email to everything-list@googlegroups.com.
Visit this group at https://groups.google.com/group/everything-list.
For more options, visit https://groups.google.com/d/optout.


Re: Consistency of Postulates of QM

2017-12-20 Thread Bruno Marchal


On 19 Dec 2017, at 20:08, agrayson2...@gmail.com wrote:




On Tuesday, December 19, 2017 at 4:48:48 PM UTC, Bruno Marchal wrote:

On 18 Dec 2017, at 00:34, agrays...@gmail.com wrote:

On Sunday, December 17, 2017 at 10:28:17 PM UTC,  
agrays...@gmail.com wrote:


On Sunday, December 17, 2017 at 3:26:05 PM UTC, Bruno Marchal wrote:

On 15 Dec 2017, at 23:54, agrays...@gmail.com wrote:

On Friday, December 15, 2017 at 5:24:39 PM UTC, Bruno Marchal wrote:

On 14 Dec 2017, at 03:01, agrays...@gmail.com wrote:

On Thursday, December 14, 2017 at 1:41:37 AM UTC, Brent wrote:

On 12/13/2017 5:24 PM, agrays...@gmail.com wrote:

On Wednesday, December 13, 2017 at 10:44:14 PM UTC, Brent wrote:

On 12/13/2017 2:20 PM, agrays...@gmail.com wrote:

On Wednesday, December 13, 2017 at 9:15:36 PM UTC, Brent wrote:


On 12/13/2017 2:45 AM, agrays...@gmail.com wrote:
BUT for a nucleus of a radioactive element, the nucleus is never  
Decayed and Undecayed SIMULTANEOUSLY.
Sure it is.  It's in a coherent superposition of those states  
until it interacts with the environment.


Brent

That's the conventional QM wisdom and the cause of the paradox of  
a cat Alive and Dead simultaneously. As I explained, the fallacy  
is rooted in an unjustified generalization of the double slit  
experiment where the probability waves do, in fact, exist  
simultaneously.  What waves do you claim are interacting for the  
radioactive nucleus to produce coherence? Tell me about them. I  
am from Missouri. AG


You seem to think that coherence requires two different waves.   
This is the wrong way to look at it.  In Young's slits experiment  
there is only one wave, which goes through both slits and  
interferes with itself.


That's exactly how I see it! Interference requires two waves  
which interact with each other.


NO.  This is false!  There are not two waves.  You can write it  
as two parts, just as you can write a description of an ocean  
wave as the part on your left and the part on your right.  But so  
long as they are coherent, maintaining a fixed phase relation,  
they are one wave.


You're splitting hairs, engaging in sophism. For the single wave  
going through both slits, Feynman calculates the norm squared of  
| A + B |, where A and B denote the waveS going through left and  
right slits respectively. Both are obviously identical, with the  
result of coherent interference. From this analysis we get the  
interpretation that the the system is simultaneously in all  
states of a superposition. AG


Noteworthy is that fact that if you reference "coherence" on  
Wiki, the description always invokes multiple waves of the same  
frequency. If you want to assert coherence without multiple  
waves, and NOT using the double slit result, you have some heavy  
lifting to do. AG



This is exactly what we see in Young's slits experiments. AG

And unstable nucleus has a probability amplitude that includes a  
"decayed" part and a "not decayed" part.  It's a tunneling  
problem.


I don't doubt the existence of amplitudes. What I do doubt. and  
in fact deny, is interference between two waves that don't exist  
simultaneously.


You keep referring to two waves.  There are not two waves.   
There's only one wave which interferes with itself.  It is  
typically written as |not-decayed> + |decayed>, but that's just a  
choice of basis.  It could as well be written |unstable nucleus>.


OK, unstable nucleus. Makes no difference to what I am arguing;  
namely, that coherence requires more than one wave,  
simultaneously, which is what double slit SHOWS, even though the  
experiment obviously starts out with one wave. AG


If there's no interference, then the cat cannot be Alive and  
Dead simultaneously. Tunneling can exist, but still, no  
simultaneous interacting, interfering waves. Is there any  
advantage to believing in two waves which don't exist  
simultaneous, can interfere with each other? AG


You are confused.

You're the one with a cat which Alive and Dead simultaneously for  
the very short time until decoherence occurs. So it is arguable  
who is really confused. AG

I agree with Brent. It is elementary quantum mechanics.

I seriously doubt you have a clue what we're discussing. If you  
take the time to read any definition of 'coherence', you will see  
it involves two or more interfering waves.
You can use two, or one wave. All you need is to gives them a  
similar phase, and it is easy to use only one wave, like the one  
going through two slit when sending, even one, photon.


You have an elaborate theory of the universe based on arithmetic  
but have difficulty counting to two. A single wave cannot exhibit  
interference. You need TWO waves! In double slit experiment, the  
slits split the original wave into TWO waves which interfere with  
each other, yielding the interference pattern. Got it? So the  
question is, how does one get interference in an experiment with a  
binary result? You can call it what you want, or 

Re: Consistency of Postulates of QM

2017-12-19 Thread Brent Meeker



On 12/19/2017 9:47 AM, Bruno Marchal wrote:


On 18 Dec 2017, at 07:48, Brent Meeker wrote:




On 12/17/2017 8:06 AM, Bruno Marchal wrote:


On 15 Dec 2017, at 22:19, Brent Meeker wrote:




On 12/15/2017 9:24 AM, Bruno Marchal wrote:
that the statistics of the observable, in arithmetic from inside, 
have to "interfere" to make Digital Mechanism making sense in 
cognitive science, so MW-appearances is not bizarre at all: it has 
to be like that. Eventually, the "negative amplitude of 
probability" comes from the self-referential constraints (the 
logic of []p & <>p on p sigma_1, for those who have studied a 
little bit).


Can you explicate this.


Usually, notions like necessity, certainty, probability 1, etc. are 
assumed to obey []p -> p. This implies also []~p -> ~p, and thus p 
-> <>p, and so, if we have []p -> p, we have [] -> <>p (in classical 
normal modal logics).


Then provability, and even more "formal provability" was considered 
as as *the* closer notion to knowledge we could hope for, 


Something a mathematician or logician might dream, but not a mistake 
any physicist would ever make. Knowledge is correspondence with 
reality, not deducibility from axioms.


Which reality?

Since Gödel we do distinguish correspondence with the arithmetical 
reality and deducibilty from axioms. We know that *all* effective 
theories can only scratch the arithmetical truth.


You seem to identify reality with physical reality. That is a strong 
physicalist axiom. When doing metaphysics with the scientific method, 
especially on the mind-body problem, it is better to be more neutral.


I identify reality with what we can empirically agree on.









and so it came as a shock that no ("rich enough") theory can prove 
its own consistency. This means for example that neither ZF nor PA 
can prove ~[]f, that is []f -> f, 


This seems to me incorrectly rely on []f->f  being equivalent to 
~f->~[]f and ~f=t.  I know that is standard first order logic, but in 
this case we're talking about the whole infinite set of expressible 
propositions.  It's not so clear to me that you can rely on the law 
of the excluded middle over this set.



We limit ourself to correct machine, by construction. It does not 
matter how they are implemented below their substitution level, and 
this is only what correct machine can prove on themselves at their 
correct substitution level, and any higher order correct 3p description.


That is all what we need to extract the "correct physics". No need to 
interview machines which believe they are Napoleons. I mean it is 
premature to invoke them in the fabric of the physical reality 
(despite it is unclear what is the part of possible lie at play here, 
cf Descarte's malin démons)








and so such machine cannot prove generally []p ->, and provability, 
for them, cannot works as a predicate for knowledge, and is at most 
a (hopefully correct) belief.


Now, this makes also possible to retrieve a classical notion of 
knowledge, by defining, for all arithmetical proposition p, the 
knowledge of p by []p & true(p).


I'm not impressed.


You should!

The beauty is that "Bp & p" leads to an explanation of why the machine 
get suck in infinities when trying to know who she is. from the 
machine's view, this looks quite like a soul, or subject of 
consciousness, which "of course" cannot justify any 3p account of him. 
from its point of view, the doppelganger is a construction which 
proves that he is not a machine, and that the doppelganger is an 
impostor! The beauty of "Bp & p" is that it says "no" to the doctor! 
The machine's elementary first insight is that she is no machines at 
all, and she is right from that points of view, as G* can justify.







Unfortunately, we cannot define true(p) in arithmetic (Tarski), nor 
can we define knowledge at all (Thomason, Scott-Montague). But for 
eaxh arithmetical p, we can still mimic knowledge by []p & p, 


Since you can't define knowledge, how can you say you can mimic it?


All (serious) philosophers agree that knowledge is well axiomatized by 
the modal logic T and S4 (T + Bp -> BBp).


I've had Edmund Gettier over for dinner and he definitely does not agree 
with this idea of knowledge.  If I'm right in assuming that T means "true".




"Bp & p", applied to the K4 reasoner (close to full self-referential 
ability) gives S4, and is called "the standard theory of knowledge" by 
both scholars in antic philosophy, and in artificial intelligence (a 
rare agreement in philosophy).


What the machine cannot do is to define itself as a knower. That is 
why she will be unable to recognize itself at any substitution level, 
and that is why she will have to trust the doctor, or prey, because 
nobody can tell her who she is, and which computations support her in 
arithmetic.








for each p, and this lead to a way to associate canonically a knower 
to the machine-prover. It obeys to a knowledge logic (with []p -> p 
becoming trivial). That logic 

Re: Consistency of Postulates of QM

2017-12-19 Thread Brent Meeker



On 12/19/2017 9:11 AM, Bruno Marchal wrote:


On 18 Dec 2017, at 07:17, Brent Meeker wrote:




On 12/17/2017 7:32 AM, Bruno Marchal wrote:

  But in fact the box is not isolated.


Oh? Just isolate the whole universe. That should be easy.




The box too is interacting with the environment.  So it's like the 
Zeno effect.  Although there is a probability at each impact of 
producing a coherent tails component, those components don't sum to 
a finite component over a finite number of impacts.


The Zeno effect makes you "staying statistically" in the universe, 
like the non-isolation of the box makes you impossible to have 
access to the universe where the coin felt on the opposite side, but 
without collapse, the superposition can simply never disappear.


I can never disappear, but it cannot reach a significant probability 
for tails in several ages of the universe.


I am not good with unities. My last attempt to evaluate the time to 
get the six outcomes of a dice, in a lattice version of the problem 
lead to only few minutes. When I have the time and the courage, I will 
compute that again. Note that the wall was an irregular lattice, to 
ensure that the dices did not get trapped in a cycle. It did use some 
"quantum chaos" idea if I remember well. Of course, we do agree that 
there is no quantum chaos, and everything is reversible, but all 
explanations that this still plays a role FAPP, by partial tracing out 
+ big numbers, can be translated into the "many-world" as explanation 
of how feeling the splits and keeping trace of the "other terms of the 
wave) is close to impossible.


I might be wrong on this. No problem. Shaking the box for several ages 
of the universe would astonished me (and force me to revise quantum 
chaos theory, if not the prime numbers!). Or it could be my poor 
handling of the scientific notation of real numbers, common for *some* 
"pure mathematician" (yellow grin).


Keep in mind that all object diffuse, so, even without shacking the 
dice, if we wait long enough, it diffuses so as we get in few shakings 
the six alternate worlds, but I guess that in this case, we do have to 
wait a very long time. The bouncing of the dies are crucial, as is the 
shape of the box.


But remember that you have to consider the classical processes as well, 
which compete with quantum diffusion.  You may say, "Well it's all 
quantum." but that's not to the point.  The classical processes are 
different in that they can, in principle, be measured without changing 
them by making measurements on the environment.


Brent

--
You received this message because you are subscribed to the Google Groups 
"Everything List" group.
To unsubscribe from this group and stop receiving emails from it, send an email 
to everything-list+unsubscr...@googlegroups.com.
To post to this group, send email to everything-list@googlegroups.com.
Visit this group at https://groups.google.com/group/everything-list.
For more options, visit https://groups.google.com/d/optout.


Re: Consistency of Postulates of QM

2017-12-19 Thread agrayson2000


On Tuesday, December 19, 2017 at 4:48:48 PM UTC, Bruno Marchal wrote:
>
>
> On 18 Dec 2017, at 00:34, agrays...@gmail.com  wrote:
>
> On Sunday, December 17, 2017 at 10:28:17 PM UTC, agrays...@gmail.com 
> wrote:
>
>>
>> On Sunday, December 17, 2017 at 3:26:05 PM UTC, Bruno Marchal wrote:
>>>
>>>
>>> On 15 Dec 2017, at 23:54, agrays...@gmail.com wrote:
>>>
>>> On Friday, December 15, 2017 at 5:24:39 PM UTC, Bruno Marchal wrote:


 On 14 Dec 2017, at 03:01, agrays...@gmail.com wrote:

 On Thursday, December 14, 2017 at 1:41:37 AM UTC, Brent wrote:

>
> On 12/13/2017 5:24 PM, agrays...@gmail.com wrote:
>
> On Wednesday, December 13, 2017 at 10:44:14 PM UTC, Brent wrote:
>
>>
>> On 12/13/2017 2:20 PM, agrays...@gmail.com wrote:
>>
>> On Wednesday, December 13, 2017 at 9:15:36 PM UTC, Brent wrote: 
>>>
>>>
>>>
>>> On 12/13/2017 2:45 AM, agrays...@gmail.com wrote:
>>>
>>> * BUT for a nucleus of a radioactive element, the nucleus is never 
>>> Decayed and Undecayed SIMULTANEOUSLY.*
>>>
>>> Sure it is.  It's in a coherent superposition of those states until 
>>> it interacts with the environment.
>>>
>>> Brent
>>>
>>
>> * That's the conventional QM wisdom and the cause of the paradox of a 
>> cat Alive and Dead simultaneously. As I explained, the fallacy is rooted 
>> in 
>> an unjustified generalization of the double slit experiment where the 
>> probability waves do, in fact, exist simultaneously.  What waves do you 
>> claim are interacting for the radioactive nucleus to produce coherence? 
>> Tell me about them. I am from Missouri. AG*
>>
>>
>> You seem to think that coherence requires two different waves.  This 
>> is the wrong way to look at it.  In Young's slits experiment there is 
>> only 
>> one wave, which goes through both slits and interferes with itself.  
>>
>
>
> *That's exactly how I see it! Interference requires two waves which 
> interact with each other. *
>
>
> *NO.  This is false! * *There are not two waves.*  You can write it 
> as two parts, just as you can write a description of an ocean wave as the 
> part on your left and the part on your right.  But so long as they are 
> coherent, maintaining a fixed phase relation, they are one wave.
>



 *You're splitting hairs, engaging in sophism. For the single wave going 
 through both slits, Feynman calculates the norm squared of | A + B |, 
 where 
 A and B denote the waveS going through left and right slits respectively. 
 Both are obviously identical, with the result of coherent interference. 
 From this analysis we get the interpretation that the the system is 
 simultaneously in all states of a superposition. AGNoteworthy is that fact 
 that if you reference "coherence" on Wiki, the description always invokes 
 multiple waves of the same frequency. If you want to assert coherence 
 without multiple waves, and NOT using the double slit result, you have 
 some 
 heavy lifting to do. AG*

>
>
> *This is exactly what we see in Young's slits experiments. AG  *
>
>> And unstable nucleus has a probability amplitude that includes a 
>> "decayed" part and a "not decayed" part.  It's a tunneling problem.
>>
>
> *I don't doubt the existence of amplitudes. What I do doubt. and in 
> fact deny, is interference between two waves that don't exist 
> simultaneously. *
>
>
> You keep referring to two waves. * There are not two waves.  *There's 
> only one wave which interferes with itself.  It is typically written as 
> |not-decayed> + |decayed>, but that's just a choice of basis.  It could 
> as 
> well be written |unstable nucleus>.
>

 *OK, unstable nucleus. Makes no difference to what I am arguing; 
 namely, that coherence requires more than one wave, simultaneously, which 
 is what double slit SHOWS, even though the experiment obviously starts out 
 with one wave. AG* 

>
> *If there's no interference, then the cat cannot be Alive and Dead 
> simultaneously. Tunneling can exist, but still, no simultaneous 
> interacting, interfering waves. Is there any advantage to believing in 
> two 
> waves which don't exist simultaneous, can interfere with each other? AG*
>
>
> You are confused.
>

 You're the one with a cat which Alive and Dead simultaneously for the 
 very short time until decoherence occurs. So it is arguable who is really 
 confused. AG 

 I agree with Brent. It is elementary quantum mechanics.

 *I seriously doubt you have a clue what we're discussing. If you take 
 the time to read any definition of 'coherence', you will see it involves 
 two or more interfering waves.*

>>> You can 

Re: Consistency of Postulates of QM

2017-12-19 Thread Bruno Marchal


On 18 Dec 2017, at 07:48, Brent Meeker wrote:




On 12/17/2017 8:06 AM, Bruno Marchal wrote:


On 15 Dec 2017, at 22:19, Brent Meeker wrote:




On 12/15/2017 9:24 AM, Bruno Marchal wrote:
that the statistics of the observable, in arithmetic from inside,  
have to "interfere" to make Digital Mechanism making sense in  
cognitive science, so MW-appearances is not bizarre at all: it  
has to be like that. Eventually, the "negative amplitude of  
probability" comes from the self-referential constraints (the  
logic of []p & <>p on p sigma_1, for those who have studied a  
little bit).


Can you explicate this.


Usually, notions like necessity, certainty, probability 1, etc. are  
assumed to obey []p -> p. This implies also []~p -> ~p, and thus p - 
> <>p, and so, if we have []p -> p, we have [] -> <>p (in classical  
normal modal logics).


Then provability, and even more "formal provability" was considered  
as as *the* closer notion to knowledge we could hope for,


Something a mathematician or logician might dream, but not a mistake  
any physicist would ever make. Knowledge is correspondence with  
reality, not deducibility from axioms.


Which reality?

Since Gödel we do distinguish correspondence with the arithmetical  
reality and deducibilty from axioms. We know that *all* effective  
theories can only scratch the arithmetical truth.


You seem to identify reality with physical reality. That is a strong  
physicalist axiom. When doing metaphysics with the scientific method,  
especially on the mind-body problem, it is better to be more neutral.








and so it came as a shock that no ("rich enough") theory can prove  
its own consistency. This means for example that neither ZF nor PA  
can prove ~[]f, that is []f -> f,


This seems to me incorrectly rely on []f->f  being equivalent to ~f- 
>~[]f and ~f=t.  I know that is standard first order logic, but in  
this case we're talking about the whole infinite set of expressible  
propositions.  It's not so clear to me that you can rely on the law  
of the excluded middle over this set.



We limit ourself to correct machine, by construction. It does not  
matter how they are implemented below their substitution level, and  
this is only what correct machine can prove on themselves at their  
correct substitution level, and any higher order correct 3p description.


That is all what we need to extract the "correct physics". No need to  
interview machines which believe they are Napoleons. I mean it is  
premature to invoke them in the fabric of the physical reality  
(despite it is unclear what is the part of possible lie at play here,  
cf Descarte's malin démons)








and so such machine cannot prove generally []p ->, and provability,  
for them, cannot works as a predicate for knowledge, and is at most  
a (hopefully correct) belief.


Now, this makes also possible to retrieve a classical notion of  
knowledge, by defining, for all arithmetical proposition p, the  
knowledge of p by []p & true(p).


I'm not impressed.


You should!

The beauty is that "Bp & p" leads to an explanation of why the machine  
get suck in infinities when trying to know who she is. from the  
machine's view, this looks quite like a soul, or subject of  
consciousness, which "of course" cannot justify any 3p account of him.  
from its point of view, the doppelganger is a construction which  
proves that he is not a machine, and that the doppelganger is an  
impostor! The beauty of "Bp & p" is that it says "no" to the doctor!  
The machine's elementary first insight is that she is no machines at  
all, and she is right from that points of view, as G* can justify.







Unfortunately, we cannot define true(p) in arithmetic (Tarski), nor  
can we define knowledge at all (Thomason, Scott-Montague). But for  
eaxh arithmetical p, we can still mimic knowledge by []p & p,


Since you can't define knowledge, how can you say you can mimic it?


All (serious) philosophers agree that knowledge is well axiomatized by  
the modal logic T and S4 (T + Bp -> BBp).


"Bp & p", applied to the K4 reasoner (close to full self-referential  
ability) gives S4, and is called "the standard theory of knowledge" by  
both scholars in antic philosophy, and in artificial intelligence (a  
rare agreement in philosophy).


What the machine cannot do is to define itself as a knower. That is  
why she will be unable to recognize itself at any substitution level,  
and that is why she will have to trust the doctor, or prey, because  
nobody can tell her who she is, and which computations support her in  
arithmetic.








for each p, and this lead to a way to associate canonically a  
knower to the machine-prover. It obeys to a knowledge logic (with  
[]p -> p becoming trivial). That logic is captured soundly and  
completely by the logic S4Grz (already described in many posts).


Similarly, the logic G of arithmetical self-reference cannot be a  
logic of probability one, due to the fact that []p 

Re: Consistency of Postulates of QM

2017-12-19 Thread Bruno Marchal


On 18 Dec 2017, at 07:17, Brent Meeker wrote:




On 12/17/2017 7:32 AM, Bruno Marchal wrote:

  But in fact the box is not isolated.


Oh? Just isolate the whole universe. That should be easy.




The box too is interacting with the environment.  So it's like the  
Zeno effect.  Although there is a probability at each impact of  
producing a coherent tails component, those components don't sum  
to a finite component over a finite number of impacts.


The Zeno effect makes you "staying statistically" in the universe,  
like the non-isolation of the box makes you impossible to have  
access to the universe where the coin felt on the opposite side,  
but without collapse, the superposition can simply never disappear.


I can never disappear, but it cannot reach a significant probability  
for tails in several ages of the universe.


I am not good with unities. My last attempt to evaluate the time to  
get the six outcomes of a dice, in a lattice version of the problem  
lead to only few minutes. When I have the time and the courage, I will  
compute that again. Note that the wall was an irregular lattice, to  
ensure that the dices did not get trapped in a cycle. It did use some  
"quantum chaos" idea if I remember well. Of course, we do agree that  
there is no quantum chaos, and everything is reversible, but all  
explanations that this still plays a role FAPP, by partial tracing out  
+ big numbers, can be translated into the "many-world" as explanation  
of how feeling the splits and keeping trace of the "other terms of the  
wave) is close to impossible.


I might be wrong on this. No problem. Shaking the box for several ages  
of the universe would astonished me (and force me to revise quantum  
chaos theory, if not the prime numbers!). Or it could be my poor  
handling of the scientific notation of real numbers, common for *some*  
"pure mathematician" (yellow grin).


Keep in mind that all object diffuse, so, even without shacking the  
dice, if we wait long enough, it diffuses so as we get in few shakings  
the six alternate worlds, but I guess that in this case, we do have to  
wait a very long time. The bouncing of the dies are crucial, as is the  
shape of the box.


Bruno





Brent

--
You received this message because you are subscribed to the Google  
Groups "Everything List" group.
To unsubscribe from this group and stop receiving emails from it,  
send an email to everything-list+unsubscr...@googlegroups.com.

To post to this group, send email to everything-list@googlegroups.com.
Visit this group at https://groups.google.com/group/everything-list.
For more options, visit https://groups.google.com/d/optout.


http://iridia.ulb.ac.be/~marchal/



--
You received this message because you are subscribed to the Google Groups 
"Everything List" group.
To unsubscribe from this group and stop receiving emails from it, send an email 
to everything-list+unsubscr...@googlegroups.com.
To post to this group, send email to everything-list@googlegroups.com.
Visit this group at https://groups.google.com/group/everything-list.
For more options, visit https://groups.google.com/d/optout.


Re: Consistency of Postulates of QM

2017-12-19 Thread Bruno Marchal


On 18 Dec 2017, at 00:34, agrayson2...@gmail.com wrote:

On Sunday, December 17, 2017 at 10:28:17 PM UTC, agrays...@gmail.com  
wrote:


On Sunday, December 17, 2017 at 3:26:05 PM UTC, Bruno Marchal wrote:

On 15 Dec 2017, at 23:54, agrays...@gmail.com wrote:

On Friday, December 15, 2017 at 5:24:39 PM UTC, Bruno Marchal wrote:

On 14 Dec 2017, at 03:01, agrays...@gmail.com wrote:

On Thursday, December 14, 2017 at 1:41:37 AM UTC, Brent wrote:

On 12/13/2017 5:24 PM, agrays...@gmail.com wrote:

On Wednesday, December 13, 2017 at 10:44:14 PM UTC, Brent wrote:

On 12/13/2017 2:20 PM, agrays...@gmail.com wrote:

On Wednesday, December 13, 2017 at 9:15:36 PM UTC, Brent wrote:


On 12/13/2017 2:45 AM, agrays...@gmail.com wrote:
BUT for a nucleus of a radioactive element, the nucleus is never  
Decayed and Undecayed SIMULTANEOUSLY.
Sure it is.  It's in a coherent superposition of those states  
until it interacts with the environment.


Brent

That's the conventional QM wisdom and the cause of the paradox of  
a cat Alive and Dead simultaneously. As I explained, the fallacy  
is rooted in an unjustified generalization of the double slit  
experiment where the probability waves do, in fact, exist  
simultaneously.  What waves do you claim are interacting for the  
radioactive nucleus to produce coherence? Tell me about them. I am  
from Missouri. AG


You seem to think that coherence requires two different waves.   
This is the wrong way to look at it.  In Young's slits experiment  
there is only one wave, which goes through both slits and  
interferes with itself.


That's exactly how I see it! Interference requires two waves which  
interact with each other.


NO.  This is false!  There are not two waves.  You can write it as  
two parts, just as you can write a description of an ocean wave as  
the part on your left and the part on your right.  But so long as  
they are coherent, maintaining a fixed phase relation, they are  
one wave.


You're splitting hairs, engaging in sophism. For the single wave  
going through both slits, Feynman calculates the norm squared of |  
A + B |, where A and B denote the waveS going through left and  
right slits respectively. Both are obviously identical, with the  
result of coherent interference. From this analysis we get the  
interpretation that the the system is simultaneously in all states  
of a superposition. AG


Noteworthy is that fact that if you reference "coherence" on Wiki,  
the description always invokes multiple waves of the same  
frequency. If you want to assert coherence without multiple waves,  
and NOT using the double slit result, you have some heavy lifting  
to do. AG



This is exactly what we see in Young's slits experiments. AG

And unstable nucleus has a probability amplitude that includes a  
"decayed" part and a "not decayed" part.  It's a tunneling problem.


I don't doubt the existence of amplitudes. What I do doubt. and  
in fact deny, is interference between two waves that don't exist  
simultaneously.


You keep referring to two waves.  There are not two waves.   
There's only one wave which interferes with itself.  It is  
typically written as |not-decayed> + |decayed>, but that's just a  
choice of basis.  It could as well be written |unstable nucleus>.


OK, unstable nucleus. Makes no difference to what I am arguing;  
namely, that coherence requires more than one wave,  
simultaneously, which is what double slit SHOWS, even though the  
experiment obviously starts out with one wave. AG


If there's no interference, then the cat cannot be Alive and Dead  
simultaneously. Tunneling can exist, but still, no simultaneous  
interacting, interfering waves. Is there any advantage to  
believing in two waves which don't exist simultaneous, can  
interfere with each other? AG


You are confused.

You're the one with a cat which Alive and Dead simultaneously for  
the very short time until decoherence occurs. So it is arguable  
who is really confused. AG

I agree with Brent. It is elementary quantum mechanics.

I seriously doubt you have a clue what we're discussing. If you  
take the time to read any definition of 'coherence', you will see  
it involves two or more interfering waves.
You can use two, or one wave. All you need is to gives them a  
similar phase, and it is easy to use only one wave, like the one  
going through two slit when sending, even one, photon.


You have an elaborate theory of the universe based on arithmetic but  
have difficulty counting to two. A single wave cannot exhibit  
interference. You need TWO waves! In double slit experiment, the  
slits split the original wave into TWO waves which interfere with  
each other, yielding the interference pattern. Got it? So the  
question is, how does one get interference in an experiment with a  
binary result? You can call it what you want, or calculate  
mindlessly, but the question persists; how is interference  
manifested in superpositions with binary results? AG





Re: Consistency of Postulates of QM

2017-12-17 Thread Brent Meeker



On 12/17/2017 8:06 AM, Bruno Marchal wrote:


On 15 Dec 2017, at 22:19, Brent Meeker wrote:




On 12/15/2017 9:24 AM, Bruno Marchal wrote:
that the statistics of the observable, in arithmetic from inside, 
have to "interfere" to make Digital Mechanism making sense in 
cognitive science, so MW-appearances is not bizarre at all: it has 
to be like that. Eventually, the "negative amplitude of probability" 
comes from the self-referential constraints (the logic of []p & <>p 
on p sigma_1, for those who have studied a little bit).


Can you explicate this.


Usually, notions like necessity, certainty, probability 1, etc. are 
assumed to obey []p -> p. This implies also []~p -> ~p, and thus p -> 
<>p, and so, if we have []p -> p, we have [] -> <>p (in classical 
normal modal logics).


Then provability, and even more "formal provability" was considered as 
as *the* closer notion to knowledge we could hope for, 


Something a mathematician or logician might dream, but not a mistake any 
physicist would ever make. Knowledge is correspondence with reality, not 
deducibility from axioms.


and so it came as a shock that no ("rich enough") theory can prove its 
own consistency. This means for example that neither ZF nor PA can 
prove ~[]f, that is []f -> f, 


This seems to me incorrectly rely on []f->f  being equivalent to 
~f->~[]f and ~f=t.  I know that is standard first order logic, but in 
this case we're talking about the whole infinite set of expressible 
propositions.  It's not so clear to me that you can rely on the law of 
the excluded middle over this set.


and so such machine cannot prove generally []p ->, and provability, 
for them, cannot works as a predicate for knowledge, and is at most a 
(hopefully correct) belief.


Now, this makes also possible to retrieve a classical notion of 
knowledge, by defining, for all arithmetical proposition p, the 
knowledge of p by []p & true(p).


I'm not impressed.

Unfortunately, we cannot define true(p) in arithmetic (Tarski), nor 
can we define knowledge at all (Thomason, Scott-Montague). But for 
eaxh arithmetical p, we can still mimic knowledge by []p & p, 


Since you can't define knowledge, how can you say you can mimic it?

for each p, and this lead to a way to associate canonically a knower 
to the machine-prover. It obeys to a knowledge logic (with []p -> p 
becoming trivial). That logic is captured soundly and completely by 
the logic S4Grz (already described in many posts).


Similarly, the logic G of arithmetical self-reference cannot be a 
logic of probability one, due to the fact that []p does not imply <>p 
(which would again contradict incompleteness). It entails in the 
Kripke semantics that each world can access to a cult-de-sac world in 
which []p is always true, despite there is no worlds accessible to 
verify such facts. 


But why should we accept that as a good model of inference?  It does not 
make intuitive sense to say []p is true in some world where p is neither 
true nor even possible.  What would be an example of such a world given 
a proposition like "7 is prime."?


We get a logic of probability by ensuring that "we are not in a 
cul-de-sac world", 


But isn't that equivalent to saying "anything is possible"?

which is the main default assumption need in probability calculus. In 
that case, you can justify, for example, that when you are duplicated 
in Washington and Moscow, the probability of getting a cup of coffee 
is one, when the protocol ensure the offering of coffee at both place: 
[]p in that case means "p is true in all accessible words, and there 
is at least one".


So, by incompleteness, [] & <>t provides a "probability one" notion, 
not reducible to simple provability ([]p).


Then, by step 8, we are in arithmetic (in the model of arithmetic, 
"model" in the logician's sense), and we translate computationalism by 
restricting the accessible "p" to the leaves of the universal 
dovetailing. By Gödel+Church-Turing-Kleene we can represent those 
"leaves" by the semi-computable predicates: the sigma_1 sentences. 
When we do this, we have to add the axiom "p->[]p" to G. This gives G1 
(and G1*). It is enough thanks to a proof by Visser. For the logic of 
the nuances brought by incompleteness, like []p & p, and []p & <>t, it 
gives the logic S4Grz1 and the logic Z1*. Then, we can extract an 
arithmetical interpretation of intuitionist logic from S4 (in a usual 
well known way), and, a bit less well known, we can extract a minimal 
quantum logic from B, and then from Z1* which is very close to B, 
using a "reverse" Goldblatt transform (as Goldblatt showed how the 
modal logic B (main axioms []p -> p, p -> []<>p, and NOT= []p -> 
[][]p) is a modal version of minimal quantum logic.


 I don't see that you have explicated negative amplitude of probability:

/*"Eventually, the "negative amplitude of probability" comes from the 
self-referential constraints (the logic of []p & <>p on p sigma_1, for 
those who have studied a 

Re: Consistency of Postulates of QM

2017-12-17 Thread Brent Meeker



On 12/17/2017 7:32 AM, Bruno Marchal wrote:

  But in fact the box is not isolated.


Oh? Just isolate the whole universe. That should be easy.




The box too is interacting with the environment.  So it's like the 
Zeno effect.  Although there is a probability at each impact of 
producing a coherent tails component, those components don't sum to a 
finite component over a finite number of impacts.


The Zeno effect makes you "staying statistically" in the universe, 
like the non-isolation of the box makes you impossible to have access 
to the universe where the coin felt on the opposite side, but without 
collapse, the superposition can simply never disappear.


I can never disappear, but it cannot reach a significant probability for 
tails in several ages of the universe.


Brent

--
You received this message because you are subscribed to the Google Groups 
"Everything List" group.
To unsubscribe from this group and stop receiving emails from it, send an email 
to everything-list+unsubscr...@googlegroups.com.
To post to this group, send email to everything-list@googlegroups.com.
Visit this group at https://groups.google.com/group/everything-list.
For more options, visit https://groups.google.com/d/optout.


Re: Consistency of Postulates of QM

2017-12-17 Thread agrayson2000
On Sunday, December 17, 2017 at 10:28:17 PM UTC, agrays...@gmail.com wrote:

>
> On Sunday, December 17, 2017 at 3:26:05 PM UTC, Bruno Marchal wrote:
>>
>>
>> On 15 Dec 2017, at 23:54, agrays...@gmail.com wrote:
>>
>> On Friday, December 15, 2017 at 5:24:39 PM UTC, Bruno Marchal wrote:
>>>
>>>
>>> On 14 Dec 2017, at 03:01, agrays...@gmail.com wrote:
>>>
>>> On Thursday, December 14, 2017 at 1:41:37 AM UTC, Brent wrote:
>>>

 On 12/13/2017 5:24 PM, agrays...@gmail.com wrote:

 On Wednesday, December 13, 2017 at 10:44:14 PM UTC, Brent wrote:

>
> On 12/13/2017 2:20 PM, agrays...@gmail.com wrote:
>
> On Wednesday, December 13, 2017 at 9:15:36 PM UTC, Brent wrote: 
>>
>>
>>
>> On 12/13/2017 2:45 AM, agrays...@gmail.com wrote:
>>
>> * BUT for a nucleus of a radioactive element, the nucleus is never 
>> Decayed and Undecayed SIMULTANEOUSLY.*
>>
>> Sure it is.  It's in a coherent superposition of those states until 
>> it interacts with the environment.
>>
>> Brent
>>
>
> * That's the conventional QM wisdom and the cause of the paradox of a 
> cat Alive and Dead simultaneously. As I explained, the fallacy is rooted 
> in 
> an unjustified generalization of the double slit experiment where the 
> probability waves do, in fact, exist simultaneously.  What waves do you 
> claim are interacting for the radioactive nucleus to produce coherence? 
> Tell me about them. I am from Missouri. AG*
>
>
> You seem to think that coherence requires two different waves.  This 
> is the wrong way to look at it.  In Young's slits experiment there is 
> only 
> one wave, which goes through both slits and interferes with itself.  
>


 *That's exactly how I see it! Interference requires two waves which 
 interact with each other. *


 *NO.  This is false! * *There are not two waves.*  You can write it as 
 two parts, just as you can write a description of an ocean wave as the 
 part 
 on your left and the part on your right.  But so long as they are 
 coherent, 
 maintaining a fixed phase relation, they are one wave.

>>>
>>>
>>>
>>> *You're splitting hairs, engaging in sophism. For the single wave going 
>>> through both slits, Feynman calculates the norm squared of | A + B |, where 
>>> A and B denote the waveS going through left and right slits respectively. 
>>> Both are obviously identical, with the result of coherent interference. 
>>> From this analysis we get the interpretation that the the system is 
>>> simultaneously in all states of a superposition. AGNoteworthy is that fact 
>>> that if you reference "coherence" on Wiki, the description always invokes 
>>> multiple waves of the same frequency. If you want to assert coherence 
>>> without multiple waves, and NOT using the double slit result, you have some 
>>> heavy lifting to do. AG*
>>>


 *This is exactly what we see in Young's slits experiments. AG  *

> And unstable nucleus has a probability amplitude that includes a 
> "decayed" part and a "not decayed" part.  It's a tunneling problem.
>

 *I don't doubt the existence of amplitudes. What I do doubt. and in 
 fact deny, is interference between two waves that don't exist 
 simultaneously. *


 You keep referring to two waves. * There are not two waves.  *There's 
 only one wave which interferes with itself.  It is typically written as 
 |not-decayed> + |decayed>, but that's just a choice of basis.  It could as 
 well be written |unstable nucleus>.

>>>
>>> *OK, unstable nucleus. Makes no difference to what I am arguing; namely, 
>>> that coherence requires more than one wave, simultaneously, which is what 
>>> double slit SHOWS, even though the experiment obviously starts out with one 
>>> wave. AG* 
>>>

 *If there's no interference, then the cat cannot be Alive and Dead 
 simultaneously. Tunneling can exist, but still, no simultaneous 
 interacting, interfering waves. Is there any advantage to believing in two 
 waves which don't exist simultaneous, can interfere with each other? AG*


 You are confused.

>>>
>>> You're the one with a cat which Alive and Dead simultaneously for the 
>>> very short time until decoherence occurs. So it is arguable who is really 
>>> confused. AG 
>>>
>>> I agree with Brent. It is elementary quantum mechanics.
>>>
>>> *I seriously doubt you have a clue what we're discussing. If you take 
>>> the time to read any definition of 'coherence', you will see it involves 
>>> two or more interfering waves.*
>>>
>> You can use two, or one wave. All you need is to gives them a similar 
>> phase, and it is easy to use only one wave, like the one going through two 
>> slit when sending, even one, photon.
>>  
>>
> You have an elaborate theory of the universe based on arithmetic but have 
> 

Re: Consistency of Postulates of QM

2017-12-17 Thread agrayson2000


On Sunday, December 17, 2017 at 3:26:05 PM UTC, Bruno Marchal wrote:
>
>
> On 15 Dec 2017, at 23:54, agrays...@gmail.com  wrote:
>
>
>
> On Friday, December 15, 2017 at 5:24:39 PM UTC, Bruno Marchal wrote:
>>
>>
>> On 14 Dec 2017, at 03:01, agrays...@gmail.com wrote:
>>
>>
>>
>> On Thursday, December 14, 2017 at 1:41:37 AM UTC, Brent wrote:
>>>
>>>
>>>
>>> On 12/13/2017 5:24 PM, agrays...@gmail.com wrote:
>>>
>>>
>>>
>>> On Wednesday, December 13, 2017 at 10:44:14 PM UTC, Brent wrote: 



 On 12/13/2017 2:20 PM, agrays...@gmail.com wrote:



 On Wednesday, December 13, 2017 at 9:15:36 PM UTC, Brent wrote: 
>
>
>
> On 12/13/2017 2:45 AM, agrays...@gmail.com wrote:
>
> * BUT for a nucleus of a radioactive element, the nucleus is never 
> Decayed and Undecayed SIMULTANEOUSLY.*
>
> Sure it is.  It's in a coherent superposition of those states until it 
> interacts with the environment.
>
> Brent
>

 * That's the conventional QM wisdom and the cause of the paradox of a 
 cat Alive and Dead simultaneously. As I explained, the fallacy is rooted 
 in 
 an unjustified generalization of the double slit experiment where the 
 probability waves do, in fact, exist simultaneously.  What waves do you 
 claim are interacting for the radioactive nucleus to produce coherence? 
 Tell me about them. I am from Missouri. AG*


 You seem to think that coherence requires two different waves.  This is 
 the wrong way to look at it.  In Young's slits experiment there is only 
 one 
 wave, which goes through both slits and interferes with itself.  

>>>
>>>
>>> *That's exactly how I see it! Interference requires two waves which 
>>> interact with each other. *
>>>
>>>
>>> *NO.  This is false! * *There are not two waves.*  You can write it as 
>>> two parts, just as you can write a description of an ocean wave as the part 
>>> on your left and the part on your right.  But so long as they are coherent, 
>>> maintaining a fixed phase relation, they are one wave.
>>>
>>
>>
>>
>> *You're splitting hairs, engaging in sophism. For the single wave going 
>> through both slits, Feynman calculates the norm squared of | A + B |, where 
>> A and B denote the waveS going through left and right slits respectively. 
>> Both are obviously identical, with the result of coherent interference. 
>> From this analysis we get the interpretation that the the system is 
>> simultaneously in all states of a superposition. AGNoteworthy is that fact 
>> that if you reference "coherence" on Wiki, the description always invokes 
>> multiple waves of the same frequency. If you want to assert coherence 
>> without multiple waves, and NOT using the double slit result, you have some 
>> heavy lifting to do. AG*
>>
>>>
>>>
>>> *This is exactly what we see in Young's slits experiments. AG  *
>>>
 And unstable nucleus has a probability amplitude that includes a 
 "decayed" part and a "not decayed" part.  It's a tunneling problem.

>>>
>>> *I don't doubt the existence of amplitudes. What I do doubt. and in fact 
>>> deny, is interference between two waves that don't exist simultaneously. *
>>>
>>>
>>> You keep referring to two waves. * There are not two waves.  *There's 
>>> only one wave which interferes with itself.  It is typically written as 
>>> |not-decayed> + |decayed>, but that's just a choice of basis.  It could as 
>>> well be written |unstable nucleus>.
>>>
>>
>> *OK, unstable nucleus. Makes no difference to what I am arguing; namely, 
>> that coherence requires more than one wave, simultaneously, which is what 
>> double slit slows, even though the experiment obviously starts out with one 
>> wave. AG* 
>>
>>>
>>> *If there's no interference, then the cat cannot be Alive and Dead 
>>> simultaneously. Tunneling can exist, but still, no simultaneous 
>>> interacting, interfering waves. Is there any advantage to believing in two 
>>> waves which don't exist simultaneous, can interfere with each other? AG*
>>>
>>>
>>> You are confused.
>>>
>>
>> You're the one with a cat which Alive and Dead simultaneously for the 
>> very short time until decoherence occurs. So it is arguable who is really 
>> confused. AG 
>>
>>
>>
>> I agree with Brent. It is elementary quantum mechanics. 
>>
>
>
> *I seriously doubt you have a clue what we're discussing. If you take the 
> time to read any definition of 'coherence', you will see it involves two or 
> more interfering waves.*
>
>
>
>
>
>
> You can use two, or one wave. All you need is to gives them a similar 
> phase, and it is easy to use only one wave, like the one going through two 
> slit when sending, even one, photon.
>

 
You have an elaborate theory of the universe based on arithmetic but have 
difficulty counting to two. A single wave cannot exhibit interference. You 
need TWO waves! In double slit experiment, the slits split the original 
wave into TWO waves 

Re: Consistency of Postulates of QM

2017-12-17 Thread Bruno Marchal


On 15 Dec 2017, at 22:19, Brent Meeker wrote:




On 12/15/2017 9:24 AM, Bruno Marchal wrote:
that the statistics of the observable, in arithmetic from inside,  
have to "interfere" to make Digital Mechanism making sense in  
cognitive science, so MW-appearances is not bizarre at all: it has  
to be like that. Eventually, the "negative amplitude of  
probability" comes from the self-referential constraints (the logic  
of []p & <>p on p sigma_1, for those who have studied a little bit).


Can you explicate this.


Usually, notions like necessity, certainty, probability 1, etc. are  
assumed to obey []p -> p. This implies also []~p -> ~p, and thus p ->  
<>p, and so, if we have []p -> p, we have [] -> <>p (in classical  
normal modal logics).


Then provability, and even more "formal provability" was considered as  
as *the* closer notion to knowledge we could hope for, and so it came  
as a shock that no ("rich enough") theory can prove its own  
consistency. This means for example that neither ZF nor PA can prove  
~[]f, that is []f -> f, and so such machine cannot prove generally []p  
->, and provability, for them, cannot works as a predicate for  
knowledge, and is at most a (hopefully correct) belief.


Now, this makes also possible to retrieve a classical notion of  
knowledge, by defining, for all arithmetical proposition p, the  
knowledge of p by []p & true(p).
Unfortunately, we cannot define true(p) in arithmetic (Tarski), nor  
can we define knowledge at all (Thomason, Scott-Montague). But for  
eaxh arithmetical p, we can still mimic knowledge by []p & p, for each  
p, and this lead to a way to associate canonically a knower to the  
machine-prover. It obeys to a knowledge logic (with []p -> p becoming  
trivial). That logic is captured soundly and completely by the logic  
S4Grz (already described in many posts).


Similarly, the logic G of arithmetical self-reference cannot be a  
logic of probability one, due to the fact that []p does not imply <>p  
(which would again contradict incompleteness). It entails in the  
Kripke semantics that each world can access to a cult-de-sac world in  
which []p is always true, despite there is no worlds accessible to  
verify such facts. We get a logic of probability by ensuring that "we  
are not in a cul-de-sac world", which is the main default assumption  
need in probability calculus. In that case, you can justify, for  
example, that when you are duplicated in Washington and Moscow, the  
probability of getting a cup of coffee is one, when the protocol  
ensure the offering of coffee at both place: []p in that case means "p  
is true in all accessible words, and there is at least one".


So, by incompleteness, [] & <>t provides a "probability one" notion,  
not reducible to simple provability ([]p).


Then, by step 8, we are in arithmetic (in the model of arithmetic,  
"model" in the logician's sense), and we translate computationalism by  
restricting the accessible "p" to the leaves of the universal  
dovetailing. By Gödel+Church-Turing-Kleene we can represent those  
"leaves" by the semi-computable predicates: the sigma_1 sentences.  
When we do this, we have to add the axiom "p->[]p" to G. This gives G1  
(and G1*). It is enough thanks to a proof by Visser. For the logic of  
the nuances brought by incompleteness, like []p & p, and []p & <>t, it  
gives the logic S4Grz1 and the logic Z1*. Then, we can extract an  
arithmetical interpretation of intuitionist logic from S4 (in a usual  
well known way), and, a bit less well known, we can extract a minimal  
quantum logic from B, and then from Z1* which is very close to B,  
using a "reverse" Goldblatt transform (as Goldblatt showed how the  
modal logic B (main axioms []p -> p, p -> []<>p, and NOT= []p -> [] 
[]p) is a modal version of minimal quantum logic.


Note that here "[] and "<>" are arithmetical predicate. We do not  
assume more than Q, and use only internal interpretabilities of the  
observer-machines.
This is explained in most of my papers, but the details are in the  
long french text "Conscience et Mécanisme".


Bruno











Brent

--
You received this message because you are subscribed to the Google  
Groups "Everything List" group.
To unsubscribe from this group and stop receiving emails from it,  
send an email to everything-list+unsubscr...@googlegroups.com.

To post to this group, send email to everything-list@googlegroups.com.
Visit this group at https://groups.google.com/group/everything-list.
For more options, visit https://groups.google.com/d/optout.


http://iridia.ulb.ac.be/~marchal/



--
You received this message because you are subscribed to the Google Groups 
"Everything List" group.
To unsubscribe from this group and stop receiving emails from it, send an email 
to everything-list+unsubscr...@googlegroups.com.
To post to this group, send email to everything-list@googlegroups.com.
Visit this group at https://groups.google.com/group/everything-list.
For more 

Re: Consistency of Postulates of QM

2017-12-17 Thread Bruno Marchal


On 15 Dec 2017, at 22:18, Brent Meeker wrote:




On 12/15/2017 9:06 AM, Bruno Marchal wrote:


On 13 Dec 2017, at 22:23, Brent Meeker wrote:




On 12/13/2017 3:15 AM, Bruno Marchal wrote:
The coin does not start in a state of the kind  {|heads> + | 
tails>}, but it starts with a state of having mulitiple positions  
and multiple momenta, spreaded in the multiverse according to the  
Heisenberg Uncertainty. The tiny difference in the position can  
lead to different bouncing in the box, and so, by shaking it a  
long time enough, I don’t see why we could avoid a superposition  
of head and tails eventually.


Decoherence is waaay faster than the time for the coin to cross  
the box from one impact to another, so I don't see how a  
superposition could develop from shaking the box.



But you just said:

<<

On 13 Dec 2017, at 22:15, Brent Meeker wrote:




On 12/13/2017 2:45 AM, agrayson2...@gmail.com wrote:
BUT for a nucleus of a radioactive element, the nucleus is never  
Decayed and Undecayed SIMULTANEOUSLY.


Sure it is.  It's in a coherent superposition of those states  
until it interacts with the environment.


>>

That is true FAPP, but in the MW, there is no collapse of the wave,  
and the coherent superposition does not disappear, it just spead on  
the environment itself, making the people belonging to those  
environment perceiving something like a collapse,  or like a divine  
choice (where of course a mechanist knows that it could be nothing  
more than a first person comp indeterminacy).


In the 3p view of the universal wave (or just the global one  
involving the coin and the shaker of the coin), the superposition  
is spreaded,


What superposition?  In the shaken coin in a box, the coin starts in  
a known state, e.g. heads,


Not really. The coin starts with the position in the superposition  
state here-1 + here-2 + here-3 allowed by our unsharp knowledge of its  
momentum (the coin is supposed to be almost still at the start).





and the question is whether the shaking puts it into a superposition  
|heads>+|tails> or whether the evolution of its state is classical.   
I think it is a question of time and amplification of quantum  
effects at each impact with the walls of the box.


Yes.


If the box+coin is isolated there is some time that would be  
sufficient to produce a superposition.


Nice, so we do agree on this. That is what I was trying to explain (to  
Bruce and Grayson).




  But in fact the box is not isolated.


Oh? Just isolate the whole universe. That should be easy.




The box too is interacting with the environment.  So it's like the  
Zeno effect.  Although there is a probability at each impact of  
producing a coherent tails component, those components don't sum to  
a finite component over a finite number of impacts.


The Zeno effect makes you "staying statistically" in the universe,  
like the non-isolation of the box makes you impossible to have access  
to the universe where the coin felt on the opposite side, but without  
collapse, the superposition can simply never disappear.


Bruno





Brent

indeed very quickly to all the material objects interacting with  
the coin, already inside the box. The decoherence time is only the  
time the universe differentiatesn and indeed, it does that very  
quickly, but this does not change the fact that the differentiation  
is not enough big to lead to full quasi orthogonal tail + head  
state after shaking the device (box + coin) long enough.


Bruno








Brent

--
You received this message because you are subscribed to the Google  
Groups "Everything List" group.
To unsubscribe from this group and stop receiving emails from it,  
send an email to everything-list+unsubscr...@googlegroups.com.
To post to this group, send email to everything-list@googlegroups.com 
.

Visit this group at https://groups.google.com/group/everything-list.
For more options, visit https://groups.google.com/d/optout.


http://iridia.ulb.ac.be/~marchal/



--
You received this message because you are subscribed to the Google  
Groups "Everything List" group.
To unsubscribe from this group and stop receiving emails from it,  
send an email to everything-list+unsubscr...@googlegroups.com.
To post to this group, send email to everything- 
l...@googlegroups.com.

Visit this group at https://groups.google.com/group/everything-list.
For more options, visit https://groups.google.com/d/optout.



--
You received this message because you are subscribed to the Google  
Groups "Everything List" group.
To unsubscribe from this group and stop receiving emails from it,  
send an email to everything-list+unsubscr...@googlegroups.com.

To post to this group, send email to everything-list@googlegroups.com.
Visit this group at https://groups.google.com/group/everything-list.
For more options, visit https://groups.google.com/d/optout.


http://iridia.ulb.ac.be/~marchal/



--
You received this message because you are subscribed to the Google Groups 

Re: Consistency of Postulates of QM

2017-12-17 Thread Bruno Marchal


On 15 Dec 2017, at 23:54, agrayson2...@gmail.com wrote:




On Friday, December 15, 2017 at 5:24:39 PM UTC, Bruno Marchal wrote:

On 14 Dec 2017, at 03:01, agrays...@gmail.com wrote:




On Thursday, December 14, 2017 at 1:41:37 AM UTC, Brent wrote:


On 12/13/2017 5:24 PM, agrays...@gmail.com wrote:



On Wednesday, December 13, 2017 at 10:44:14 PM UTC, Brent wrote:


On 12/13/2017 2:20 PM, agrays...@gmail.com wrote:



On Wednesday, December 13, 2017 at 9:15:36 PM UTC, Brent wrote:


On 12/13/2017 2:45 AM, agrays...@gmail.com wrote:
BUT for a nucleus of a radioactive element, the nucleus is never  
Decayed and Undecayed SIMULTANEOUSLY.
Sure it is.  It's in a coherent superposition of those states  
until it interacts with the environment.


Brent

That's the conventional QM wisdom and the cause of the paradox of  
a cat Alive and Dead simultaneously. As I explained, the fallacy  
is rooted in an unjustified generalization of the double slit  
experiment where the probability waves do, in fact, exist  
simultaneously.  What waves do you claim are interacting for the  
radioactive nucleus to produce coherence? Tell me about them. I  
am from Missouri. AG


You seem to think that coherence requires two different waves.   
This is the wrong way to look at it.  In Young's slits experiment  
there is only one wave, which goes through both slits and  
interferes with itself.


That's exactly how I see it! Interference requires two waves which  
interact with each other.


NO.  This is false!  There are not two waves.  You can write it as  
two parts, just as you can write a description of an ocean wave as  
the part on your left and the part on your right.  But so long as  
they are coherent, maintaining a fixed phase relation, they are one  
wave.


You're splitting hairs, engaging in sophism. For the single wave  
going through both slits, Feynman calculates the norm squared of |  
A + B |, where A and B denote the waveS going through left and  
right slits respectively. Both are obviously identical, with the  
result of coherent interference. From this analysis we get the  
interpretation that the the system is simultaneously in all states  
of a superposition. AG


Noteworthy is that fact that if you reference "coherence" on Wiki,  
the description always invokes multiple waves of the same  
frequency. If you want to assert coherence without multiple waves,  
and NOT using the double slit result, you have some heavy lifting  
to do. AG



This is exactly what we see in Young's slits experiments. AG

And unstable nucleus has a probability amplitude that includes a  
"decayed" part and a "not decayed" part.  It's a tunneling problem.


I don't doubt the existence of amplitudes. What I do doubt. and in  
fact deny, is interference between two waves that don't exist  
simultaneously.


You keep referring to two waves.  There are not two waves.  There's  
only one wave which interferes with itself.  It is typically  
written as |not-decayed> + |decayed>, but that's just a choice of  
basis.  It could as well be written |unstable nucleus>.


OK, unstable nucleus. Makes no difference to what I am arguing;  
namely, that coherence requires more than one wave, simultaneously,  
which is what double slit slows, even though the experiment  
obviously starts out with one wave. AG


If there's no interference, then the cat cannot be Alive and Dead  
simultaneously. Tunneling can exist, but still, no simultaneous  
interacting, interfering waves. Is there any advantage to  
believing in two waves which don't exist simultaneous, can  
interfere with each other? AG


You are confused.

You're the one with a cat which Alive and Dead simultaneously for  
the very short time until decoherence occurs. So it is arguable who  
is really confused. AG



I agree with Brent. It is elementary quantum mechanics.


I seriously doubt you have a clue what we're discussing. If you take  
the time to read any definition of 'coherence', you will see it  
involves two or more interfering waves.






You can use two, or one wave. All you need is to gives them a similar  
phase, and it is easy to use only one wave, like the one going through  
two slit when sending, even one, photon.





Don't take my word. Check for yourself.


I did.



Moreover, I don't doubt that nuclear decay is a tunneling problem,  
with probability amplitudes that agree with experiments and allow  
nuclear weapons to function as advertised -- as you write,  
"elementary quantum mechanics".


That is why I prefer to recast the "schroedinger cat" with an  
amplifucation of the spin up+down state, instead of nuclear decay.  
That is what Bohm did. The up+down state is invariant with time, and  
made the thought experiment easir. No need to invoke quantum  
tunneling. The point is that the superposition never disappear if  
there is no collapse, and that the observer get themselves into a  
superposition. They cannot notice it by "Elementary Mechanist Theory  
of 

Re: Consistency of Postulates of QM

2017-12-15 Thread agrayson2000


On Friday, December 15, 2017 at 5:24:39 PM UTC, Bruno Marchal wrote:
>
>
> On 14 Dec 2017, at 03:01, agrays...@gmail.com  wrote:
>
>
>
> On Thursday, December 14, 2017 at 1:41:37 AM UTC, Brent wrote:
>>
>>
>>
>> On 12/13/2017 5:24 PM, agrays...@gmail.com wrote:
>>
>>
>>
>> On Wednesday, December 13, 2017 at 10:44:14 PM UTC, Brent wrote: 
>>>
>>>
>>>
>>> On 12/13/2017 2:20 PM, agrays...@gmail.com wrote:
>>>
>>>
>>>
>>> On Wednesday, December 13, 2017 at 9:15:36 PM UTC, Brent wrote: 



 On 12/13/2017 2:45 AM, agrays...@gmail.com wrote:

 * BUT for a nucleus of a radioactive element, the nucleus is never 
 Decayed and Undecayed SIMULTANEOUSLY.*

 Sure it is.  It's in a coherent superposition of those states until it 
 interacts with the environment.

 Brent

>>>
>>> * That's the conventional QM wisdom and the cause of the paradox of a 
>>> cat Alive and Dead simultaneously. As I explained, the fallacy is rooted in 
>>> an unjustified generalization of the double slit experiment where the 
>>> probability waves do, in fact, exist simultaneously.  What waves do you 
>>> claim are interacting for the radioactive nucleus to produce coherence? 
>>> Tell me about them. I am from Missouri. AG*
>>>
>>>
>>> You seem to think that coherence requires two different waves.  This is 
>>> the wrong way to look at it.  In Young's slits experiment there is only one 
>>> wave, which goes through both slits and interferes with itself.  
>>>
>>
>>
>> *That's exactly how I see it! Interference requires two waves which 
>> interact with each other. *
>>
>>
>> *NO.  This is false! * *There are not two waves.*  You can write it as 
>> two parts, just as you can write a description of an ocean wave as the part 
>> on your left and the part on your right.  But so long as they are coherent, 
>> maintaining a fixed phase relation, they are one wave.
>>
>
>
>
> *You're splitting hairs, engaging in sophism. For the single wave going 
> through both slits, Feynman calculates the norm squared of | A + B |, where 
> A and B denote the waveS going through left and right slits respectively. 
> Both are obviously identical, with the result of coherent interference. 
> From this analysis we get the interpretation that the the system is 
> simultaneously in all states of a superposition. AGNoteworthy is that fact 
> that if you reference "coherence" on Wiki, the description always invokes 
> multiple waves of the same frequency. If you want to assert coherence 
> without multiple waves, and NOT using the double slit result, you have some 
> heavy lifting to do. AG*
>
>>
>>
>> *This is exactly what we see in Young's slits experiments. AG  *
>>
>>> And unstable nucleus has a probability amplitude that includes a 
>>> "decayed" part and a "not decayed" part.  It's a tunneling problem.
>>>
>>
>> *I don't doubt the existence of amplitudes. What I do doubt. and in fact 
>> deny, is interference between two waves that don't exist simultaneously. *
>>
>>
>> You keep referring to two waves. * There are not two waves.  *There's 
>> only one wave which interferes with itself.  It is typically written as 
>> |not-decayed> + |decayed>, but that's just a choice of basis.  It could as 
>> well be written |unstable nucleus>.
>>
>
> *OK, unstable nucleus. Makes no difference to what I am arguing; namely, 
> that coherence requires more than one wave, simultaneously, which is what 
> double slit slows, even though the experiment obviously starts out with one 
> wave. AG* 
>
>>
>> *If there's no interference, then the cat cannot be Alive and Dead 
>> simultaneously. Tunneling can exist, but still, no simultaneous 
>> interacting, interfering waves. Is there any advantage to believing in two 
>> waves which don't exist simultaneous, can interfere with each other? AG*
>>
>>
>> You are confused.
>>
>
> You're the one with a cat which Alive and Dead simultaneously for the very 
> short time until decoherence occurs. So it is arguable who is really 
> confused. AG 
>
>
>
> I agree with Brent. It is elementary quantum mechanics. 
>




*I seriously doubt you have a clue what we're discussing. If you take the 
time to read any definition of 'coherence', you will see it involves two or 
more interfering waves. Don't take my word. Check for yourself. Moreover, I 
don't doubt that nuclear decay is a tunneling problem, with probability 
amplitudes that agree with experiments and allow nuclear weapons to 
function as advertised -- as you write, "elementary quantum mechanics". But 
in order for the decay process, when tied to a cat, yields a cat which is 
simultaneously Alive and Dead seems to depend on giving the wf ontological 
status. I suppose I have to change my previously stated view, and assert 
that the wf has only epistemological content. This also disposes of your 
infinity of cats; a conjecture, btw, that you never "knew", but rather 
believed, and is now based on elaborate, contrived, arithmetic mumbo-jumbo. 

Re: Consistency of Postulates of QM

2017-12-15 Thread Brent Meeker



On 12/15/2017 9:24 AM, Bruno Marchal wrote:
that the statistics of the observable, in arithmetic from inside, have 
to "interfere" to make Digital Mechanism making sense in cognitive 
science, so MW-appearances is not bizarre at all: it has to be like 
that. Eventually, the "negative amplitude of probability" comes from 
the self-referential constraints (the logic of []p & <>p on p sigma_1, 
for those who have studied a little bit).


Can you explicate this.

Brent

--
You received this message because you are subscribed to the Google Groups 
"Everything List" group.
To unsubscribe from this group and stop receiving emails from it, send an email 
to everything-list+unsubscr...@googlegroups.com.
To post to this group, send email to everything-list@googlegroups.com.
Visit this group at https://groups.google.com/group/everything-list.
For more options, visit https://groups.google.com/d/optout.


Re: Consistency of Postulates of QM

2017-12-15 Thread Brent Meeker



On 12/15/2017 9:06 AM, Bruno Marchal wrote:


On 13 Dec 2017, at 22:23, Brent Meeker wrote:




On 12/13/2017 3:15 AM, Bruno Marchal wrote:
The coin does not start in a state of the kind  {|heads> + |tails>}, 
but it starts with a state of having mulitiple positions and 
multiple momenta, spreaded in the multiverse according to the 
Heisenberg Uncertainty. The tiny difference in the position can lead 
to different bouncing in the box, and so, by shaking it a long time 
enough, I don’t see why we could avoid a superposition of head and 
tails eventually.


Decoherence is waaay faster than the time for the coin to cross the 
box from one impact to another, so I don't see how a superposition 
could develop from shaking the box.



But you just said:

<<

On 13 Dec 2017, at 22:15, Brent Meeker wrote:




On 12/13/2017 2:45 AM, agrayson2...@gmail.com wrote:
*BUT for a nucleus of a radioactive element, the nucleus is never 
Decayed and Undecayed SIMULTANEOUSLY.*


Sure it is.  It's in a coherent superposition of those states until 
it interacts with the environment.


>>

That is true FAPP, but in the MW, there is no collapse of the wave, 
and the coherent superposition does not disappear, it just spead on 
the environment itself, making the people belonging to those 
environment perceiving something like a collapse,  or like a divine 
choice (where of course a mechanist knows that it could be nothing 
more than a first person comp indeterminacy).


In the 3p view of the universal wave (or just the global one involving 
the coin and the shaker of the coin), the superposition is spreaded,


What superposition?  In the shaken coin in a box, the coin starts in a 
known state, e.g. heads, and the question is whether the shaking puts it 
into a superposition |heads>+|tails> or whether the evolution of its 
state is classical.  I think it is a question of time and amplification 
of quantum effects at each impact with the walls of the box.  If the 
box+coin is isolated there is some time that would be sufficient to 
produce a superposition.   But in fact the box is not isolated.  The box 
too is interacting with the environment.  So it's like the Zeno effect.  
Although there is a probability at each impact of producing a coherent 
tails component, those components don't sum to a finite component over a 
finite number of impacts.


Brent

indeed very quickly to all the material objects interacting with the 
coin, already inside the box. The decoherence time is only the time 
the universe differentiatesn and indeed, it does that very quickly, 
but this does not change the fact that the differentiation is not 
enough big to lead to full quasi orthogonal tail + head state after 
shaking the device (box + coin) long enough.


Bruno








Brent

--
You received this message because you are subscribed to the Google 
Groups "Everything List" group.
To unsubscribe from this group and stop receiving emails from it, 
send an email to everything-list+unsubscr...@googlegroups.com 
.
To post to this group, send email to everything-list@googlegroups.com 
.

Visit this group at https://groups.google.com/group/everything-list.
For more options, visit https://groups.google.com/d/optout.


http://iridia.ulb.ac.be/~marchal/ 



--
You received this message because you are subscribed to the Google 
Groups "Everything List" group.
To unsubscribe from this group and stop receiving emails from it, send 
an email to everything-list+unsubscr...@googlegroups.com 
.
To post to this group, send email to everything-list@googlegroups.com 
.

Visit this group at https://groups.google.com/group/everything-list.
For more options, visit https://groups.google.com/d/optout.


--
You received this message because you are subscribed to the Google Groups 
"Everything List" group.
To unsubscribe from this group and stop receiving emails from it, send an email 
to everything-list+unsubscr...@googlegroups.com.
To post to this group, send email to everything-list@googlegroups.com.
Visit this group at https://groups.google.com/group/everything-list.
For more options, visit https://groups.google.com/d/optout.


Re: Consistency of Postulates of QM

2017-12-15 Thread John Clark
On Thu, Dec 14, 2017 at 4:35 PM, Bruce Kellett 
wrote:

>
> ​>> ​
>> David Deutsch proposed a test of Many Worlds about 30 years ago in his
>> book "The Ghost In The Atom", but
>> ​ it ​
>> would be very difficult to perform. The reason it's so difficult to test
>> is not
>> ​ the ​M
>> any
>> ​ World's
>> ​ ​
>> theory fault, the reason is that the conventional view says that
>> conscious
>> ​ ​
>> observers obey different laws of physics, many worlds says they do not,
>> so
>> ​ ​
>> to test who's right we need a mind that uses quantum properties.
>> ​ ​
>> Quantum Computers have advanced enormously over the last 30 years
>> ​ so​
>>  I wouldn't be surprised if it or something very much like it
>> ​ is ​
>> actually performed in the decade or two.
>>
> ​
>>
>> Now the mind uses
>> ​ ​
>> quantum erasure to completely destroy
>> ​ its​
>>  memory of
>> ​ which slit any of the ​photons went through; t
>> he only
>> ​ ​
>> part remaining is the document
>> ​ which states that each photon went through one and only one slit and
>> the mind (at the time) knew which one. ​
>> Now develop the photographic plate and look at it.
>> ​ I​
>> f you see interference bands then the many world interpretation is
>> ​ ​
>> correct. If you do not see interference bands then there are no worlds but
>> ​ ​
>> this one and the conventional
>> ​ quantum ​
>> interpretation is correct.
>>
> ​This works because ​in the Copenhagen interpretation when the results of
>> a measurement enters the consciousness of an observer the wave function
>> collapses, in effect all the universes except one disappear without a
>> trace so you get no interference. In the many worlds model all the other 
>> worlds
>> will converge back into one universe because information on which slit
>> the various photons went through was the only thing that made one universe
>> different from another, so when that was erased they became identical again
>> and merged, but their influence will still be felt, you'll see indications
>> that the photon went through slot A only and indications it went through
>> slot B only, and that's what causes interference. An intelligent quantum
>> computer shoots
>> ​ ​
>> photons at a metal plate
>> ​ ​
>> one at a time
>> ​ ​
>> that has 2 small slits in
>> ​ ​it,
>> and then the photons hit a photographic plate.
>> ​ ​
>> Nobody looks at the photographic plate till the very end of the
>> experiment.
>> ​
>> The quantum mind has detectors near each slit so it knows which slit
>> ​ ​
>> the various electrons went through.
>> ​ ​
>> After each photon passes the slits but before they hit the photographic
>> plate the quantum mind signs a document
>> ​ ​
>> saying that it has observed each and every
>> ​ photon​
>>  and knows
>> ​ which​
>>  slit each
>> ​ ​
>> photon
>> ​ ​
>> went through. It is very important that the document does not say
>> ​ ​
>> which slit
>> ​ ​
>> any
>> ​ ​photon
>>  went through, it only says that they went through
>> ​ ​
>> one slit
>> ​ ​
>> and one slit only and the mind has knowledge of which one.
>> ​ ​
>> There is a signed document to this effect for every photon it shot.
>> ​  ​
>> Now the mind uses
>> ​ ​
>> quantum erasure to completely destroy
>> ​its​
>>  memory of
>> ​which slit any of the ​photons went through; t
>> he only
>> ​ ​
>> part remaining is the document
>> ​ which states that each photon went through one and only one slit and
>> the mind (at the time) knew which one. ​
>> Now develop the photographic plate and look at it.
>> ​I​
>> f you see interference bands then the many world interpretation is
>> ​ ​
>> correct. If you do not see interference bands then there are no worlds but
>> ​ ​
>> this one and the conventional
>> ​quantum ​
>> interpretation is correct.
>>
>> ​This works because ​in the Copenhagen interpretation when the results
>> of a measurement enters the consciousness of an observer the wave function
>> collapses, in effect all the universes except one disappear without a trace
>> so you get no interference. In the many worlds model all the other worlds
>> will converge back into one universe because information on which slit
>> the various photons went through was the only thing that made one universe
>> different from another, so when that was erased they became identical again
>> and merged, but their influence will still be felt, you'll see indications
>> that the photon went through slot A only and indications it went through
>> slot B only, and that's what causes interference.
>>
>
> ​>* ​*
> *I think this argument pre-dates the work by Zeh and Zurek developing the
> idea of decoherence. Decoherence remove the oddities of Copenhagen as
> presented above in that it is not consciousness that does the work, but
> decoherence. Bohr was saying essentially the same thing (though he didn't
> know the words) when he talked about the importance of the whole
> experimental set up.That aside, Deutsch's idea fails because he has not
> fully 

Re: Consistency of Postulates of QM

2017-12-15 Thread Bruno Marchal


On 14 Dec 2017, at 03:01, agrayson2...@gmail.com wrote:




On Thursday, December 14, 2017 at 1:41:37 AM UTC, Brent wrote:


On 12/13/2017 5:24 PM, agrays...@gmail.com wrote:



On Wednesday, December 13, 2017 at 10:44:14 PM UTC, Brent wrote:


On 12/13/2017 2:20 PM, agrays...@gmail.com wrote:



On Wednesday, December 13, 2017 at 9:15:36 PM UTC, Brent wrote:


On 12/13/2017 2:45 AM, agrays...@gmail.com wrote:
BUT for a nucleus of a radioactive element, the nucleus is never  
Decayed and Undecayed SIMULTANEOUSLY.
Sure it is.  It's in a coherent superposition of those states  
until it interacts with the environment.


Brent

That's the conventional QM wisdom and the cause of the paradox of  
a cat Alive and Dead simultaneously. As I explained, the fallacy  
is rooted in an unjustified generalization of the double slit  
experiment where the probability waves do, in fact, exist  
simultaneously.  What waves do you claim are interacting for the  
radioactive nucleus to produce coherence? Tell me about them. I am  
from Missouri. AG


You seem to think that coherence requires two different waves.   
This is the wrong way to look at it.  In Young's slits experiment  
there is only one wave, which goes through both slits and  
interferes with itself.


That's exactly how I see it! Interference requires two waves which  
interact with each other.


NO.  This is false!  There are not two waves.  You can write it as  
two parts, just as you can write a description of an ocean wave as  
the part on your left and the part on your right.  But so long as  
they are coherent, maintaining a fixed phase relation, they are one  
wave.


You're splitting hairs, engaging in sophism. For the single wave  
going through both slits, Feynman calculates the norm squared of | A  
+ B |, where A and B denote the waveS going through left and right  
slits respectively. Both are obviously identical, with the result of  
coherent interference. From this analysis we get the interpretation  
that the the system is simultaneously in all states of a  
superposition. AG


Noteworthy is that fact that if you reference "coherence" on Wiki,  
the description always invokes multiple waves of the same frequency.  
If you want to assert coherence without multiple waves, and NOT  
using the double slit result, you have some heavy lifting to do. AG



This is exactly what we see in Young's slits experiments. AG

And unstable nucleus has a probability amplitude that includes a  
"decayed" part and a "not decayed" part.  It's a tunneling problem.


I don't doubt the existence of amplitudes. What I do doubt. and in  
fact deny, is interference between two waves that don't exist  
simultaneously.


You keep referring to two waves.  There are not two waves.  There's  
only one wave which interferes with itself.  It is typically written  
as |not-decayed> + |decayed>, but that's just a choice of basis.  It  
could as well be written |unstable nucleus>.


OK, unstable nucleus. Makes no difference to what I am arguing;  
namely, that coherence requires more than one wave, simultaneously,  
which is what double slit slows, even though the experiment  
obviously starts out with one wave. AG


If there's no interference, then the cat cannot be Alive and Dead  
simultaneously. Tunneling can exist, but still, no simultaneous  
interacting, interfering waves. Is there any advantage to believing  
in two waves which don't exist simultaneous, can interfere with  
each other? AG


You are confused.

You're the one with a cat which Alive and Dead simultaneously for  
the very short time until decoherence occurs. So it is arguable who  
is really confused. AG



I agree with Brent. It is elementary quantum mechanics. Then the  
decoherence is only in the mind of those looking at the cat, as  
explained entirely by the formalism itself (although there is a  
technical debate on the question of the derivation of the Born Rule,  
but for me that problem has been solved a very long time ago by  
Paulette Février-Destouches, and simple (non rigorous) proof exists in  
Preskill's course on quantum computation, or in Selestnick's book  
"Quanta, Logic and Space-Time"), and personally, I think this is  
solved completely by Gleason theorem.


Then, I already knew, before knewing QM, that infinitely many cats  
live in arithmetic, and that the statistics of the observable, in  
arithmetic from inside, have to "interfere" to make Digital Mechanism  
making sense in cognitive science, so MW-appearances is not bizarre at  
all: it has to be like that. Eventually, the "negative amplitude of  
probability" comes from the self-referential constraints (the logic of  
[]p & <>p on p sigma_1, for those who have studied a little bit).


Bruno







Brent

--
You received this message because you are subscribed to the Google  
Groups "Everything List" group.
To unsubscribe from this group and stop receiving emails from it,  
send an email to 

Re: Consistency of Postulates of QM

2017-12-15 Thread Bruno Marchal


On 13 Dec 2017, at 22:23, Brent Meeker wrote:




On 12/13/2017 3:15 AM, Bruno Marchal wrote:
The coin does not start in a state of the kind  {|heads> + | 
tails>}, but it starts with a state of having mulitiple positions  
and multiple momenta, spreaded in the multiverse according to the  
Heisenberg Uncertainty. The tiny difference in the position can  
lead to different bouncing in the box, and so, by shaking it a long  
time enough, I don’t see why we could avoid a superposition of head  
and tails eventually.


Decoherence is waaay faster than the time for the coin to cross the  
box from one impact to another, so I don't see how a superposition  
could develop from shaking the box.



But you just said:

<<

On 13 Dec 2017, at 22:15, Brent Meeker wrote:




On 12/13/2017 2:45 AM, agrayson2...@gmail.com wrote:
BUT for a nucleus of a radioactive element, the nucleus is never  
Decayed and Undecayed SIMULTANEOUSLY.


Sure it is.  It's in a coherent superposition of those states until  
it interacts with the environment.


>>

That is true FAPP, but in the MW, there is no collapse of the wave,  
and the coherent superposition does not disappear, it just spead on  
the environment itself, making the people belonging to those  
environment perceiving something like a collapse,  or like a divine  
choice (where of course a mechanist knows that it could be nothing  
more than a first person comp indeterminacy).


In the 3p view of the universal wave (or just the global one involving  
the coin and the shaker of the coin), the superposition is spreaded,  
indeed very quickly to all the material objects interacting with the  
coin, already inside the box. The decoherence time is only the time  
the universe differentiatesn and indeed, it does that very quickly,  
but this does not change the fact that the differentiation is not  
enough big to lead to full quasi orthogonal tail + head state after  
shaking the device (box + coin) long enough.


Bruno








Brent

--
You received this message because you are subscribed to the Google  
Groups "Everything List" group.
To unsubscribe from this group and stop receiving emails from it,  
send an email to everything-list+unsubscr...@googlegroups.com.

To post to this group, send email to everything-list@googlegroups.com.
Visit this group at https://groups.google.com/group/everything-list.
For more options, visit https://groups.google.com/d/optout.


http://iridia.ulb.ac.be/~marchal/



--
You received this message because you are subscribed to the Google Groups 
"Everything List" group.
To unsubscribe from this group and stop receiving emails from it, send an email 
to everything-list+unsubscr...@googlegroups.com.
To post to this group, send email to everything-list@googlegroups.com.
Visit this group at https://groups.google.com/group/everything-list.
For more options, visit https://groups.google.com/d/optout.


Re: Consistency of Postulates of QM

2017-12-15 Thread smitra

On 15-12-2017 00:36, Bruce Kellett wrote:

On 15/12/2017 10:15 am, smitra wrote:

On 14-12-2017 22:35, Bruce Kellett wrote:


 I think this argument pre-dates the work by Zeh and Zurek developing
the idea of decoherence. Decoherence remove the oddities of 
Copenhagen
as presented above in that it is not consciousness that does the 
work,
but decoherence. Bohr was saying essentially the same thing (though 
he

didn't know the words) when he talked about the importance of the
whole experimental set up.

 That aside, Deutsch's idea fails because he has not fully 
implemented

quantum erasure. If a record exists of the fact that a 'welcher weg'
measurement was made, entanglement of the rest of the world with the
result of that measurement is not erased by merely resetting the
memory of the mind or computer. So in the proposed experiment, the
interference pattern is absent, and it is not a proper 'delayed
choice' situation.

Deutsch has also given a rigorous variant of this that I summarized in 
my paper. It is possible in theory to experimentally prove the 
existence of other branches where a different measurement result was 
obtained. It's only impossible FAPP.


From your paper, summarizing Deutsch:
"Suppose an observer measures the z-component of a spin that is
polarized in the x-direction. Then there exits a unitary operator that
disentangles the observer from the spin, causing the observer to
forget the result of the measurement. However, he would still remember
having measured the z-component of the spin. In the MWI, the spin will
be in its original state and therefore measuring the x-component will
yield spin up with 100% probability. In any collapse interpretation,
measuring the x-component will yield spin up or spin down with 50%
probability."

This is equivalent to the argument John reported, and your reference
dates from 1985. That is before decoherence theory and entanglement
with the environment was fully understood, and predates the
significant quantum erasure experiments. For that reason, Deutsch has
got it seriously wrong, because the unitary operation he envisages
does not exist, even in principle, and he has not fully implemented
quantum erasure.

His scenario can be approximated in quantum erasure experiments, and
because the erasure is not complete, the results obtained will be
identical to those predicted by a collapse model. In order to use time
reversal invariance to recover the initial state in MWI, you have to
reverse absolutely everything, so you cannot leave a record in the
environment that some measurement was made -- such a record means that
there has not been compete reversal, so the original state is not
recovered.



For the record (due to lack of time I have to stop participating but I 
think it's worthwhile to make the point that David Deutsch was not 
wrong):


 This is not true, Deutsch has rigorously demonstrated the existence of 
a unitary transform that restores the original state save for the record 
that a measurement was made (a simple exercise if you replace spins by 
qubits and use CNOT and Hadamard transforms to show this).


Of course, decoherence will make it a practical impossibility to 
actually implement this using real spins,  but then you could in 
principle do it using quantum computers where you then need to simulate 
an entire observer.


The whole point of the thought experiment is to demonstrate the reality 
of both branches under unitary QM, in particular that a modification of 
QM that erases the alternative branches after a measurement is not 
equivalent to standard QM.  This is only the case FAPP.


That decoherence theory had yet to be worked out in detail, is not 
relevant. It's a bit like saying that Maxwell was wrong about Maxwell's 
demon because he didn't know all the details about how it wouldn't work 
in practice. The point of that discussion ultimately was about 
clarifying the relation between entropy and information. Similarly here 
it's about the structure of the multiverse, and decoherence theory 
yields practical constraints about such things as quantum computers, it 
doesn't make QM non-unitary.



Saibal

--
You received this message because you are subscribed to the Google Groups 
"Everything List" group.
To unsubscribe from this group and stop receiving emails from it, send an email 
to everything-list+unsubscr...@googlegroups.com.
To post to this group, send email to everything-list@googlegroups.com.
Visit this group at https://groups.google.com/group/everything-list.
For more options, visit https://groups.google.com/d/optout.


Re: Consistency of Postulates of QM

2017-12-14 Thread agrayson2000


On Wednesday, December 13, 2017 at 10:44:14 PM UTC, Brent wrote:
>
>
>
> On 12/13/2017 2:20 PM, agrays...@gmail.com  wrote:
>
>
>
> On Wednesday, December 13, 2017 at 9:15:36 PM UTC, Brent wrote: 
>>
>>
>>
>> On 12/13/2017 2:45 AM, agrays...@gmail.com wrote:
>>
>> * BUT for a nucleus of a radioactive element, the nucleus is never 
>> Decayed and Undecayed SIMULTANEOUSLY.*
>>
>> Sure it is.  It's in a coherent superposition of those states until it 
>> interacts with the environment.
>>
>> Brent
>>
>
> * That's the conventional QM wisdom and the cause of the paradox of a cat 
> Alive and Dead simultaneously. As I explained, the fallacy is rooted in an 
> unjustified generalization of the double slit experiment where the 
> probability waves do, in fact, exist simultaneously.  What waves do you 
> claim are interacting for the radioactive nucleus to produce coherence? 
> Tell me about them. I am from Missouri. AG*
>
>
> You seem to think that coherence requires two different waves.  This is 
> the wrong way to look at it.  In Young's slits experiment there is only one 
> wave, which goes through both slits and interferes with itself.  And 
> unstable nucleus has a probability amplitude that includes a "decayed" part 
> and a "not decayed" part.  It's a tunneling problem.
>
> Brent
>

*See,  https://en.wikipedia.org/wiki/Coherence_(physics*)

*How can you have coherence with a single wave? What does it mean? How 
would you define it? AG*

-- 
You received this message because you are subscribed to the Google Groups 
"Everything List" group.
To unsubscribe from this group and stop receiving emails from it, send an email 
to everything-list+unsubscr...@googlegroups.com.
To post to this group, send email to everything-list@googlegroups.com.
Visit this group at https://groups.google.com/group/everything-list.
For more options, visit https://groups.google.com/d/optout.


Re: Consistency of Postulates of QM

2017-12-14 Thread agrayson2000


On Wednesday, December 13, 2017 at 10:45:40 AM UTC, agrays...@gmail.com 
wrote:
>
>
>
> On Wednesday, December 13, 2017 at 7:41:00 AM UTC, agrays...@gmail.com 
> wrote:
>>
>>
>>
>> On Wednesday, December 13, 2017 at 5:55:59 AM UTC, Bruce wrote:
>>>
>>> On 13/12/2017 11:41 am, agrays...@gmail.com wrote:
>>>
>>> On Tuesday, December 12, 2017 at 10:52:12 PM UTC, Bruce wrote: 

 On 13/12/2017 9:45 am, agrays...@gmail.com wrote:

 On Tuesday, December 12, 2017 at 10:14:01 PM UTC, Bruce wrote: 
>
>
> So Schrödinger's cat was once a coherent state of a cat in a box, and 
> the splitting occurs with the decay of a nucleus; 


 *Unlike the double slit experiment, which can only be understood with 
 interference and the model that the electron wave, say, goes through both 
 slits, the nuclear decay is a BINARY event, very roughly like a coin toss 
 where there is no interference. Generalizing interference to every quantum 
 state is where Schrodinger went wrong. The cat, which shares or inherits 
 the wf from the radioactive decay, is never in both Alive and Dead states 
 simultaneously. AG*


 In the double slit, the paths through the two slits remain coherent, so 
 they can interfere when they come together. In the case of nuclear decay, 
 the coherence is lost immediately, so the nucleus does not interfere with 
 the decay products.

 Bruce

>>>
>>> *So you agree or disagree with my conclusion; namely, the cat is never 
>>> in a superposition of states? That is, no situation where cat is Alive and 
>>> Dead simultaneously. I think you disagree and think the nuclear state is 
>>> superposed with interference existing. AG*
>>>
>>>
>>> The superposition of |live + dead> or |live - dead> does not exist in 
>>> any single world since such states are not stable against decoherence. But 
>>> if you take the pedantic view of the many worlds of MWI, the superposition 
>>> of live and dead cats, together with everything entangled with them, exists 
>>> for ever in the multiverse. What good that does anyone, I fail to 
>>> understand.
>>>
>>> Bruce
>>>
>>
>> *I am not referring to the MWI. I am referring to whether in 
>> Schrodinger's cat experiment, the wf of the radioactive source, ( |decayed> 
>> + |undecayed> ) , is a superposition of states without interference between 
>> its components. If that's the case, perhaps what you would call an 
>> "incoherent superposition", then the cat which shares or inherits this wf 
>> in Schrodinger's set up, is never in a state of Alive and Dead 
>> simultaneously. AG*
>>
>> *You can look at it this way; in the double slit experiment, the 
>> probability wave of the electron, say, passes through both slits 
>> SIMULTANEOUSLY. As a result, it is conceptually feasible to model 
>> interference between the waves, giving rise to the correct distribution on 
>> the screen. This is the seminal experiment in QM and gives rise to the 
>> general interpretation that a system in a superposition can be in multiple 
>> states simultaneously. BUT for a nucleus of a radioactive element, the 
>> nucleus is never Decayed and Undecayed SIMULTANEOUSLY. Thus, it makes no 
>> sense to believe there can be interference between the states. Absent 
>> interference means the cat tied to the radioactive source can never be in 
>> the Alive and Dead states SIMULTANEOUSLY. AG*
>>
>
And the answer is (drum roll please); SHUT UP AND CALCULATE! AG

-- 
You received this message because you are subscribed to the Google Groups 
"Everything List" group.
To unsubscribe from this group and stop receiving emails from it, send an email 
to everything-list+unsubscr...@googlegroups.com.
To post to this group, send email to everything-list@googlegroups.com.
Visit this group at https://groups.google.com/group/everything-list.
For more options, visit https://groups.google.com/d/optout.


Re: Consistency of Postulates of QM

2017-12-14 Thread Bruce Kellett

On 15/12/2017 10:15 am, smitra wrote:

On 14-12-2017 22:35, Bruce Kellett wrote:


 I think this argument pre-dates the work by Zeh and Zurek developing
the idea of decoherence. Decoherence remove the oddities of Copenhagen
as presented above in that it is not consciousness that does the work,
but decoherence. Bohr was saying essentially the same thing (though he
didn't know the words) when he talked about the importance of the
whole experimental set up.

 That aside, Deutsch's idea fails because he has not fully implemented
quantum erasure. If a record exists of the fact that a 'welcher weg'
measurement was made, entanglement of the rest of the world with the
result of that measurement is not erased by merely resetting the
memory of the mind or computer. So in the proposed experiment, the
interference pattern is absent, and it is not a proper 'delayed
choice' situation.

Deutsch has also given a rigorous variant of this that I summarized in 
my paper. It is possible in theory to experimentally prove the 
existence of other branches where a different measurement result was 
obtained. It's only impossible FAPP.


From your paper, summarizing Deutsch:
"Suppose an observer measures the z-component of a spin that is 
polarized in the x-direction. Then there exits a unitary operator that 
disentangles the observer from the spin, causing the observer to forget 
the result of the measurement. However, he would still remember having 
measured the z-component of the spin. In the MWI, the spin will be in 
its original state and therefore measuring the x-component will yield 
spin up with 100% probability. In any collapse interpretation, measuring 
the x-component will yield spin up or spin down with 50% probability."


This is equivalent to the argument John reported, and your reference 
dates from 1985. That is before decoherence theory and entanglement with 
the environment was fully understood, and predates the significant 
quantum erasure experiments. For that reason, Deutsch has got it 
seriously wrong, because the unitary operation he envisages does not 
exist, even in principle, and he has not fully implemented quantum erasure.


His scenario can be approximated in quantum erasure experiments, and 
because the erasure is not complete, the results obtained will be 
identical to those predicted by a collapse model. In order to use time 
reversal invariance to recover the initial state in MWI, you have to 
reverse absolutely everything, so you cannot leave a record in the 
environment that some measurement was made -- such a record means that 
there has not been compete reversal, so the original state is not recovered.


Bruce

--
You received this message because you are subscribed to the Google Groups 
"Everything List" group.
To unsubscribe from this group and stop receiving emails from it, send an email 
to everything-list+unsubscr...@googlegroups.com.
To post to this group, send email to everything-list@googlegroups.com.
Visit this group at https://groups.google.com/group/everything-list.
For more options, visit https://groups.google.com/d/optout.


Re: Consistency of Postulates of QM

2017-12-14 Thread Bruce Kellett

On 15/12/2017 10:08 am, smitra wrote:

On 14-12-2017 22:39, Bruce Kellett wrote:


The thing about mixed states is that they are inevitable if you write
the state of a system as a tensor product of the separate states of
subsystems. The separate subsystems are not pure states because of
entanglement between them.

There is entanglement with the environment but the same computational 
state appears in different "decoherent" branches.


This is conjecture only -- has not been demonstrated.

You don't per se need to invoke any mixed states. Also, it's a well 
known fact that any mixed state can be considered to be a pure state 
of a larger system, the so-called "purification" of the state.


Maybe, but then your calculation doesn't go through.

But all this is rather beside the point. We can talk about your paper 
separately if you wish, but the main point is that you still have not 
risen to the challenge I offered: start from the decohered state |coin> 
and derive the state {|heads > + |tails>} by purely unitary interactions 
with the environment -- such as by shaking etc (Not by a contrived 
entanglement with some single quantum process such as a nuclear decay.)


Bruce

--
You received this message because you are subscribed to the Google Groups 
"Everything List" group.
To unsubscribe from this group and stop receiving emails from it, send an email 
to everything-list+unsubscr...@googlegroups.com.
To post to this group, send email to everything-list@googlegroups.com.
Visit this group at https://groups.google.com/group/everything-list.
For more options, visit https://groups.google.com/d/optout.


Re: Consistency of Postulates of QM

2017-12-14 Thread smitra

On 15-12-2017 00:08, smitra wrote:

On 14-12-2017 22:39, Bruce Kellett wrote:

On 14/12/2017 11:20 pm, smitra wrote:

On 14-12-2017 12:43, Bruce Kellett wrote:

On 14/12/2017 9:23 pm, smitra wrote:

On 14-12-2017 02:35, Bruce Kellett wrote:

On 14/12/2017 11:52 am, smitra wrote:

On 13-12-2017 22:55, Bruce Kellett wrote:

On 14/12/2017 8:23 am, smitra wrote:


So, let's examine this more closely. We start with a state that 
is a superposition of branches that each undergo classical 
evolution, so I'm not now appealing to the arguments in the 
paper by Albrecht. Then we end up with a state of the form:


 Sum over j [|heads(j)>|env(heads,j)> +

 Sum over j |tails(j)>|env(tails,j)>]

where j enumerates microstates of the coin and the state of the 
environment in each sector where the coin is heads or tails 
depends on the microstate of the coin. The coin states are not 
normalized, the norms are chosen to yield the correct 
probabilities.  E.g. if tails has zero probability then all the 
|tail(j)> are given a norm of zero. So, the above expression is 
completely general.


Now, I'm part of the environment, so we can write:

|env(X,j)> = Sum over k of |me(X,Y,j,k)>|env'(X,Y,j,k)>

where X is heads or tails, Y, denotes my macrostate that I'll 
define below, j denotes the microstate of the coin and k sums 
over microstates corresponding to macrostate Y.


This splitting of my state into a macrostate Y and microstate k 
is in principle arbitrary, we can choose this splitting such 
that Y keep tracts of what I'm aware of and then k takes into 
account all the other degrees of freedom in my body and brain 
that I'm not aware of. E.g. if I were a digital computer then Y 
would correspond to some bitstring defined by the computational 
state of the computer and we then sum over all the possible 
microstates that correspond to some fixed macrostate.


Then when I'm not yet aware of the result of the coin throw, we 
have:


|me(X,Y,j,k)> = |Awareness(Y)>|body(X,Y,j,k)>

where Y contains all the information that I am aware of, and 
that doesn't include the result if the coin throw nor 
information contained only at the microscopic scale. So, 
Awareness (Y) for a given Y defines who I am, where I am , and 
what I'm experiencing.


It should be clear that despite decoherence, |Awareness(Y)> 
will factor out of the global superposition as it doesn't 
depend on X, k and j.


So, given what I know, I cannot tell in which branch I am. 
Therefore unless all the branches corresponding to one outcome 
have zero norm,  I will have copies in both branches.


There is just no way that all the information describing what 
I'm aware of at some moment is going to constrain how someone 
else can throw a coin to such a degree that the outcome will be 
fixed.


So the end point of your analysis is that you can't see how it 
could
be possible that you are not right? The outcome is fixed -- and 
the

same -- in all worlds in which you participate!

But it is quite easy to see what is going on. Since it is all
independent of your awareness, we can factor that out of the
summations. And you still do not get a coherent superposition 
from a

decoherent mixture. What goes on in the world is independent of
whether or not the rock on the floor is aware of it or not. You 
are
not in a position any different from that of said rock. Look at 
the

expansion for Schrödinger's cat:

  |nucleus>|box>|cat>you>|environment> -->

 {|decayed>|poison spilt>|cat dead>|You>|environment records 
dead cat>
 + |undecayed>|poison intact>|cat alive>|You>|environment 
records live cat>}


You can see that your state does not change (is irrelevant) 
until you
interact with the environment in such a way that your 
consciousness
becomes entangled with the state of the cat. This is not 
mysterious,

and it is not changed just because you claim there is some magic
associated with consciousness that makes a difference. You are 
just
reverting to Copenhagen or many minds where consciousness is 
necessary

for the understanding of QM.



The lack of coherence is not relevant unless it affects the two 
states denoted by |you>.


That is not true.

If you have a coherent superposition then it's clear that the two 
|you>'s are identical.


No, it is not. In the above expansion of the cat scenario, I have 
a
coherent superposition of live and dead cats -- in different 
branches

of course, but until decoherence separates the worlds, the
superposition is intact. And Bruno would claim that it is never
broken. So your awareness (or not) of the facts about the world or 
not

does not affect anything.

If it isn't then it's not clear if that's the case or not and one 
needs to take a deeper look. However, it's then also a 
philosophical issue as probabilities are not going to be 
different from a classical analysis.


There is nothing wrong to invoke consciousness, walking away from 
that just because that's the popular thing to do, doesn't make it 
right. What I'm arguing for 

Re: Consistency of Postulates of QM

2017-12-14 Thread smitra

On 14-12-2017 22:35, Bruce Kellett wrote:

On 15/12/2017 6:25 am, John Clark wrote:


On Wed, Dec 13, 2017 at 10:38 PM,  wrote:


​ > ​ I notice you don't gave a damn about having a non
falsifiable theory.


David Deutsch proposed a test of Many Worlds about 30 years ago in
his book "The Ghost In The Atom", but
​ it ​ would be very difficult to perform. The reason it's so
difficult to test is not
​ the ​M any
​ World's
​ ​ theory fault, the reason is that the conventional view says
that conscious
​ ​ observers obey different laws of physics, many worlds says
they do not, so
​ ​ to test who's right we need a mind that uses quantum
properties.
​ ​ Quantum Computers have advanced enormously over the last 30
years
​ so​ I wouldn't be surprised if it or something very much like
it
​ is ​ actually performed in the decade or two.

​ An intelligent quantum computer shoots
​ ​ photons at a metal plate
​ ​ one at a time
​ ​ that has 2 small slits in
​ ​it, and then the photons hit a photographic plate.
​ ​ Nobody looks at the photographic plate till the very end of
the experiment.
​ The quantum mind has detectors near each slit so it knows which
slit
​ ​ the various electrons went through.
​ ​ After each photon passes the slits but before they hit the
photographic plate the quantum mind signs a document
​ ​ saying that it has observed each and every
​ photon​ and knows
​ which​ slit each
​ ​ photon
​ ​ went through. It is very important that the document does
not say
​ ​ which slit
​ ​ any
​ ​photon went through, it only says that they went through
​ ​ one slit
​ ​ and one slit only and the mind has knowledge of which one.
​ ​ There is a signed document to this effect for every photon
it shot.

Now the mind uses
​ ​ quantum erasure to completely destroy
​ its​ memory of
​ which slit any of the ​photons went through; t he only
​ ​ part remaining is the document
​ which states that each photon went through one and only one slit
and the mind (at the time) knew which one. ​ Now develop the
photographic plate and look at it.
​ I​ f you see interference bands then the many world
interpretation is
​ ​ correct. If you do not see interference bands then there are
no worlds but
​ ​ this one and the conventional
​ quantum ​ interpretation is correct.

​This works because ​in the Copenhagen interpretation when the
results of a measurement enters the consciousness of an observer the
wave function collapses, in effect all the universes except one
disappear without a trace so you get no interference. In the many
worlds model all the other worlds will converge back into one
universe because information on which slit the various photons went
through was the only thing that made one universe different from
another, so when that was erased they became identical again and
merged, but their influence will still be felt, you'll see
indications that the photon went through slot A only and indications
it went through slot B only, and that's what causes interference.


 I think this argument pre-dates the work by Zeh and Zurek developing
the idea of decoherence. Decoherence remove the oddities of Copenhagen
as presented above in that it is not consciousness that does the work,
but decoherence. Bohr was saying essentially the same thing (though he
didn't know the words) when he talked about the importance of the
whole experimental set up.

 That aside, Deutsch's idea fails because he has not fully implemented
quantum erasure. If a record exists of the fact that a 'welcher weg'
measurement was made, entanglement of the rest of the world with the
result of that measurement is not erased by merely resetting the
memory of the mind or computer. So in the proposed experiment, the
interference pattern is absent, and it is not a proper 'delayed
choice' situation.

Deutsch has also given a rigorous variant of this that I summarized in 
my paper. It is possible in theory to experimentally prove the existence 
of other branches where a different measurement result was obtained. 
It's only impossible FAPP.


Saibal

--
You received this message because you are subscribed to the Google Groups 
"Everything List" group.
To unsubscribe from this group and stop receiving emails from it, send an email 
to everything-list+unsubscr...@googlegroups.com.
To post to this group, send email to everything-list@googlegroups.com.
Visit this group at https://groups.google.com/group/everything-list.
For more options, visit https://groups.google.com/d/optout.


Re: Consistency of Postulates of QM

2017-12-14 Thread smitra

On 14-12-2017 22:39, Bruce Kellett wrote:

On 14/12/2017 11:20 pm, smitra wrote:

On 14-12-2017 12:43, Bruce Kellett wrote:

On 14/12/2017 9:23 pm, smitra wrote:

On 14-12-2017 02:35, Bruce Kellett wrote:

On 14/12/2017 11:52 am, smitra wrote:

On 13-12-2017 22:55, Bruce Kellett wrote:

On 14/12/2017 8:23 am, smitra wrote:


So, let's examine this more closely. We start with a state that 
is a superposition of branches that each undergo classical 
evolution, so I'm not now appealing to the arguments in the 
paper by Albrecht. Then we end up with a state of the form:


 Sum over j [|heads(j)>|env(heads,j)> +

 Sum over j |tails(j)>|env(tails,j)>]

where j enumerates microstates of the coin and the state of the 
environment in each sector where the coin is heads or tails 
depends on the microstate of the coin. The coin states are not 
normalized, the norms are chosen to yield the correct 
probabilities.  E.g. if tails has zero probability then all the 
|tail(j)> are given a norm of zero. So, the above expression is 
completely general.


Now, I'm part of the environment, so we can write:

|env(X,j)> = Sum over k of |me(X,Y,j,k)>|env'(X,Y,j,k)>

where X is heads or tails, Y, denotes my macrostate that I'll 
define below, j denotes the microstate of the coin and k sums 
over microstates corresponding to macrostate Y.


This splitting of my state into a macrostate Y and microstate k 
is in principle arbitrary, we can choose this splitting such 
that Y keep tracts of what I'm aware of and then k takes into 
account all the other degrees of freedom in my body and brain 
that I'm not aware of. E.g. if I were a digital computer then Y 
would correspond to some bitstring defined by the computational 
state of the computer and we then sum over all the possible 
microstates that correspond to some fixed macrostate.


Then when I'm not yet aware of the result of the coin throw, we 
have:


|me(X,Y,j,k)> = |Awareness(Y)>|body(X,Y,j,k)>

where Y contains all the information that I am aware of, and 
that doesn't include the result if the coin throw nor 
information contained only at the microscopic scale. So, 
Awareness (Y) for a given Y defines who I am, where I am , and 
what I'm experiencing.


It should be clear that despite decoherence, |Awareness(Y)> will 
factor out of the global superposition as it doesn't depend on 
X, k and j.


So, given what I know, I cannot tell in which branch I am. 
Therefore unless all the branches corresponding to one outcome 
have zero norm,  I will have copies in both branches.


There is just no way that all the information describing what 
I'm aware of at some moment is going to constrain how someone 
else can throw a coin to such a degree that the outcome will be 
fixed.


So the end point of your analysis is that you can't see how it 
could
be possible that you are not right? The outcome is fixed -- and 
the

same -- in all worlds in which you participate!

But it is quite easy to see what is going on. Since it is all
independent of your awareness, we can factor that out of the
summations. And you still do not get a coherent superposition 
from a

decoherent mixture. What goes on in the world is independent of
whether or not the rock on the floor is aware of it or not. You 
are
not in a position any different from that of said rock. Look at 
the

expansion for Schrödinger's cat:

  |nucleus>|box>|cat>you>|environment> -->

 {|decayed>|poison spilt>|cat dead>|You>|environment records dead 
cat>
 + |undecayed>|poison intact>|cat alive>|You>|environment records 
live cat>}


You can see that your state does not change (is irrelevant) until 
you
interact with the environment in such a way that your 
consciousness
becomes entangled with the state of the cat. This is not 
mysterious,

and it is not changed just because you claim there is some magic
associated with consciousness that makes a difference. You are 
just
reverting to Copenhagen or many minds where consciousness is 
necessary

for the understanding of QM.



The lack of coherence is not relevant unless it affects the two 
states denoted by |you>.


That is not true.

If you have a coherent superposition then it's clear that the two 
|you>'s are identical.


No, it is not. In the above expansion of the cat scenario, I have a
coherent superposition of live and dead cats -- in different 
branches

of course, but until decoherence separates the worlds, the
superposition is intact. And Bruno would claim that it is never
broken. So your awareness (or not) of the facts about the world or 
not

does not affect anything.

If it isn't then it's not clear if that's the case or not and one 
needs to take a deeper look. However, it's then also a 
philosophical issue as probabilities are not going to be different 
from a classical analysis.


There is nothing wrong to invoke consciousness, walking away from 
that just because that's the popular thing to do, doesn't make it 
right. What I'm arguing for is to define conscious experience as 

Re: Consistency of Postulates of QM

2017-12-14 Thread Bruce Kellett

On 14/12/2017 11:20 pm, smitra wrote:

On 14-12-2017 12:43, Bruce Kellett wrote:

On 14/12/2017 9:23 pm, smitra wrote:

On 14-12-2017 02:35, Bruce Kellett wrote:

On 14/12/2017 11:52 am, smitra wrote:

On 13-12-2017 22:55, Bruce Kellett wrote:

On 14/12/2017 8:23 am, smitra wrote:


So, let's examine this more closely. We start with a state that 
is a superposition of branches that each undergo classical 
evolution, so I'm not now appealing to the arguments in the 
paper by Albrecht. Then we end up with a state of the form:


 Sum over j [|heads(j)>|env(heads,j)> +

 Sum over j |tails(j)>|env(tails,j)>]

where j enumerates microstates of the coin and the state of the 
environment in each sector where the coin is heads or tails 
depends on the microstate of the coin. The coin states are not 
normalized, the norms are chosen to yield the correct 
probabilities.  E.g. if tails has zero probability then all the 
|tail(j)> are given a norm of zero. So, the above expression is 
completely general.


Now, I'm part of the environment, so we can write:

|env(X,j)> = Sum over k of |me(X,Y,j,k)>|env'(X,Y,j,k)>

where X is heads or tails, Y, denotes my macrostate that I'll 
define below, j denotes the microstate of the coin and k sums 
over microstates corresponding to macrostate Y.


This splitting of my state into a macrostate Y and microstate k 
is in principle arbitrary, we can choose this splitting such 
that Y keep tracts of what I'm aware of and then k takes into 
account all the other degrees of freedom in my body and brain 
that I'm not aware of. E.g. if I were a digital computer then Y 
would correspond to some bitstring defined by the computational 
state of the computer and we then sum over all the possible 
microstates that correspond to some fixed macrostate.


Then when I'm not yet aware of the result of the coin throw, we 
have:


|me(X,Y,j,k)> = |Awareness(Y)>|body(X,Y,j,k)>

where Y contains all the information that I am aware of, and 
that doesn't include the result if the coin throw nor 
information contained only at the microscopic scale. So, 
Awareness (Y) for a given Y defines who I am, where I am , and 
what I'm experiencing.


It should be clear that despite decoherence, |Awareness(Y)> will 
factor out of the global superposition as it doesn't depend on 
X, k and j.


So, given what I know, I cannot tell in which branch I am. 
Therefore unless all the branches corresponding to one outcome 
have zero norm,  I will have copies in both branches.


There is just no way that all the information describing what 
I'm aware of at some moment is going to constrain how someone 
else can throw a coin to such a degree that the outcome will be 
fixed.


So the end point of your analysis is that you can't see how it could
be possible that you are not right? The outcome is fixed -- and the
same -- in all worlds in which you participate!

But it is quite easy to see what is going on. Since it is all
independent of your awareness, we can factor that out of the
summations. And you still do not get a coherent superposition from a
decoherent mixture. What goes on in the world is independent of
whether or not the rock on the floor is aware of it or not. You are
not in a position any different from that of said rock. Look at the
expansion for Schrödinger's cat:

  |nucleus>|box>|cat>you>|environment> -->

 {|decayed>|poison spilt>|cat dead>|You>|environment records dead 
cat>
 + |undecayed>|poison intact>|cat alive>|You>|environment records 
live cat>}


You can see that your state does not change (is irrelevant) until 
you

interact with the environment in such a way that your consciousness
becomes entangled with the state of the cat. This is not mysterious,
and it is not changed just because you claim there is some magic
associated with consciousness that makes a difference. You are just
reverting to Copenhagen or many minds where consciousness is 
necessary

for the understanding of QM.



The lack of coherence is not relevant unless it affects the two 
states denoted by |you>.


That is not true.

If you have a coherent superposition then it's clear that the two 
|you>'s are identical.


No, it is not. In the above expansion of the cat scenario, I have a
coherent superposition of live and dead cats -- in different branches
of course, but until decoherence separates the worlds, the
superposition is intact. And Bruno would claim that it is never
broken. So your awareness (or not) of the facts about the world or not
does not affect anything.

If it isn't then it's not clear if that's the case or not and one 
needs to take a deeper look. However, it's then also a 
philosophical issue as probabilities are not going to be different 
from a classical analysis.


There is nothing wrong to invoke consciousness, walking away from 
that just because that's the popular thing to do, doesn't make it 
right. What I'm arguing for is to define conscious experience as 
the computational state of the relevant machine (or 

Re: Consistency of Postulates of QM

2017-12-14 Thread Bruce Kellett

On 15/12/2017 6:25 am, John Clark wrote:
On Wed, Dec 13, 2017 at 10:38 PM, >wrote:


​ > ​
I notice you don't gave a damn about having a non falsifiable theory.


David Deutsch proposed a test of Many Worlds about 30 years ago in his 
book "The Ghost In The Atom", but

​ it ​
would be very difficult to perform. The reason it's so difficult to 
test is not

​ the ​M
any
​ World's
​ ​
theory fault, the reason is that the conventional view says that 
conscious

​ ​
observers obey different laws of physics, many worlds says they do 
not, so

​ ​
to test who's right we need a mind that uses quantum properties.
​ ​
Quantum Computers have advanced enormously over the last 30 years
​ so​
 I wouldn't be surprised if it or something very much like it
​ is ​
actually performed in the decade or two.

​
An intelligent quantum computer shoots
​ ​
photons at a metal plate
​ ​
one at a time
​ ​
that has 2 small slits in
​ ​it,
and then the photons hit a photographic plate.
​ ​
Nobody looks at the photographic plate till the very end of the 
experiment.

​
The quantum mind has detectors near each slit so it knows which slit
​ ​
the various electrons went through.
​ ​
After each photon passes the slits but before they hit the 
photographic plate the quantum mind signs a document

​ ​
saying that it has observed each and every
​ photon​
 and knows
​ which​
 slit each
​ ​
photon
​ ​
went through. It is very important that the document does not say
​ ​
which slit
​ ​
any
​ ​photon
 went through, it only says that they went through
​ ​
one slit
​ ​
and one slit only and the mind has knowledge of which one.
​ ​
There is a signed document to this effect for every photon it shot.

Now the mind uses
​ ​
quantum erasure to completely destroy
​ its​
 memory of
​ which slit any of the ​photons went through; t
he only
​ ​
part remaining is the document
​ which states that each photon went through one and only one slit and 
the mind (at the time) knew which one. ​

Now develop the photographic plate and look at it.
​ I​
f you see interference bands then the many world interpretation is
​ ​
correct. If you do not see interference bands then there are no worlds 
but

​ ​
this one and the conventional
​ quantum ​
interpretation is correct.

​This works because ​in the Copenhagen interpretation when the results 
of a measurement enters the consciousness of an observer the wave 
function collapses, in effect all the universes except one disappear 
without a trace so you get no interference. In the many worlds model 
all the other worlds will converge back into one universe because 
information on which slit the various photons went through was the 
only thing that made one universe different from another, so when that 
was erased they became identical again and merged, but their influence 
will still be felt, you'll see indications that the photon went 
through slot A only and indications it went through slot B only, and 
that's what causes interference.


I think this argument pre-dates the work by Zeh and Zurek developing the 
idea of decoherence. Decoherence remove the oddities of Copenhagen as 
presented above in that it is not consciousness that does the work, but 
decoherence. Bohr was saying essentially the same thing (though he 
didn't know the words) when he talked about the importance of the whole 
experimental set up.


That aside, Deutsch's idea fails because he has not fully implemented 
quantum erasure. If a record exists of the fact that a 'welcher weg' 
measurement was made, entanglement of the rest of the world with the 
result of that measurement is not erased by merely resetting the memory 
of the mind or computer. So in the proposed experiment, the interference 
pattern is absent, and it is not a proper 'delayed choice' situation.


Bruce

--
You received this message because you are subscribed to the Google Groups 
"Everything List" group.
To unsubscribe from this group and stop receiving emails from it, send an email 
to everything-list+unsubscr...@googlegroups.com.
To post to this group, send email to everything-list@googlegroups.com.
Visit this group at https://groups.google.com/group/everything-list.
For more options, visit https://groups.google.com/d/optout.


Re: Consistency of Postulates of QM

2017-12-14 Thread John Clark
On Wed, Dec 13, 2017 at 10:38 PM,  wrote:

​> ​
> I notice you don't gave a damn about having a non falsifiable theory.
>

David Deutsch proposed a test of Many Worlds about 30 years ago in his book
"The Ghost In The Atom", but
​it ​
would be very difficult to perform. The reason it's so difficult to test is
not
​the ​M
any
​World's
​ ​
theory fault, the reason is that the conventional view says that conscious
​ ​
observers obey different laws of physics, many worlds says they do not, so
​ ​
to test who's right we need a mind that uses quantum properties.
​ ​
Quantum Computers have advanced enormously over the last 30 years
​so​
 I wouldn't be surprised if it or something very much like it
​is ​
actually performed in the decade or two.

​
An intelligent quantum computer shoots
​ ​
photons at a metal plate
​ ​
one at a time
​ ​
that has 2 small slits in
​ ​it,
and then the photons hit a photographic plate.
​ ​
Nobody looks at the photographic plate till the very end of the experiment.
​
The quantum mind has detectors near each slit so it knows which slit
​ ​
the various electrons went through.
​ ​
After each photon passes the slits but before they hit the photographic
plate the quantum mind signs a document
​ ​
saying that it has observed each and every
​photon​
 and knows
​which​
 slit each
​ ​
photon
​ ​
went through. It is very important that the document does not say
​ ​
which slit
​ ​
any
​ ​photon
 went through, it only says that they went through
​ ​
one slit
​ ​
and one slit only and the mind has knowledge of which one.
​ ​
There is a signed document to this effect for every photon it shot.

Now the mind uses
​ ​
quantum erasure to completely destroy
​its​
 memory of
​which slit any of the ​photons went through; t
he only
​ ​
part remaining is the document
​ which states that each photon went through one and only one slit and the
mind (at the time) knew which one. ​
Now develop the photographic plate and look at it.
​I​
f you see interference bands then the many world interpretation is
​ ​
correct. If you do not see interference bands then there are no worlds but
​ ​
this one and the conventional
​quantum ​
interpretation is correct.

​This works because ​in the Copenhagen interpretation when the results of a
measurement enters the consciousness of an observer the wave function
collapses, in effect all the universes except one disappear without a trace
so you get no interference. In the many worlds model all the other worlds
will converge back into one universe because information on which slit
the various photons went through was the only thing that made one universe
different from another, so when that was erased they became identical again
and merged, but their influence will still be felt, you'll see indications
that the photon went through slot A only and indications it went through
slot B only, and that's what causes interference.

  John K Clark

-- 
You received this message because you are subscribed to the Google Groups 
"Everything List" group.
To unsubscribe from this group and stop receiving emails from it, send an email 
to everything-list+unsubscr...@googlegroups.com.
To post to this group, send email to everything-list@googlegroups.com.
Visit this group at https://groups.google.com/group/everything-list.
For more options, visit https://groups.google.com/d/optout.


Re: Consistency of Postulates of QM

2017-12-14 Thread smitra

On 14-12-2017 12:43, Bruce Kellett wrote:

On 14/12/2017 9:23 pm, smitra wrote:

On 14-12-2017 02:35, Bruce Kellett wrote:

On 14/12/2017 11:52 am, smitra wrote:

On 13-12-2017 22:55, Bruce Kellett wrote:

On 14/12/2017 8:23 am, smitra wrote:


So, let's examine this more closely. We start with a state that is 
a superposition of branches that each undergo classical evolution, 
so I'm not now appealing to the arguments in the paper by 
Albrecht. Then we end up with a state of the form:


 Sum over j [|heads(j)>|env(heads,j)> +

 Sum over j |tails(j)>|env(tails,j)>]

where j enumerates microstates of the coin and the state of the 
environment in each sector where the coin is heads or tails 
depends on the microstate of the coin. The coin states are not 
normalized, the norms are chosen to yield the correct 
probabilities.  E.g. if tails has zero probability then all the 
|tail(j)> are given a norm of zero. So, the above expression is 
completely general.


Now, I'm part of the environment, so we can write:

|env(X,j)> = Sum over k of |me(X,Y,j,k)>|env'(X,Y,j,k)>

where X is heads or tails, Y, denotes my macrostate that I'll 
define below, j denotes the microstate of the coin and k sums over 
microstates corresponding to macrostate Y.


This splitting of my state into a macrostate Y and microstate k is 
in principle arbitrary, we can choose this splitting such that Y 
keep tracts of what I'm aware of and then k takes into account all 
the other degrees of freedom in my body and brain that I'm not 
aware of. E.g. if I were a digital computer then Y would 
correspond to some bitstring defined by the computational state of 
the computer and we then sum over all the possible microstates 
that correspond to some fixed macrostate.


Then when I'm not yet aware of the result of the coin throw, we 
have:


|me(X,Y,j,k)> = |Awareness(Y)>|body(X,Y,j,k)>

where Y contains all the information that I am aware of, and that 
doesn't include the result if the coin throw nor information 
contained only at the microscopic scale. So, Awareness (Y) for a 
given Y defines who I am, where I am , and what I'm experiencing.


It should be clear that despite decoherence, |Awareness(Y)> will 
factor out of the global superposition as it doesn't depend on X, 
k and j.


So, given what I know, I cannot tell in which branch I am. 
Therefore unless all the branches corresponding to one outcome 
have zero norm,  I will have copies in both branches.


There is just no way that all the information describing what I'm 
aware of at some moment is going to constrain how someone else can 
throw a coin to such a degree that the outcome will be fixed.


So the end point of your analysis is that you can't see how it 
could

be possible that you are not right? The outcome is fixed -- and the
same -- in all worlds in which you participate!

But it is quite easy to see what is going on. Since it is all
independent of your awareness, we can factor that out of the
summations. And you still do not get a coherent superposition from 
a

decoherent mixture. What goes on in the world is independent of
whether or not the rock on the floor is aware of it or not. You are
not in a position any different from that of said rock. Look at the
expansion for Schrödinger's cat:

  |nucleus>|box>|cat>you>|environment> -->

 {|decayed>|poison spilt>|cat dead>|You>|environment records dead 
cat>
 + |undecayed>|poison intact>|cat alive>|You>|environment records 
live cat>}


You can see that your state does not change (is irrelevant) until 
you

interact with the environment in such a way that your consciousness
becomes entangled with the state of the cat. This is not 
mysterious,

and it is not changed just because you claim there is some magic
associated with consciousness that makes a difference. You are just
reverting to Copenhagen or many minds where consciousness is 
necessary

for the understanding of QM.



The lack of coherence is not relevant unless it affects the two 
states denoted by |you>.


That is not true.

If you have a coherent superposition then it's clear that the two 
|you>'s are identical.


No, it is not. In the above expansion of the cat scenario, I have a
coherent superposition of live and dead cats -- in different branches
of course, but until decoherence separates the worlds, the
superposition is intact. And Bruno would claim that it is never
broken. So your awareness (or not) of the facts about the world or 
not

does not affect anything.

If it isn't then it's not clear if that's the case or not and one 
needs to take a deeper look. However, it's then also a philosophical 
issue as probabilities are not going to be different from a  
classical analysis.


There is nothing wrong to invoke consciousness, walking away from 
that just because that's the popular thing to do, doesn't make it 
right. What I'm arguing for is to define conscious experience as the 
computational state of the relevant machine (or brain), which is 
then given by some 

Re: Consistency of Postulates of QM

2017-12-14 Thread Bruce Kellett

On 14/12/2017 9:23 pm, smitra wrote:

On 14-12-2017 02:35, Bruce Kellett wrote:

On 14/12/2017 11:52 am, smitra wrote:

On 13-12-2017 22:55, Bruce Kellett wrote:

On 14/12/2017 8:23 am, smitra wrote:


So, let's examine this more closely. We start with a state that is 
a superposition of branches that each undergo classical evolution, 
so I'm not now appealing to the arguments in the paper by 
Albrecht. Then we end up with a state of the form:


 Sum over j [|heads(j)>|env(heads,j)> +

 Sum over j |tails(j)>|env(tails,j)>]

where j enumerates microstates of the coin and the state of the 
environment in each sector where the coin is heads or tails 
depends on the microstate of the coin. The coin states are not 
normalized, the norms are chosen to yield the correct 
probabilities.  E.g. if tails has zero probability then all the 
|tail(j)> are given a norm of zero. So, the above expression is 
completely general.


Now, I'm part of the environment, so we can write:

|env(X,j)> = Sum over k of |me(X,Y,j,k)>|env'(X,Y,j,k)>

where X is heads or tails, Y, denotes my macrostate that I'll 
define below, j denotes the microstate of the coin and k sums over 
microstates corresponding to macrostate Y.


This splitting of my state into a macrostate Y and microstate k is 
in principle arbitrary, we can choose this splitting such that Y 
keep tracts of what I'm aware of and then k takes into account all 
the other degrees of freedom in my body and brain that I'm not 
aware of. E.g. if I were a digital computer then Y would 
correspond to some bitstring defined by the computational state of 
the computer and we then sum over all the possible microstates 
that correspond to some fixed macrostate.


Then when I'm not yet aware of the result of the coin throw, we have:

|me(X,Y,j,k)> = |Awareness(Y)>|body(X,Y,j,k)>

where Y contains all the information that I am aware of, and that 
doesn't include the result if the coin throw nor information 
contained only at the microscopic scale. So, Awareness (Y) for a 
given Y defines who I am, where I am , and what I'm experiencing.


It should be clear that despite decoherence, |Awareness(Y)> will 
factor out of the global superposition as it doesn't depend on X, 
k and j.


So, given what I know, I cannot tell in which branch I am. 
Therefore unless all the branches corresponding to one outcome 
have zero norm,  I will have copies in both branches.


There is just no way that all the information describing what I'm 
aware of at some moment is going to constrain how someone else can 
throw a coin to such a degree that the outcome will be fixed.


So the end point of your analysis is that you can't see how it could
be possible that you are not right? The outcome is fixed -- and the
same -- in all worlds in which you participate!

But it is quite easy to see what is going on. Since it is all
independent of your awareness, we can factor that out of the
summations. And you still do not get a coherent superposition from a
decoherent mixture. What goes on in the world is independent of
whether or not the rock on the floor is aware of it or not. You are
not in a position any different from that of said rock. Look at the
expansion for Schrödinger's cat:

  |nucleus>|box>|cat>you>|environment> -->

 {|decayed>|poison spilt>|cat dead>|You>|environment records dead cat>
 + |undecayed>|poison intact>|cat alive>|You>|environment records 
live cat>}


You can see that your state does not change (is irrelevant) until you
interact with the environment in such a way that your consciousness
becomes entangled with the state of the cat. This is not mysterious,
and it is not changed just because you claim there is some magic
associated with consciousness that makes a difference. You are just
reverting to Copenhagen or many minds where consciousness is necessary
for the understanding of QM.



The lack of coherence is not relevant unless it affects the two 
states denoted by |you>.


That is not true.

If you have a coherent superposition then it's clear that the two 
|you>'s are identical.


No, it is not. In the above expansion of the cat scenario, I have a
coherent superposition of live and dead cats -- in different branches
of course, but until decoherence separates the worlds, the
superposition is intact. And Bruno would claim that it is never
broken. So your awareness (or not) of the facts about the world or not
does not affect anything.

If it isn't then it's not clear if that's the case or not and one 
needs to take a deeper look. However, it's then also a philosophical 
issue as probabilities are not going to be different from a  
classical analysis.


There is nothing wrong to invoke consciousness, walking away from 
that just because that's the popular thing to do, doesn't make it 
right. What I'm arguing for is to define conscious experience as the 
computational state of the relevant machine (or brain), which is 
then given by some bistring. This can then be included in the 
quantum state 

Re: Consistency of Postulates of QM

2017-12-14 Thread smitra

On 14-12-2017 02:35, Bruce Kellett wrote:

On 14/12/2017 11:52 am, smitra wrote:

On 13-12-2017 22:55, Bruce Kellett wrote:

On 14/12/2017 8:23 am, smitra wrote:

On 12-12-2017 23:13, Bruce Kellett wrote:

On 13/12/2017 2:12 am, smitra wrote:

On 12-12-2017 12:33, Bruce Kellett wrote:

On 12/12/2017 9:46 pm, smitra wrote:


Yes, it's only an estimation but it yields a good order of 
magnitude estimate for the center of mass. What the calculation 
shows is that quantum superpositions do exists at the 
macroscopic level and these can then be amplified by chaotic 
dynamics. Of course,  it then becomes incoherent, but in the MWI 
that's besides the point.


MWI splitting depends on coherence, so it is certainly not beside 
the

point for the coin toss.


It doesn't depend on coherence. Why would it matter if the state 
of the coin gets entangled with a zillion other environmental 
degrees of freedom? The dynamics according to unitary time 
evolution leads toa  superposition, no matter how many degrees of 
freedom are involved in the entanglement.


You are missing the point. Splitting according to the Schrödinger
equation does depend on coherence. The decoherence that entangles 
the

coin with a zillion other environmental degrees of freedom occurs
after the splitting. Given decoherence, the process is irreversible
FAPP, which means that there is no practical way, by design or 
chance,
that a decohered state can recohere. Sure, in the many worlds of 
MWI
the superposition, if it once existed, is still intact. But if no 
such
superposition ever existed, then it can't be created from 
non-coherent

interactions.

So Schrödinger's cat was once a coherent state of a cat in a box, 
and

the splitting occurs with the decay of a nucleus; decoherent
entanglement then leads to the splitting of worlds FAPP. But given 
an

arbitrary coin, it is already non-coherently entangled with many
environmental degrees of freedom, but there is no state that can 
lead
to {heads>+|tails>} in a unitary manner, so there is no state that 
can
then evolve into a splitting and decoherence into worlds 
distinguished

by either |heads> or |tails>. If you think that there is, write out
the schematic sequence of states evolving under the SE that leads 
to

this result.


So, let's examine this more closely. We start with a state that is a 
superposition of branches that each undergo classical evolution, so 
I'm not now appealing to the arguments in the paper by Albrecht. 
Then we end up with a state of the form:


 Sum over j [|heads(j)>|env(heads,j)> +

 Sum over j |tails(j)>|env(tails,j)>]

where j enumerates microstates of the coin and the state of the 
environment in each sector where the coin is heads or tails depends 
on the microstate of the coin. The coin states are not normalized, 
the norms are chosen to yield the correct probabilities.  E.g. if 
tails has zero probability then all the |tail(j)> are given a norm 
of zero. So, the above expression is completely general.


Now, I'm part of the environment, so we can write:

|env(X,j)> = Sum over k of |me(X,Y,j,k)>|env'(X,Y,j,k)>

where X is heads or tails, Y, denotes my macrostate that I'll define 
below, j denotes the microstate of the coin and k sums over 
microstates corresponding to macrostate Y.


This splitting of my state into a macrostate Y and microstate k is 
in principle arbitrary, we can choose this splitting such that Y 
keep tracts of what I'm aware of and then k takes into account all 
the other degrees of freedom in my body and brain that I'm not aware 
of. E.g. if I were a digital computer then Y would correspond to 
some bitstring defined by the computational state of the computer 
and we then sum over all the possible microstates that correspond to 
some fixed macrostate.


Then when I'm not yet aware of the result of the coin throw, we 
have:


|me(X,Y,j,k)> = |Awareness(Y)>|body(X,Y,j,k)>

where Y contains all the information that I am aware of, and that 
doesn't include the result if the coin throw nor information 
contained only at the microscopic scale. So, Awareness (Y) for a 
given Y defines who I am, where I am , and what I'm experiencing.


It should be clear that despite decoherence, |Awareness(Y)> will 
factor out of the global superposition as it doesn't depend on X, k 
and j.


So, given what I know, I cannot tell in which branch I am. Therefore 
unless all the branches corresponding to one outcome have zero norm, 
 I will have copies in both branches.


There is just no way that all the information describing what I'm 
aware of at some moment is going to constrain how someone else can 
throw a coin to such a degree that the outcome will be fixed.


So the end point of your analysis is that you can't see how it could
be possible that you are not right? The outcome is fixed -- and the
same -- in all worlds in which you participate!

But it is quite easy to see what is going on. Since it is all
independent of your awareness, we can factor that out of 

Re: Consistency of Postulates of QM

2017-12-14 Thread agrayson2000


On Thursday, December 14, 2017 at 1:41:37 AM UTC, Brent wrote:
>
>
>
> On 12/13/2017 5:24 PM, agrays...@gmail.com  wrote:
>
>
>
> On Wednesday, December 13, 2017 at 10:44:14 PM UTC, Brent wrote: 
>>
>>
>>
>> On 12/13/2017 2:20 PM, agrays...@gmail.com wrote:
>>
>>
>>
>> On Wednesday, December 13, 2017 at 9:15:36 PM UTC, Brent wrote: 
>>>
>>>
>>>
>>> On 12/13/2017 2:45 AM, agrays...@gmail.com wrote:
>>>
>>> * BUT for a nucleus of a radioactive element, the nucleus is never 
>>> Decayed and Undecayed SIMULTANEOUSLY.*
>>>
>>> Sure it is.  It's in a coherent superposition of those states until it 
>>> interacts with the environment.
>>>
>>> Brent
>>>
>>
>> * That's the conventional QM wisdom and the cause of the paradox of a cat 
>> Alive and Dead simultaneously. As I explained, the fallacy is rooted in an 
>> unjustified generalization of the double slit experiment where the 
>> probability waves do, in fact, exist simultaneously.  What waves do you 
>> claim are interacting for the radioactive nucleus to produce coherence? 
>> Tell me about them. I am from Missouri. AG*
>>
>>
>> You seem to think that coherence requires two different waves.  This is 
>> the wrong way to look at it.  In Young's slits experiment there is only one 
>> wave, which goes through both slits and interferes with itself.  
>>
>
>
> *That's exactly how I see it! Interference requires two waves which 
> interact with each other. *
>
>
> *NO.  This is false! * *There are not two waves.*  You can write it as 
> two parts, just as you can write a description of an ocean wave as the part 
> on your left and the part on your right.  But so long as they are coherent, 
> maintaining a fixed phase relation, they are one wave.
>
>
> *This is exactly what we see in Young's slits experiments. AG  *
>
>> And unstable nucleus has a probability amplitude that includes a 
>> "decayed" part and a "not decayed" part.  It's a tunneling problem.
>>
>
> *I don't doubt the existence of amplitudes. What I do doubt. and in fact 
> deny, is interference between two waves that don't exist simultaneously. *
>
>
> You keep referring to two waves. * There are not two waves.  *There's 
> only one wave which interferes with itself.  It is typically written as 
> |not-decayed> + |decayed>, but that's just a choice of basis.  It could as 
> well be written |unstable nucleus>.
>

*How can one wave interfere with itself? Using double slit model, the one 
wave you're referring to, must somehow split. How does that splitting occur 
in a nuclear decay? If no splitting, then the concept of interference makes 
no sense. Without interference, the cat is never simultaneously Alive and 
Dead. AG*
 

> Us

-- 
You received this message because you are subscribed to the Google Groups 
"Everything List" group.
To unsubscribe from this group and stop receiving emails from it, send an email 
to everything-list+unsubscr...@googlegroups.com.
To post to this group, send email to everything-list@googlegroups.com.
Visit this group at https://groups.google.com/group/everything-list.
For more options, visit https://groups.google.com/d/optout.


Re: Consistency of Postulates of QM

2017-12-13 Thread Brent Meeker



On 12/13/2017 7:38 PM, agrayson2...@gmail.com wrote:



On Thursday, December 14, 2017 at 2:39:12 AM UTC, John Clark wrote:


On Wed, Dec 13, 2017 at 9:10 PM,  wrote:

​
​>>​
Detected? I thought consciousness had nothing to do with
it. Who is doing the detection?​


​> ​
For the nth time, it can be an instrument.


​
​Fine, ​t
he instrument can be in 2 states, the photon activated state and
the photon not activated state, and those 2 states are different,
so the universe splits.  ​Detection implies consciousness or at
least knowledge, but all many worlds needs is a change, that's why
in many worlds a brick wall would work as well as a photographic
plate, the universe would still split at the slits and still merge
back when the photons hit the brick wall and no longer existed in
both universes.


For would-be physicists nothing is easier than splitting universes and 
making uncountable copies of themselves. AG




> In the case of destroying interference, the instrument is looking
-- that is sending photons to each of the slits -- which destroys
the interference.

If that is the explanation then why does the interference come
back if the which way information is erased *AFTER* the photon
passed the slits but before is hit the photographic plate?



And even /*after*/ the photon has "hit the photographic plate", 
arXiv:quant-ph/0106078 v1  And even space-like relative to the 
detection: arXiv:1206.6578v2


Brent

--
You received this message because you are subscribed to the Google Groups 
"Everything List" group.
To unsubscribe from this group and stop receiving emails from it, send an email 
to everything-list+unsubscr...@googlegroups.com.
To post to this group, send email to everything-list@googlegroups.com.
Visit this group at https://groups.google.com/group/everything-list.
For more options, visit https://groups.google.com/d/optout.


Re: Consistency of Postulates of QM

2017-12-13 Thread agrayson2000


On Thursday, December 14, 2017 at 2:39:12 AM UTC, John Clark wrote:
>
>
> On Wed, Dec 13, 2017 at 9:10 PM,  
> wrote:
>
> ​
>>> ​>>​
>>> Detected? I thought consciousness had nothing to do with it. Who is 
>>> doing the detection?​
>>>  
>>>
>>
>> ​> ​
>> For the nth time, it can be an instrument.
>>
>
> ​
> ​Fine, ​t
> he instrument can be in 2 states, the photon activated state and the 
> photon not activated state, and those 2 states are different, so the 
> universe splits.  ​Detection implies consciousness or at least knowledge, 
> but all many worlds needs is a change, that's why in many worlds a brick 
> wall would work as well as a photographic plate, the universe would still 
> split at the slits and still merge back when the photons hit the brick wall 
> and no longer existed in both universes.  
>

For would-be physicists nothing is easier than splitting universes and 
making uncountable copies of themselves. AG 

>
>
>  > In the case of destroying interference, the instrument is looking -- 
> that is sending photons to each of the slits -- which destroys the 
> interference. 
>
> If that is the explanation then why does the interference come back if the 
> which way information is erased *AFTER* the photon passed the slits but 
> before is hit the photographic plate?  
>

I don't know, but how does another world answer this? AG 

>  
>
>
>>> ​>​
 ​>>​
  Before I can comment about Wheeler's delayed choice experiment, I need 
 to know how the information is erased.

>>>
>>> ​>>​
>>> ​Try reading.​
>>>
>>> https://en.wikipedia.org/wiki/Wheeler%27s_delayed_choice_experiment
>>>
>>> And try knowing something before writing about something.
>>>
>>
>> ​>​
>> Replying to you isn't the only thing on my plate right now.
>>
>
> *​So you've got time to write stuff but no time to learn what you're 
> writing about.​ *
>

I notice you don't gave a damn about having a non falsifiable theory. So 
let's not split hairs about who has time for what. I will study the 
experiment and get back to you, but don't hold your breathe. AG 

>
>
> *​John K Clark​*
>
>
>>
>

-- 
You received this message because you are subscribed to the Google Groups 
"Everything List" group.
To unsubscribe from this group and stop receiving emails from it, send an email 
to everything-list+unsubscr...@googlegroups.com.
To post to this group, send email to everything-list@googlegroups.com.
Visit this group at https://groups.google.com/group/everything-list.
For more options, visit https://groups.google.com/d/optout.


Re: Consistency of Postulates of QM

2017-12-13 Thread John Clark
On Wed, Dec 13, 2017 at 9:10 PM,  wrote:

​
>> ​>>​
>> Detected? I thought consciousness had nothing to do with it. Who is doing
>> the detection?​
>>
>>
>
> ​> ​
> For the nth time, it can be an instrument.
>

​
​Fine, ​t
he instrument can be in 2 states, the photon activated state and the photon
not activated state, and those 2 states are different, so the universe
splits.  ​Detection implies consciousness or at least knowledge, but all
many worlds needs is a change, that's why in many worlds a brick wall would
work as well as a photographic plate, the universe would still split at the
slits and still merge back when the photons hit the brick wall and no
longer existed in both universes.

 > In the case of destroying interference, the instrument is looking --
that is sending photons to each of the slits -- which destroys the
interference.

If that is the explanation then why does the interference come back if the
which way information is erased *AFTER* the photon passed the slits but
before is hit the photographic plate?


>> ​>​
>>> ​>>​
>>>  Before I can comment about Wheeler's delayed choice experiment, I need
>>> to know how the information is erased.
>>>
>>
>> ​>>​
>> ​Try reading.​
>>
>> https://en.wikipedia.org/wiki/Wheeler%27s_delayed_choice_experiment
>>
>> And try knowing something before writing about something.
>>
>
> ​>​
> Replying to you isn't the only thing on my plate right now.
>

*​So you've got time to write stuff but no time to learn what you're
writing about.​ *


*​John K Clark​*


>

-- 
You received this message because you are subscribed to the Google Groups 
"Everything List" group.
To unsubscribe from this group and stop receiving emails from it, send an email 
to everything-list+unsubscr...@googlegroups.com.
To post to this group, send email to everything-list@googlegroups.com.
Visit this group at https://groups.google.com/group/everything-list.
For more options, visit https://groups.google.com/d/optout.


Re: Consistency of Postulates of QM

2017-12-13 Thread agrayson2000


On Thursday, December 14, 2017 at 1:51:39 AM UTC, John Clark wrote:
>
> On Wed, Dec 13, 2017 at 8:36 PM,  
> wrote:
>
>
>> ​>​
>> All I am saying is that electrons (and all particles) move as waves when 
>> undetected, and are localized in space when detected.
>>
>
> ​Detected? I thought consciousness had nothing to do with it. Who is doing 
> the detection?​
>  
>

For the nth time, it can be an instrument. In the case of destroying 
interference, the instrument is looking -- that is sending photons to each 
of the slits -- which destroys the interference.  No one knows how that 
happens, but we know it happens, as distinguished from your Many Worlds 
which remain undetectable. AG

>
>  
>
>> ​>​
>>  Before I can comment about Wheeler's delayed choice experiment, I need 
>> to know how the information is erased.
>>
>
>
> ​Try reading.​
>
> https://en.wikipedia.org/wiki/Wheeler%27s_delayed_choice_experiment
>
> And try knowing something before writing about something.
>

Replying to you isn't the only thing on my plate right now.  AG

>
> John K Clark
>
>
>
>>
>>>

-- 
You received this message because you are subscribed to the Google Groups 
"Everything List" group.
To unsubscribe from this group and stop receiving emails from it, send an email 
to everything-list+unsubscr...@googlegroups.com.
To post to this group, send email to everything-list@googlegroups.com.
Visit this group at https://groups.google.com/group/everything-list.
For more options, visit https://groups.google.com/d/optout.


Re: Consistency of Postulates of QM

2017-12-13 Thread agrayson2000


On Thursday, December 14, 2017 at 1:41:37 AM UTC, Brent wrote:
>
>
>
> On 12/13/2017 5:24 PM, agrays...@gmail.com  wrote:
>
>
>
> On Wednesday, December 13, 2017 at 10:44:14 PM UTC, Brent wrote: 
>>
>>
>>
>> On 12/13/2017 2:20 PM, agrays...@gmail.com wrote:
>>
>>
>>
>> On Wednesday, December 13, 2017 at 9:15:36 PM UTC, Brent wrote: 
>>>
>>>
>>>
>>> On 12/13/2017 2:45 AM, agrays...@gmail.com wrote:
>>>
>>> * BUT for a nucleus of a radioactive element, the nucleus is never 
>>> Decayed and Undecayed SIMULTANEOUSLY.*
>>>
>>> Sure it is.  It's in a coherent superposition of those states until it 
>>> interacts with the environment.
>>>
>>> Brent
>>>
>>
>> * That's the conventional QM wisdom and the cause of the paradox of a cat 
>> Alive and Dead simultaneously. As I explained, the fallacy is rooted in an 
>> unjustified generalization of the double slit experiment where the 
>> probability waves do, in fact, exist simultaneously.  What waves do you 
>> claim are interacting for the radioactive nucleus to produce coherence? 
>> Tell me about them. I am from Missouri. AG*
>>
>>
>> You seem to think that coherence requires two different waves.  This is 
>> the wrong way to look at it.  In Young's slits experiment there is only one 
>> wave, which goes through both slits and interferes with itself.  
>>
>
>
> *That's exactly how I see it! Interference requires two waves which 
> interact with each other. *
>
>
> *NO.  This is false! * *There are not two waves.*  You can write it as 
> two parts, just as you can write a description of an ocean wave as the part 
> on your left and the part on your right.  But so long as they are coherent, 
> maintaining a fixed phase relation, they are one wave.
>



*You're splitting hairs, engaging in sophism. For the single wave going 
through both slits, Feynman calculates the norm squared of | A + B |, where 
A and B denote the waveS going through left and right slits respectively. 
Both are obviously identical, with the result of coherent interference. 
>From this analysis we get the interpretation that the the system is 
simultaneously in all states of a superposition. AGNoteworthy is that fact 
that if you reference "coherence" on Wiki, the description always invokes 
multiple waves of the same frequency. If you want to assert coherence 
without multiple waves, and NOT using the double slit result, you have some 
heavy lifting to do. AG*

>
>
> *This is exactly what we see in Young's slits experiments. AG  *
>
>> And unstable nucleus has a probability amplitude that includes a 
>> "decayed" part and a "not decayed" part.  It's a tunneling problem.
>>
>
> *I don't doubt the existence of amplitudes. What I do doubt. and in fact 
> deny, is interference between two waves that don't exist simultaneously. *
>
>
> You keep referring to two waves. * There are not two waves.  *There's 
> only one wave which interferes with itself.  It is typically written as 
> |not-decayed> + |decayed>, but that's just a choice of basis.  It could as 
> well be written |unstable nucleus>.
>

*OK, unstable nucleus. Makes no difference to what I am arguing; namely, 
that coherence requires more than one wave, simultaneously, which is what 
double slit slows, even though the experiment obviously starts out with one 
wave. AG* 

>
> *If there's no interference, then the cat cannot be Alive and Dead 
> simultaneously. Tunneling can exist, but still, no simultaneous 
> interacting, interfering waves. Is there any advantage to believing in two 
> waves which don't exist simultaneous, can interfere with each other? AG*
>
>
> You are confused.
>

You're the one with a cat which Alive and Dead simultaneously for the very 
short time until decoherence occurs. So it is arguable who is really 
confused. AG 

>
> Brent
>

-- 
You received this message because you are subscribed to the Google Groups 
"Everything List" group.
To unsubscribe from this group and stop receiving emails from it, send an email 
to everything-list+unsubscr...@googlegroups.com.
To post to this group, send email to everything-list@googlegroups.com.
Visit this group at https://groups.google.com/group/everything-list.
For more options, visit https://groups.google.com/d/optout.


Re: Consistency of Postulates of QM

2017-12-13 Thread John Clark
On Wed, Dec 13, 2017 at 8:36 PM,  wrote:


> ​>​
> All I am saying is that electrons (and all particles) move as waves when
> undetected, and are localized in space when detected.
>

​Detected? I thought consciousness had nothing to do with it. Who is doing
the detection?​




> ​>​
>  Before I can comment about Wheeler's delayed choice experiment, I need to
> know how the information is erased.
>


​Try reading.​

https://en.wikipedia.org/wiki/Wheeler%27s_delayed_choice_experiment

And try knowing something before writing about something.

John K Clark



>
>>

-- 
You received this message because you are subscribed to the Google Groups 
"Everything List" group.
To unsubscribe from this group and stop receiving emails from it, send an email 
to everything-list+unsubscr...@googlegroups.com.
To post to this group, send email to everything-list@googlegroups.com.
Visit this group at https://groups.google.com/group/everything-list.
For more options, visit https://groups.google.com/d/optout.


Re: Consistency of Postulates of QM

2017-12-13 Thread Brent Meeker



On 12/13/2017 5:24 PM, agrayson2...@gmail.com wrote:



On Wednesday, December 13, 2017 at 10:44:14 PM UTC, Brent wrote:



On 12/13/2017 2:20 PM, agrays...@gmail.com  wrote:



On Wednesday, December 13, 2017 at 9:15:36 PM UTC, Brent wrote:



On 12/13/2017 2:45 AM, agrays...@gmail.com wrote:

*BUT for a nucleus of a radioactive element, the nucleus is
never Decayed and Undecayed SIMULTANEOUSLY.*

Sure it is.  It's in a coherent superposition of those states
until it interacts with the environment.

Brent

*
That's the conventional QM wisdom and the cause of the paradox of
a cat Alive and Dead simultaneously. As I explained, the fallacy
is rooted in an unjustified generalization of the double slit
experiment where the probability waves do, in fact, exist
simultaneously.  What waves do you claim are interacting for the
radioactive nucleus to produce coherence? Tell me about them. I
am from Missouri. AG*


You seem to think that coherence requires two different waves. 
This is the wrong way to look at it.  In Young's slits experiment
there is only one wave, which goes through both slits and
interferes with itself.


*That's exactly how I see it! Interference requires two waves which 
interact with each other.

*


/*NO.  This is false! */ /*There are not two waves.*/ You can write it 
as two parts, just as you can write a description of an ocean wave as 
the part on your left and the part on your right.  But so long as they 
are coherent, maintaining a fixed phase relation, they are one wave.



*This is exactly what we see in Young's slits experiments. AG
*

And unstable nucleus has a probability amplitude that includes a
"decayed" part and a "not decayed" part.  It's a tunneling problem.


*I don't doubt the existence of amplitudes. What I do doubt. and in 
fact deny, is interference between two waves that don't exist 
simultaneously. *


You keep referring to two waves. /*There are not two waves. */There's 
only one wave which interferes with itself.  It is typically written as 
|not-decayed> + |decayed>, but that's just a choice of basis.  It could 
as well be written |unstable nucleus>.


*If there's no interference, then the cat cannot be Alive and Dead 
simultaneously. Tunneling can exist, but still, no simultaneous 
interacting, interfering waves. Is there any advantage to believing in 
two waves which don't exist simultaneous, can interfere with each 
other? AG*


You are confused.

Brent

--
You received this message because you are subscribed to the Google Groups 
"Everything List" group.
To unsubscribe from this group and stop receiving emails from it, send an email 
to everything-list+unsubscr...@googlegroups.com.
To post to this group, send email to everything-list@googlegroups.com.
Visit this group at https://groups.google.com/group/everything-list.
For more options, visit https://groups.google.com/d/optout.


Re: Consistency of Postulates of QM

2017-12-13 Thread agrayson2000


On Thursday, December 14, 2017 at 1:04:32 AM UTC, John Clark wrote:
>
> On Wed, Dec 13, 2017 at 5:25 PM,  
> wrote:
>
> ​> ​
>> In the double slit experiment, the photon, or electron, or whatever, 
>> travels as a wave and goes through both slits in THIS world.
>>
>
> *​Not if you look at one of the slits it doesn't! If you put a device that 
> can detect electrons then you know the electron went though just one slit 
> and you know which one and no interference pattern builds up on the 
> photographic plate. However if you record the information on which slit the 
> electron went through but erase it before it hits the photographic plate​ 
> then you don't know what the electron did at the slits and 
> the interference pattern comes back. This is called Wheeler's delayed 
> choice experiment​ and I think its absolutely absurd. The most absurd thing 
> about it is its true. *
>

All I am saying is that electrons (and all particles) move as waves when 
undetected, and are localized in space when detected. That's what 
experiments seem to show. No need for Many Worlds. Before I can comment 
about Wheeler's delayed choice experiment, I need to know how the 
information is erased. Offhand, I don't see why the results of Wheeler's 
experiment, however strange, falsifies my claim. AG 

>
> https://en.wikipedia.org/wiki/Wheeler%27s_delayed_choice_experiment
>
>  
>
>> ​> ​
>> No need for multiple worlds to explain anything.
>>
>
> *​Oh yeah? Well then explain to me how that works. I'm all ears. ​*
>

Tell me how uncountable worlds are created? The stuff of dreams by 
religious fanatics, aka would-be physicists? AG

>
> *​John K Clark ​ *
>
>

-- 
You received this message because you are subscribed to the Google Groups 
"Everything List" group.
To unsubscribe from this group and stop receiving emails from it, send an email 
to everything-list+unsubscr...@googlegroups.com.
To post to this group, send email to everything-list@googlegroups.com.
Visit this group at https://groups.google.com/group/everything-list.
For more options, visit https://groups.google.com/d/optout.


Re: Consistency of Postulates of QM

2017-12-13 Thread Bruce Kellett

On 14/12/2017 11:52 am, smitra wrote:

On 13-12-2017 22:55, Bruce Kellett wrote:

On 14/12/2017 8:23 am, smitra wrote:

On 12-12-2017 23:13, Bruce Kellett wrote:

On 13/12/2017 2:12 am, smitra wrote:

On 12-12-2017 12:33, Bruce Kellett wrote:

On 12/12/2017 9:46 pm, smitra wrote:


Yes, it's only an estimation but it yields a good order of 
magnitude estimate for the center of mass. What the calculation 
shows is that quantum superpositions do exists at the 
macroscopic level and these can then be amplified by chaotic 
dynamics. Of course,  it then becomes incoherent, but in the MWI 
that's besides the point.


MWI splitting depends on coherence, so it is certainly not beside 
the

point for the coin toss.


It doesn't depend on coherence. Why would it matter if the state 
of the coin gets entangled with a zillion other environmental 
degrees of freedom? The dynamics according to unitary time 
evolution leads toa  superposition, no matter how many degrees of 
freedom are involved in the entanglement.


You are missing the point. Splitting according to the Schrödinger
equation does depend on coherence. The decoherence that entangles the
coin with a zillion other environmental degrees of freedom occurs
after the splitting. Given decoherence, the process is irreversible
FAPP, which means that there is no practical way, by design or chance,
that a decohered state can recohere. Sure, in the many worlds of MWI
the superposition, if it once existed, is still intact. But if no such
superposition ever existed, then it can't be created from non-coherent
interactions.

So Schrödinger's cat was once a coherent state of a cat in a box, and
the splitting occurs with the decay of a nucleus; decoherent
entanglement then leads to the splitting of worlds FAPP. But given an
arbitrary coin, it is already non-coherently entangled with many
environmental degrees of freedom, but there is no state that can lead
to {heads>+|tails>} in a unitary manner, so there is no state that can
then evolve into a splitting and decoherence into worlds distinguished
by either |heads> or |tails>. If you think that there is, write out
the schematic sequence of states evolving under the SE that leads to
this result.


So, let's examine this more closely. We start with a state that is a 
superposition of branches that each undergo classical evolution, so 
I'm not now appealing to the arguments in the paper by Albrecht. 
Then we end up with a state of the form:


 Sum over j [|heads(j)>|env(heads,j)> +

 Sum over j |tails(j)>|env(tails,j)>]

where j enumerates microstates of the coin and the state of the 
environment in each sector where the coin is heads or tails depends 
on the microstate of the coin. The coin states are not normalized, 
the norms are chosen to yield the correct probabilities.  E.g. if 
tails has zero probability then all the |tail(j)> are given a norm 
of zero. So, the above expression is completely general.


Now, I'm part of the environment, so we can write:

|env(X,j)> = Sum over k of |me(X,Y,j,k)>|env'(X,Y,j,k)>

where X is heads or tails, Y, denotes my macrostate that I'll define 
below, j denotes the microstate of the coin and k sums over 
microstates corresponding to macrostate Y.


This splitting of my state into a macrostate Y and microstate k is 
in principle arbitrary, we can choose this splitting such that Y 
keep tracts of what I'm aware of and then k takes into account all 
the other degrees of freedom in my body and brain that I'm not aware 
of. E.g. if I were a digital computer then Y would correspond to 
some bitstring defined by the computational state of the computer 
and we then sum over all the possible microstates that correspond to 
some fixed macrostate.


Then when I'm not yet aware of the result of the coin throw, we have:

|me(X,Y,j,k)> = |Awareness(Y)>|body(X,Y,j,k)>

where Y contains all the information that I am aware of, and that 
doesn't include the result if the coin throw nor information 
contained only at the microscopic scale. So, Awareness (Y) for a 
given Y defines who I am, where I am , and what I'm experiencing.


It should be clear that despite decoherence, |Awareness(Y)> will 
factor out of the global superposition as it doesn't depend on X, k 
and j.


So, given what I know, I cannot tell in which branch I am. Therefore 
unless all the branches corresponding to one outcome have zero 
norm,  I will have copies in both branches.


There is just no way that all the information describing what I'm 
aware of at some moment is going to constrain how someone else can 
throw a coin to such a degree that the outcome will be fixed.


So the end point of your analysis is that you can't see how it could
be possible that you are not right? The outcome is fixed -- and the
same -- in all worlds in which you participate!

But it is quite easy to see what is going on. Since it is all
independent of your awareness, we can factor that out of the
summations. And you still do not get a coherent 

Re: Consistency of Postulates of QM

2017-12-13 Thread agrayson2000


On Wednesday, December 13, 2017 at 10:44:14 PM UTC, Brent wrote:
>
>
>
> On 12/13/2017 2:20 PM, agrays...@gmail.com  wrote:
>
>
>
> On Wednesday, December 13, 2017 at 9:15:36 PM UTC, Brent wrote: 
>>
>>
>>
>> On 12/13/2017 2:45 AM, agrays...@gmail.com wrote:
>>
>> * BUT for a nucleus of a radioactive element, the nucleus is never 
>> Decayed and Undecayed SIMULTANEOUSLY.*
>>
>> Sure it is.  It's in a coherent superposition of those states until it 
>> interacts with the environment.
>>
>> Brent
>>
>
> * That's the conventional QM wisdom and the cause of the paradox of a cat 
> Alive and Dead simultaneously. As I explained, the fallacy is rooted in an 
> unjustified generalization of the double slit experiment where the 
> probability waves do, in fact, exist simultaneously.  What waves do you 
> claim are interacting for the radioactive nucleus to produce coherence? 
> Tell me about them. I am from Missouri. AG*
>
>
> You seem to think that coherence requires two different waves.  This is 
> the wrong way to look at it.  In Young's slits experiment there is only one 
> wave, which goes through both slits and interferes with itself.  
>


*That's exactly how I see it! Interference requires two waves which 
interact with each other. This is exactly what we see in Young's slits 
experiments. AG *

> And unstable nucleus has a probability amplitude that includes a "decayed" 
> part and a "not decayed" part.  It's a tunneling problem.
>

*I don't doubt the existence of amplitudes. What I do doubt. and in fact 
deny, is interference between two waves that don't exist simultaneously. If 
there's no interference, then the cat cannot be Alive and Dead 
simultaneously. Tunneling can exist, but still, no simultaneous 
interacting, interfering waves. Is there any advantage to believing in two 
waves which don't exist simultaneous, can interfere with each other? AG* 

 

>
> Brent
>

-- 
You received this message because you are subscribed to the Google Groups 
"Everything List" group.
To unsubscribe from this group and stop receiving emails from it, send an email 
to everything-list+unsubscr...@googlegroups.com.
To post to this group, send email to everything-list@googlegroups.com.
Visit this group at https://groups.google.com/group/everything-list.
For more options, visit https://groups.google.com/d/optout.


Re: Consistency of Postulates of QM

2017-12-13 Thread John Clark
On Wed, Dec 13, 2017 at 5:25 PM,  wrote:

​> ​
> In the double slit experiment, the photon, or electron, or whatever,
> travels as a wave and goes through both slits in THIS world.
>

*​Not if you look at one of the slits it doesn't! If you put a device that
can detect electrons then you know the electron went though just one slit
and you know which one and no interference pattern builds up on the
photographic plate. However if you record the information on which slit the
electron went through but erase it before it hits the photographic plate​
then you don't know what the electron did at the slits and
the interference pattern comes back. This is called Wheeler's delayed
choice experiment​ and I think its absolutely absurd. The most absurd thing
about it is its true. ​*

https://en.wikipedia.org/wiki/Wheeler%27s_delayed_choice_experiment



> ​> ​
> No need for multiple worlds to explain anything.
>

*​Oh yeah? Well then explain to me how that works. I'm all ears. ​*

*​John K Clark ​ *

-- 
You received this message because you are subscribed to the Google Groups 
"Everything List" group.
To unsubscribe from this group and stop receiving emails from it, send an email 
to everything-list+unsubscr...@googlegroups.com.
To post to this group, send email to everything-list@googlegroups.com.
Visit this group at https://groups.google.com/group/everything-list.
For more options, visit https://groups.google.com/d/optout.


Re: Consistency of Postulates of QM

2017-12-13 Thread smitra

On 13-12-2017 22:55, Bruce Kellett wrote:

On 14/12/2017 8:23 am, smitra wrote:

On 12-12-2017 23:13, Bruce Kellett wrote:

On 13/12/2017 2:12 am, smitra wrote:

On 12-12-2017 12:33, Bruce Kellett wrote:

On 12/12/2017 9:46 pm, smitra wrote:


Yes, it's only an estimation but it yields a good order of 
magnitude estimate for the center of mass.  What the calculation 
shows is that quantum superpositions do exists at the macroscopic 
level and these can then be amplified by chaotic dynamics. Of 
course,  it then becomes incoherent, but in the MWI that's besides 
the point.


MWI splitting depends on coherence, so it is certainly not beside 
the

point for the coin toss.


It doesn't depend on coherence. Why would it matter if the state of 
the coin gets entangled with a zillion other environmental degrees 
of freedom? The dynamics according to unitary time evolution leads 
toa  superposition, no matter how many degrees of freedom are 
involved in the entanglement.


You are missing the point. Splitting according to the Schrödinger
equation does depend on coherence. The decoherence that entangles the
coin with a zillion other environmental degrees of freedom occurs
after the splitting. Given decoherence, the process is irreversible
FAPP, which means that there is no practical way, by design or 
chance,

that a decohered state can recohere. Sure, in the many worlds of MWI
the superposition, if it once existed, is still intact. But if no 
such
superposition ever existed, then it can't be created from 
non-coherent

interactions.

So Schrödinger's cat was once a coherent state of a cat in a box, and
the splitting occurs with the decay of a nucleus; decoherent
entanglement then leads to the splitting of worlds FAPP. But given an
arbitrary coin, it is already non-coherently entangled with many
environmental degrees of freedom, but there is no state that can lead
to {heads>+|tails>} in a unitary manner, so there is no state that 
can
then evolve into a splitting and decoherence into worlds 
distinguished

by either |heads> or |tails>. If you think that there is, write out
the schematic sequence of states evolving under the SE that leads to
this result.


So, let's examine this more closely. We start with a state that is a 
superposition of branches that each undergo classical evolution, so 
I'm not now appealing to the arguments in the paper by Albrecht. Then 
we end up with a state of the form:


 Sum over j [|heads(j)>|env(heads,j)> +

 Sum over j |tails(j)>|env(tails,j)>]

where j enumerates microstates of the coin and the state of the 
environment in each sector where the coin is heads or tails depends on 
the microstate of the coin. The coin states are not normalized, the 
norms are chosen to yield the correct probabilities.  E.g. if tails 
has zero probability then all the |tail(j)> are given a norm of zero. 
So, the above expression is completely general.


Now, I'm part of the environment, so we can write:

|env(X,j)> = Sum over k of |me(X,Y,j,k)>|env'(X,Y,j,k)>

where X is heads or tails, Y, denotes my macrostate that I'll define 
below, j denotes the microstate of the coin and k sums over 
microstates corresponding to macrostate Y.


This splitting of my state into a macrostate Y and microstate k is in 
principle arbitrary, we can choose this splitting such that Y keep 
tracts of what I'm aware of and then k takes into account all the 
other degrees of freedom in my body and brain that I'm not aware of. 
E.g. if I were a digital computer then Y would correspond to some 
bitstring defined by the computational state of the computer and we 
then sum over all the possible microstates that correspond to some 
fixed macrostate.


Then when I'm not yet aware of the result of the coin throw, we have:

|me(X,Y,j,k)> = |Awareness(Y)>|body(X,Y,j,k)>

where Y contains all the information that I am aware of, and that 
doesn't include the result if the coin throw nor information contained 
only at the microscopic scale. So, Awareness (Y) for a given Y defines 
who I am, where I am , and what I'm experiencing.


It should be clear that despite decoherence, |Awareness(Y)> will 
factor out of the global superposition as it doesn't depend on X, k 
and j.


So, given what I know, I cannot tell in which branch I am. Therefore 
unless all the branches corresponding to one outcome have zero norm,  
I will have copies in both branches.


There is just no way that all the information describing what I'm 
aware of at some moment is going to constrain how someone else can 
throw a coin to such a degree that the outcome will be fixed.


So the end point of your analysis is that you can't see how it could
be possible that you are not right? The outcome is fixed -- and the
same -- in all worlds in which you participate!

But it is quite easy to see what is going on. Since it is all
independent of your awareness, we can factor that out of the
summations. And you still do not get a coherent superposition from a
decoherent mixture. 

Re: Consistency of Postulates of QM

2017-12-13 Thread Brent Meeker



On 12/13/2017 2:20 PM, agrayson2...@gmail.com wrote:



On Wednesday, December 13, 2017 at 9:15:36 PM UTC, Brent wrote:



On 12/13/2017 2:45 AM, agrays...@gmail.com  wrote:

*BUT for a nucleus of a radioactive element, the nucleus is never
Decayed and Undecayed SIMULTANEOUSLY.*

Sure it is.  It's in a coherent superposition of those states
until it interacts with the environment.

Brent

*
That's the conventional QM wisdom and the cause of the paradox of a 
cat Alive and Dead simultaneously. As I explained, the fallacy is 
rooted in an unjustified generalization of the double slit experiment 
where the probability waves do, in fact, exist simultaneously.  What 
waves do you claim are interacting for the radioactive nucleus to 
produce coherence? Tell me about them. I am from Missouri. AG*


You seem to think that coherence requires two different waves.  This is 
the wrong way to look at it.  In Young's slits experiment there is only 
one wave, which goes through both slits and interferes with itself.  And 
unstable nucleus has a probability amplitude that includes a "decayed" 
part and a "not decayed" part.  It's a tunneling problem.


Brent

--
You received this message because you are subscribed to the Google Groups 
"Everything List" group.
To unsubscribe from this group and stop receiving emails from it, send an email 
to everything-list+unsubscr...@googlegroups.com.
To post to this group, send email to everything-list@googlegroups.com.
Visit this group at https://groups.google.com/group/everything-list.
For more options, visit https://groups.google.com/d/optout.


Re: Consistency of Postulates of QM

2017-12-13 Thread agrayson2000


On Wednesday, December 13, 2017 at 10:25:12 PM UTC, agrays...@gmail.com 
wrote:
>
>
>
> On Wednesday, December 13, 2017 at 6:10:43 PM UTC, John Clark wrote:
>>
>>
>>
>> On Tue, Dec 12, 2017 at 9:57 PM, Brent Meeker  
>> wrote:
>>
>> ​ 
 ​>> ​
 I've been asking all along exactly what is it that collapses the wave 
 function. If its not an observer and its not a measurement and its not 
 consciousness then what is it?  
>>>
>>>
>>> ​> ​
>>> It is interaction with something with lots degrees of freedom
>>>
>>
>> ​In other words a change, a difference.​
>>  
>> ​That works for me and it works for manny worlds too.​ It would not 
>> violate the laws of physics if a photon went through either slit, so the 
>> universe splits, in one universe it goes through the left slit in the other 
>> it goes through the right. Normally the universes stay split because they 
>> stay different and the 2 photons continue on into infinite space, but if 
>> you put up a photographic plate after passing the slits it hits the plate 
>> and so no longer exists in either universe, so there is no longer a 
>> difference between universes so they merge back together. That act of 
>> splitting and then coming back together makes the interference pattern.
>>
>
> In the double slit experiment, the photon, or electron, or whatever, 
> travels as a wave and goes through both slits in THIS world. No need for 
> multiple worlds to explain anything. Even wonder why, in a slit experiment, 
> if the source is close to, and centered between the slits, anything goes 
> through to the screen? AG
>

*Correction: EVER wonder why ... AG* 

>
>
>>  If you want to strip "measurement" and "observation" back to their 
>> absolute minimal essential you arrive at a simple change, and that would be 
>> fine but then in effect you've just turned Copenhagen into Many Worlds.  
>>
>> ​> ​
>>> I pointed out that is inconsistent with SWE to say that anything 
>>> possible actually happens.  "Possible" needs to be qualified.  For example 
>>> the SWE in a Young's slit experiment tells you that the probability of a 
>>> particle striking the detector is zero at some places.  It's logically 
>>> possible for a particle to strike there, but not nomologically possible. 
>>>  
>>
>>
>> ​I'm talking about physics not mathematics. ​
>>  
>> ​As far as I know ​
>> ​there would be no logical inconsistency if Newtonian physics (but not 
>> Aristotelian  ​physics) was the only physics there is, but that's not the 
>> way things are so we need Relativity and Quantum Mechanics.
>>
>>  John K Clark
>>
>>
>>
>>

-- 
You received this message because you are subscribed to the Google Groups 
"Everything List" group.
To unsubscribe from this group and stop receiving emails from it, send an email 
to everything-list+unsubscr...@googlegroups.com.
To post to this group, send email to everything-list@googlegroups.com.
Visit this group at https://groups.google.com/group/everything-list.
For more options, visit https://groups.google.com/d/optout.


Re: Consistency of Postulates of QM

2017-12-13 Thread agrayson2000


On Wednesday, December 13, 2017 at 6:10:43 PM UTC, John Clark wrote:
>
>
>
> On Tue, Dec 12, 2017 at 9:57 PM, Brent Meeker  > wrote:
>
> ​ 
>>> ​>> ​
>>> I've been asking all along exactly what is it that collapses the wave 
>>> function. If its not an observer and its not a measurement and its not 
>>> consciousness then what is it?  
>>
>>
>> ​> ​
>> It is interaction with something with lots degrees of freedom
>>
>
> ​In other words a change, a difference.​
>  
> ​That works for me and it works for manny worlds too.​ It would not 
> violate the laws of physics if a photon went through either slit, so the 
> universe splits, in one universe it goes through the left slit in the other 
> it goes through the right. Normally the universes stay split because they 
> stay different and the 2 photons continue on into infinite space, but if 
> you put up a photographic plate after passing the slits it hits the plate 
> and so no longer exists in either universe, so there is no longer a 
> difference between universes so they merge back together. That act of 
> splitting and then coming back together makes the interference pattern.
>

In the double slit experiment, the photon, or electron, or whatever, 
travels as a wave and goes through both slits in THIS world. No need for 
multiple worlds to explain anything. Even wonder why, in a slit experiment, 
if the source is close to, and centered between the slits, anything goes 
through to the screen? AG 

>
>  If you want to strip "measurement" and "observation" back to their 
> absolute minimal essential you arrive at a simple change, and that would be 
> fine but then in effect you've just turned Copenhagen into Many Worlds.  
>
> ​> ​
>> I pointed out that is inconsistent with SWE to say that anything possible 
>> actually happens.  "Possible" needs to be qualified.  For example the SWE 
>> in a Young's slit experiment tells you that the probability of a particle 
>> striking the detector is zero at some places.  It's logically possible for 
>> a particle to strike there, but not nomologically possible.  
>
>
> ​I'm talking about physics not mathematics. ​
>  
> ​As far as I know ​
> ​there would be no logical inconsistency if Newtonian physics (but not 
> Aristotelian  ​physics) was the only physics there is, but that's not the 
> way things are so we need Relativity and Quantum Mechanics.
>
>  John K Clark
>
>
>
>

-- 
You received this message because you are subscribed to the Google Groups 
"Everything List" group.
To unsubscribe from this group and stop receiving emails from it, send an email 
to everything-list+unsubscr...@googlegroups.com.
To post to this group, send email to everything-list@googlegroups.com.
Visit this group at https://groups.google.com/group/everything-list.
For more options, visit https://groups.google.com/d/optout.


Re: Consistency of Postulates of QM

2017-12-13 Thread agrayson2000


On Wednesday, December 13, 2017 at 9:15:36 PM UTC, Brent wrote:
>
>
>
> On 12/13/2017 2:45 AM, agrays...@gmail.com  wrote:
>
> * BUT for a nucleus of a radioactive element, the nucleus is never Decayed 
> and Undecayed SIMULTANEOUSLY.*
>
> Sure it is.  It's in a coherent superposition of those states until it 
> interacts with the environment.
>
> Brent
>

*That's the conventional QM wisdom and the cause of the paradox of a cat 
Alive and Dead simultaneously. As I explained, the fallacy is rooted in an 
unjustified generalization of the double slit experiment where the 
probability waves do, in fact, exist simultaneously.  What waves do you 
claim are interacting for the radioactive nucleus to produce coherence? 
Tell me about them. I am from Missouri. AG*

-- 
You received this message because you are subscribed to the Google Groups 
"Everything List" group.
To unsubscribe from this group and stop receiving emails from it, send an email 
to everything-list+unsubscr...@googlegroups.com.
To post to this group, send email to everything-list@googlegroups.com.
Visit this group at https://groups.google.com/group/everything-list.
For more options, visit https://groups.google.com/d/optout.


Re: Consistency of Postulates of QM

2017-12-13 Thread Bruce Kellett

On 14/12/2017 8:23 am, smitra wrote:

On 12-12-2017 23:13, Bruce Kellett wrote:

On 13/12/2017 2:12 am, smitra wrote:

On 12-12-2017 12:33, Bruce Kellett wrote:

On 12/12/2017 9:46 pm, smitra wrote:


Yes, it's only an estimation but it yields a good order of 
magnitude estimate for the center of mass.  What the calculation 
shows is that quantum superpositions do exists at the macroscopic 
level and these can then be amplified by chaotic dynamics. Of 
course,  it then becomes incoherent, but in the MWI that's besides 
the point.


MWI splitting depends on coherence, so it is certainly not beside the
point for the coin toss.


It doesn't depend on coherence. Why would it matter if the state of 
the coin gets entangled with a zillion other environmental degrees 
of freedom? The dynamics according to unitary time evolution leads 
toa  superposition, no matter how many degrees of freedom are 
involved in the entanglement.


You are missing the point. Splitting according to the Schrödinger
equation does depend on coherence. The decoherence that entangles the
coin with a zillion other environmental degrees of freedom occurs
after the splitting. Given decoherence, the process is irreversible
FAPP, which means that there is no practical way, by design or chance,
that a decohered state can recohere. Sure, in the many worlds of MWI
the superposition, if it once existed, is still intact. But if no such
superposition ever existed, then it can't be created from non-coherent
interactions.

So Schrödinger's cat was once a coherent state of a cat in a box, and
the splitting occurs with the decay of a nucleus; decoherent
entanglement then leads to the splitting of worlds FAPP. But given an
arbitrary coin, it is already non-coherently entangled with many
environmental degrees of freedom, but there is no state that can lead
to {heads>+|tails>} in a unitary manner, so there is no state that can
then evolve into a splitting and decoherence into worlds distinguished
by either |heads> or |tails>. If you think that there is, write out
the schematic sequence of states evolving under the SE that leads to
this result.


So, let's examine this more closely. We start with a state that is a 
superposition of branches that each undergo classical evolution, so 
I'm not now appealing to the arguments in the paper by Albrecht. Then 
we end up with a state of the form:


 Sum over j [|heads(j)>|env(heads,j)> +

 Sum over j |tails(j)>|env(tails,j)>]

where j enumerates microstates of the coin and the state of the 
environment in each sector where the coin is heads or tails depends on 
the microstate of the coin. The coin states are not normalized, the 
norms are chosen to yield the correct probabilities.  E.g. if tails 
has zero probability then all the |tail(j)> are given a norm of zero. 
So, the above expression is completely general.


Now, I'm part of the environment, so we can write:

|env(X,j)> = Sum over k of |me(X,Y,j,k)>|env'(X,Y,j,k)>

where X is heads or tails, Y, denotes my macrostate that I'll define 
below, j denotes the microstate of the coin and k sums over 
microstates corresponding to macrostate Y.


This splitting of my state into a macrostate Y and microstate k is in 
principle arbitrary, we can choose this splitting such that Y keep 
tracts of what I'm aware of and then k takes into account all the 
other degrees of freedom in my body and brain that I'm not aware of. 
E.g. if I were a digital computer then Y would correspond to some 
bitstring defined by the computational state of the computer and we 
then sum over all the possible microstates that correspond to some 
fixed macrostate.


Then when I'm not yet aware of the result of the coin throw, we have:

|me(X,Y,j,k)> = |Awareness(Y)>|body(X,Y,j,k)>

where Y contains all the information that I am aware of, and that 
doesn't include the result if the coin throw nor information contained 
only at the microscopic scale. So, Awareness (Y) for a given Y defines 
who I am, where I am , and what I'm experiencing.


It should be clear that despite decoherence, |Awareness(Y)> will 
factor out of the global superposition as it doesn't depend on X, k 
and j.


So, given what I know, I cannot tell in which branch I am. Therefore 
unless all the branches corresponding to one outcome have zero norm,  
I will have copies in both branches.


There is just no way that all the information describing what I'm 
aware of at some moment is going to constrain how someone else can 
throw a coin to such a degree that the outcome will be fixed.


So the end point of your analysis is that you can't see how it could be 
possible that you are not right? The outcome is fixed -- and the same -- 
in all worlds in which you participate!


But it is quite easy to see what is going on. Since it is all 
independent of your awareness, we can factor that out of the summations. 
And you still do not get a coherent superposition from a decoherent 
mixture. What goes on in the world is independent of 

Re: Consistency of Postulates of QM

2017-12-13 Thread smitra

On 12-12-2017 23:13, Bruce Kellett wrote:

On 13/12/2017 2:12 am, smitra wrote:

On 12-12-2017 12:33, Bruce Kellett wrote:

On 12/12/2017 9:46 pm, smitra wrote:


Yes, it's only an estimation but it yields a good order of magnitude 
estimate for the center of mass.  What the calculation shows is that 
quantum superpositions do exists at the macroscopic level and these 
can then be amplified by chaotic dynamics. Of course,  it then 
becomes incoherent, but in the MWI that's besides the point.


MWI splitting depends on coherence, so it is certainly not beside the
point for the coin toss.


It doesn't depend on coherence. Why would it matter if the state of 
the coin gets entangled with a zillion other environmental degrees of 
freedom? The dynamics according to unitary time evolution leads toa  
superposition, no matter how many degrees of freedom are involved in 
the entanglement.


You are missing the point. Splitting according to the Schrödinger
equation does depend on coherence. The decoherence that entangles the
coin with a zillion other environmental degrees of freedom occurs
after the splitting. Given decoherence, the process is irreversible
FAPP, which means that there is no practical way, by design or chance,
that a decohered state can recohere. Sure, in the many worlds of MWI
the superposition, if it once existed, is still intact. But if no such
superposition ever existed, then it can't be created from non-coherent
interactions.

So Schrödinger's cat was once a coherent state of a cat in a box, and
the splitting occurs with the decay of a nucleus; decoherent
entanglement then leads to the splitting of worlds FAPP. But given an
arbitrary coin, it is already non-coherently entangled with many
environmental degrees of freedom, but there is no state that can lead
to {heads>+|tails>} in a unitary manner, so there is no state that can
then evolve into a splitting and decoherence into worlds distinguished
by either |heads> or |tails>. If you think that there is, write out
the schematic sequence of states evolving under the SE that leads to
this result.


So, let's examine this more closely. We start with a state that is a 
superposition of branches that each undergo classical evolution, so I'm 
not now appealing to the arguments in the paper by Albrecht. Then we end 
up with a state of the form:


 Sum over j [|heads(j)>|env(heads,j)> +

 Sum over j |tails(j)>|env(tails,j)>]

where j enumerates microstates of the coin and the state of the 
environment in each sector where the coin is heads or tails depends on 
the microstate of the coin. The coin states are not normalized, the 
norms are chosen to yield the correct probabilities.  E.g. if tails has 
zero probability then all the |tail(j)> are given a norm of zero. So, 
the above expression is completely general.


Now, I'm part of the environment, so we can write:

|env(X,j)> = Sum over k of |me(X,Y,j,k)>|env'(X,Y,j,k)>

where X is heads or tails, Y, denotes my macrostate that I'll define 
below, j denotes the microstate of the coin and k sums over microstates 
corresponding to macrostate Y.


This splitting of my state into a macrostate Y and microstate k is in 
principle arbitrary, we can choose this splitting such that Y keep 
tracts of what I'm aware of and then k takes into account all the other 
degrees of freedom in my body and brain that I'm not aware of. E.g. if I 
were a digital computer then Y would correspond to some bitstring 
defined by the computational state of the computer and we then sum over 
all the possible microstates that correspond to some fixed macrostate.


Then when I'm not yet aware of the result of the coin throw, we have:

|me(X,Y,j,k)> = |Awareness(Y)>|body(X,Y,j,k)>

where Y contains all the information that I am aware of, and that 
doesn't include the result if the coin throw nor information contained 
only at the microscopic scale. So, Awareness (Y) for a given Y defines 
who I am, where I am , and what I'm experiencing.


It should be clear that despite decoherence, |Awareness(Y)> will factor 
out of the global superposition as it doesn't depend on X, k and j.


So, given what I know, I cannot tell in which branch I am. Therefore 
unless all the branches corresponding to one outcome have zero norm,  I 
will have copies in both branches.


There is just no way that all the information describing what I'm aware 
of at some moment is going to constrain how someone else can throw a 
coin to such a degree that the outcome will be fixed.


Saibal

--
You received this message because you are subscribed to the Google Groups 
"Everything List" group.
To unsubscribe from this group and stop receiving emails from it, send an email 
to everything-list+unsubscr...@googlegroups.com.
To post to this group, send email to everything-list@googlegroups.com.
Visit this group at https://groups.google.com/group/everything-list.
For more options, visit https://groups.google.com/d/optout.


Re: Consistency of Postulates of QM

2017-12-13 Thread Brent Meeker



On 12/13/2017 3:15 AM, Bruno Marchal wrote:
The coin does not start in a state of the kind  {|heads> + |tails>}, 
but it starts with a state of having mulitiple positions and multiple 
momenta, spreaded in the multiverse according to the Heisenberg 
Uncertainty. The tiny difference in the position can lead to different 
bouncing in the box, and so, by shaking it a long time enough, I don’t 
see why we could avoid a superposition of head and tails eventually.


Decoherence is waaay faster than the time for the coin to cross the box 
from one impact to another, so I don't see how a superposition could 
develop from shaking the box.


Brent

--
You received this message because you are subscribed to the Google Groups 
"Everything List" group.
To unsubscribe from this group and stop receiving emails from it, send an email 
to everything-list+unsubscr...@googlegroups.com.
To post to this group, send email to everything-list@googlegroups.com.
Visit this group at https://groups.google.com/group/everything-list.
For more options, visit https://groups.google.com/d/optout.


Re: Consistency of Postulates of QM

2017-12-13 Thread Brent Meeker



On 12/13/2017 2:45 AM, agrayson2...@gmail.com wrote:
*BUT for a nucleus of a radioactive element, the nucleus is never 
Decayed and Undecayed SIMULTANEOUSLY.*


Sure it is.  It's in a coherent superposition of those states until it 
interacts with the environment.


Brent

--
You received this message because you are subscribed to the Google Groups 
"Everything List" group.
To unsubscribe from this group and stop receiving emails from it, send an email 
to everything-list+unsubscr...@googlegroups.com.
To post to this group, send email to everything-list@googlegroups.com.
Visit this group at https://groups.google.com/group/everything-list.
For more options, visit https://groups.google.com/d/optout.


Re: Consistency of Postulates of QM

2017-12-13 Thread John Clark
On Tue, Dec 12, 2017 at 9:57 PM, Brent Meeker  wrote:

​
>> ​>> ​
>> I've been asking all along exactly what is it that collapses the wave
>> function. If its not an observer and its not a measurement and its not
>> consciousness then what is it?
>
>
> ​> ​
> It is interaction with something with lots degrees of freedom
>

​In other words a change, a difference.​

​That works for me and it works for manny worlds too.​ It would not violate
the laws of physics if a photon went through either slit, so the universe
splits, in one universe it goes through the left slit in the other it goes
through the right. Normally the universes stay split because they stay
different and the 2 photons continue on into infinite space, but if you put
up a photographic plate after passing the slits it hits the plate and so no
longer exists in either universe, so there is no longer a difference
between universes so they merge back together. That act of splitting and
then coming back together makes the interference pattern.

 If you want to strip "measurement" and "observation" back to their
absolute minimal essential you arrive at a simple change, and that would be
fine but then in effect you've just turned Copenhagen into Many Worlds.

​> ​
> I pointed out that is inconsistent with SWE to say that anything possible
> actually happens.  "Possible" needs to be qualified.  For example the SWE
> in a Young's slit experiment tells you that the probability of a particle
> striking the detector is zero at some places.  It's logically possible for
> a particle to strike there, but not nomologically possible.


​I'm talking about physics not mathematics. ​

​As far as I know ​
​there would be no logical inconsistency if Newtonian physics (but not
Aristotelian  ​physics) was the only physics there is, but that's not the
way things are so we need Relativity and Quantum Mechanics.

 John K Clark

-- 
You received this message because you are subscribed to the Google Groups 
"Everything List" group.
To unsubscribe from this group and stop receiving emails from it, send an email 
to everything-list+unsubscr...@googlegroups.com.
To post to this group, send email to everything-list@googlegroups.com.
Visit this group at https://groups.google.com/group/everything-list.
For more options, visit https://groups.google.com/d/optout.


Re: Consistency of Postulates of QM

2017-12-13 Thread Bruno Marchal


On 13 Dec 2017, at 04:04, Brent Meeker wrote:




On 12/12/2017 6:21 PM, John Clark wrote:
​> ​ He seemed to claim it negated the main claim of MWI, that  
everything that CAN happen, DOES happen.


​I don't see how.​


I pointed out that is inconsistent with SWE to say that anything  
possible actually happens.  "Possible" needs to be qualified.  For  
example the SWE in a Young's slit experiment tells you that the  
probability of a particle striking the detector is zero at some  
places.  It's logically possible for a particle to strike there, but  
not nomologically possible.  Similary if you measure the value of a  
variable for a state you can only get values that are eigenvalues of  
the operator.  Others are logically possible, but not nomologically.


Provable and consistent (possible) depends on the theory which is  
postulated. If QM is derivable from arithmetic, as it should if  
Computationalism is true, then there is a sense to say that what you  
call nomological impossibility is also logically impossible, or at  
least arithmetico-logically impossible. Here you use the assumption of  
physicalism implicitly. I single that out, because that implicit  
assumption is a frequent source of misunderstanding of the  
consequences of computationalism.


Bruno




Brent

--
You received this message because you are subscribed to the Google  
Groups "Everything List" group.
To unsubscribe from this group and stop receiving emails from it,  
send an email to everything-list+unsubscr...@googlegroups.com.

To post to this group, send email to everything-list@googlegroups.com.
Visit this group at https://groups.google.com/group/everything-list.
For more options, visit https://groups.google.com/d/optout.


http://iridia.ulb.ac.be/~marchal/



--
You received this message because you are subscribed to the Google Groups 
"Everything List" group.
To unsubscribe from this group and stop receiving emails from it, send an email 
to everything-list+unsubscr...@googlegroups.com.
To post to this group, send email to everything-list@googlegroups.com.
Visit this group at https://groups.google.com/group/everything-list.
For more options, visit https://groups.google.com/d/optout.


Re: Consistency of Postulates of QM

2017-12-13 Thread Bruno Marchal


On 13 Dec 2017, at 04:27, Bruce Kellett wrote:


On 13/12/2017 11:41 am, agrayson2...@gmail.com wrote:

On Tuesday, December 12, 2017 at 10:52:12 PM UTC, Bruce wrote:
On 13/12/2017 9:45 am, agrays...@gmail.com wrote:

On Tuesday, December 12, 2017 at 10:14:01 PM UTC, Bruce wrote:

So Schrödinger's cat was once a coherent state of a cat in a box,  
and

the splitting occurs with the decay of a nucleus;

Unlike the double slit experiment, which can only be understood  
with interference and the model that the electron wave, say, goes  
through both slits, the nuclear decay is a BINARY event, very  
roughly like a coin toss where there is no interference.  
Generalizing interference to every quantum state is where  
Schrodinger went wrong. The cat, which shares or inherits the wf  
from the radioactive decay, is never in both Alive and Dead states  
simultaneously. AG


In the double slit, the paths through the two slits remain  
coherent, so they can interfere when they come together. In the  
case of nuclear decay, the coherence is lost immediately, so the  
nucleus does not interfere with the decay products.


Bruce

So you agree or disagree with my conclusion; namely, the cat is  
never in a superposition of states? That is, no situation where cat  
is Alive and Dead simultaneously. I think you disagree and think  
the nuclear state is superposed with interference existing. AG


The superposition of |live + dead> or |live - dead> does not exist  
in any single world since such states are not stable against  
decoherence. But if you take the pedantic view of the many worlds of  
MWI, the superposition of live and dead cats, together with  
everything entangled with them, exists for ever in the  
multiverse.What good that does anyone, I fail to understand.


Searching the good and searching the truth might be different thing,  
unless you add the quite platonic axiom that truth is good, which I  
hope, but that is hardly usable in a reasoning.


Bruno





Bruce

--
You received this message because you are subscribed to the Google  
Groups "Everything List" group.
To unsubscribe from this group and stop receiving emails from it,  
send an email to everything-list+unsubscr...@googlegroups.com.

To post to this group, send email to everything-list@googlegroups.com.
Visit this group at https://groups.google.com/group/everything-list.
For more options, visit https://groups.google.com/d/optout.


http://iridia.ulb.ac.be/~marchal/



--
You received this message because you are subscribed to the Google Groups 
"Everything List" group.
To unsubscribe from this group and stop receiving emails from it, send an email 
to everything-list+unsubscr...@googlegroups.com.
To post to this group, send email to everything-list@googlegroups.com.
Visit this group at https://groups.google.com/group/everything-list.
For more options, visit https://groups.google.com/d/optout.


Re: Consistency of Postulates of QM

2017-12-13 Thread Bruno Marchal


On 12 Dec 2017, at 13:17, Bruce Kellett wrote:


On 12/12/2017 10:47 pm, Bruno Marchal wrote:

On 12 Dec 2017, at 11:12, Bruce Kellett wrote:

On 12/12/2017 8:20 pm, Bruno Marchal wrote:

On 11 Dec 2017, at 23:15, Bruce Kellett wrote:
On 12/12/2017 1:12 am, Bruno Marchal wrote:

On 10 Dec 2017, at 23:38, Bruce Kellett wrote:

On 11/12/2017 2:19 am, Bruno Marchal wrote:

On 09 Dec 2017, at 00:03, Bruce Kellett wrote:

On 9/12/2017 4:21 am, Bruno Marchal wrote:
Similarly, a shroedinger car, once alive + dead, will never  
become a pure alive, or dead cat. It will only seems so for  
anyone looking at the cat, in the {alive, dead} base/ 
apparatus. Superposition never disappear, and a coin moree  
or less with a precise position, is always a superposition  
of a coin with more or less precise momenta. The relation  
is given by the Fourier transforms, which gives the  
relative accessible states/worlds.


I pointed out that for a macroscopic object such as a coin,  
the uncertainty relations give uncertainties in positions  
and/or momentum far below any level of possible detection.


Of possible practical detection. That is good FAPP, but  
irrelevant for theoretical consideration.


This is a purely rhetorical objection, Bruno. And when you  
trot this out, as you do regularly, I know that your purpose  
is to obfuscate, and hide the fact that you have no rational  
argument to offer.


You confuse physics and metaphysics. The difference is not  
rhetorical, but fundamental in this thread.


Rubbish. The central point of contention on this thread is  
whether a coin toss can be regarded as a classical event, with  
probabilities given by ignorance of the initial conditions, or  
as a quantum event with probabilities given by purely quantum  
uncertainties.


This is a straightforward question of physics, and has nothing  
to do with metaphysics. As usual, you introduce the term  
'metaphysics' merely to obfuscate, because you have no  
intelligent response to the clear physics of the situation.



I was using "metaphysics" in opposition to your instrumentalist  
FAPP distraction. Metaphysics and/or theology involves the  
question of the existence of the universe(s), parallel or not.


Then you make ad hominem insult in place of argument. I guess  
your goal is not to enlighten me.


You are the one making the personal attacks. You can do  
metaphysics in the privacy of your own home, but here in public, I  
talk about verifiable physics.


My attack was on your use of the "FAPP" to change the subject of  
the discussion which was on metaphysics at the start: the existence  
of the (obviously undetectable) other term of some macroscopic  
superposition. (It was not ad hominem at all).


Neither were any of my comments personal attacks. The original  
discussion was about whether or not the world split on the coin  
toss. Very little to do with metaphysics, and your continued  
introduction of irrelevant considerations has been nothing more than  
a distraction from the main discussion, adding nothing.


Yes, the question was about some world, which we assume to exist. But  
that is a metaphysical assumption in all cases, even in classical  
physics, and even more about orthogonal classical state that we cannot  
“rec-cohere” in practice. The metaphysical assumption that there is a  
“universe” (in the Aristotelian primary sense) i.e. (weak) materialism  
is even an assumption which can be shown to be logically incompatible  
with Mechanism (in cognitive science or theology/metaphysics).


I guess that you might be confusing evidence for physics with evidence  
for some metaphysics.


Bruno







Bruce

--
You received this message because you are subscribed to the Google  
Groups "Everything List" group.
To unsubscribe from this group and stop receiving emails from it,  
send an email to everything-list+unsubscr...@googlegroups.com.

To post to this group, send email to everything-list@googlegroups.com.
Visit this group at https://groups.google.com/group/everything-list.
For more options, visit https://groups.google.com/d/optout.


http://iridia.ulb.ac.be/~marchal/



--
You received this message because you are subscribed to the Google Groups 
"Everything List" group.
To unsubscribe from this group and stop receiving emails from it, send an email 
to everything-list+unsubscr...@googlegroups.com.
To post to this group, send email to everything-list@googlegroups.com.
Visit this group at https://groups.google.com/group/everything-list.
For more options, visit https://groups.google.com/d/optout.


Re: Consistency of Postulates of QM

2017-12-13 Thread Bruno Marchal


On 12 Dec 2017, at 13:13, Bruce Kellett wrote:


On 12/12/2017 10:55 pm, Bruno Marchal wrote:

On 12 Dec 2017, at 11:14, Bruce Kellett wrote:

On 12/12/2017 8:26 pm, Bruno Marchal wrote:

On 12 Dec 2017, at 02:02, Bruce Kellett wrote:

On 12/12/2017 11:44 am, smitra wrote:

On 11-12-2017 23:15, Bruce Kellett wrote:

On 12/12/2017 1:12 am, Bruno Marchal wrote:

On 10 Dec 2017, at 23:38, Bruce Kellett wrote:

On 11/12/2017 2:19 am, Bruno Marchal wrote:

On 09 Dec 2017, at 00:03, Bruce Kellett wrote:

On 9/12/2017 4:21 am, Bruno Marchal wrote:

Similarly, a shroedinger car, once alive + dead, will never
become a pure alive, or dead cat. It will only seems so for
anyone looking at the cat, in the {alive, dead} base/ 
apparatus.
Superposition never disappear, and a coin moree or less  
with a
precise position, is always a superposition of a coin  
with more
or less precise momenta. The relation is given by the  
Fourier
transforms, which gives the relative accessible states/ 
worlds.


I pointed out that for a macroscopic object such as a  
coin, the

uncertainty relations give uncertainties in positions and/or
momentum far below any level of possible detection.


Of possible practical detection. That is good FAPP, but  
irrelevant

for theoretical consideration.


This is a purely rhetorical objection, Bruno. And when you  
trot

this out, as you do regularly, I know that your purpose is to
obfuscate, and hide the fact that you have no rational  
argument to

offer.


You confuse physics and metaphysics. The difference is not
rhetorical, but fundamental in this thread.


Rubbish. The central point of contention on this thread is  
whether a
coin toss can be regarded as a classical event, with  
probabilities
given by ignorance of the initial conditions, or as a quantum  
event

with probabilities given by purely quantum uncertainties.

This is a straightforward question of physics, and has nothing  
to do

with metaphysics. As usual, you introduce the term 'metaphysics'
merely to obfuscate, because you have no intelligent response  
to the

clear physics of the situation.



That the probabilities are given by classical physics does not  
imply

that there is no branching due to the coin toss.


It does, because there is no superposition of head/tails -- no  
possibility of interference between heads and tails.


You are begging the question. The point was that without  
collapse, the shaking of the dice or coin can make the  
superposition of the positions (inherent in the Heisenberg  
uncertainty) can add up to make the coin behaving sufficiently  
differently to obtain a superposition of the head+tail or 1+2+... 
+6 superposition.


No, shaking the coin cannot make non-coherent uncertainties add up  
to anything. The physics is against you here, Bruno. There is no  
superposition, and no splitting of worlds on the toss. If you  
think different, prove it.



Just to be sure, do you agree that without collapse, the  
schroedinger cat remains in the state alive+dead, even after  
observation, and we see it alive OR dead, just by the first-person  
mechanist indeterminacy (or something akin to it) ?


The superposition exists in the original quantum state --  
essentially the radioactive nucleus which is a quantum system in a  
superposition of decayed/not-decayed. Because of the experimental  
set up, this superposition is amplified so that it becomes entangled  
with the environment, including the cat and us. So:


  |nucleus>|box>|cat>|observer>|rest of the world> --> (by unitary  
evolution)
  {|decayed>|poison spilt>|cat dead>|see dead cat>|rest of world  
confirms dead cat> +
  |not decayed>|poison bottle intact>|cat alive>|see live cat>|rest  
of world confirms live cat>}


And the decoherence of the quantum phases into the |rest of the  
world> environmental states diagonalizes the density matrix FAPP.


Yes. FAPP.




If you insist, the superposition is intact in the bird view, but  
such a superposition can never recohere, and there are NO  
consequences of the existence of other branches, practical or  
otherwise. Maintaining their existence might satisfy your  
existential angst, but it has nothing to do with physics or  
experience.



The existence of those branches is more frightening than angst  
appeasing. But we cannot make them disappear in the frame of the  
quantum mechanics without collapse. It is just a question of being  
consistent with what we are assuming.






Also, how could the quantum uncertainties becoming non-coherent?  
Decoherence, without collapse is something relative to the  
environment when its get itself entangled with the superposition of  
the object observed. The splitting of worlds is not due to the  
toss, but to the fact that the position of the coin diffuse, which  
means get different in the multiverse. Instead of shaking the box,  
waiting long enough would work as well.


The coin case is different in an essential way -- it does not start  
from a single 

Re: Consistency of Postulates of QM

2017-12-13 Thread agrayson2000


On Wednesday, December 13, 2017 at 7:41:00 AM UTC, agrays...@gmail.com 
wrote:
>
>
>
> On Wednesday, December 13, 2017 at 5:55:59 AM UTC, Bruce wrote:
>>
>> On 13/12/2017 11:41 am, agrays...@gmail.com wrote:
>>
>> On Tuesday, December 12, 2017 at 10:52:12 PM UTC, Bruce wrote: 
>>>
>>> On 13/12/2017 9:45 am, agrays...@gmail.com wrote:
>>>
>>> On Tuesday, December 12, 2017 at 10:14:01 PM UTC, Bruce wrote: 


 So Schrödinger's cat was once a coherent state of a cat in a box, and 
 the splitting occurs with the decay of a nucleus; 
>>>
>>>
>>> *Unlike the double slit experiment, which can only be understood with 
>>> interference and the model that the electron wave, say, goes through both 
>>> slits, the nuclear decay is a BINARY event, very roughly like a coin toss 
>>> where there is no interference. Generalizing interference to every quantum 
>>> state is where Schrodinger went wrong. The cat, which shares or inherits 
>>> the wf from the radioactive decay, is never in both Alive and Dead states 
>>> simultaneously. AG*
>>>
>>>
>>> In the double slit, the paths through the two slits remain coherent, so 
>>> they can interfere when they come together. In the case of nuclear decay, 
>>> the coherence is lost immediately, so the nucleus does not interfere with 
>>> the decay products.
>>>
>>> Bruce
>>>
>>
>> *So you agree or disagree with my conclusion; namely, the cat is never in 
>> a superposition of states? That is, no situation where cat is Alive and 
>> Dead simultaneously. I think you disagree and think the nuclear state is 
>> superposed with interference existing. AG*
>>
>>
>> The superposition of |live + dead> or |live - dead> does not exist in any 
>> single world since such states are not stable against decoherence. But if 
>> you take the pedantic view of the many worlds of MWI, the superposition of 
>> live and dead cats, together with everything entangled with them, exists 
>> for ever in the multiverse. What good that does anyone, I fail to 
>> understand.
>>
>> Bruce
>>
>
> *I am not referring to the MWI. I am referring to whether in Schrodinger's 
> cat experiment, the wf of the radioactive source, ( |decayed> + |undecayed> 
> ) , is a superposition of states without interference between its 
> components. If that's the case, perhaps what you would call an "incoherent 
> superposition", then the cat which shares or inherits this wf in 
> Schrodinger's set up, is never in a state of Alive and Dead simultaneously. 
> AG*
>
> *You can look at it this way; in the double slit experiment, the 
> probability wave of the electron, say, passes through both slits 
> SIMULTANEOUSLY. As a result, it is conceptually feasible to model 
> interference between the waves, giving rise to the correct distribution on 
> the screen. This is the seminal experiment in QM and gives rise to the 
> general interpretation that a system in a superposition can be in multiple 
> states simultaneously. BUT for a nucleus of a radioactive element, the 
> nucleus is never Decayed and Undecayed SIMULTANEOUSLY. Thus, it makes no 
> sense to believe there can be interference between the states. Absent 
> interference means the cat tied to the radioactive source can never be in 
> the Alive and Dead states SIMULTANEOUSLY. AG*
>
 

-- 
You received this message because you are subscribed to the Google Groups 
"Everything List" group.
To unsubscribe from this group and stop receiving emails from it, send an email 
to everything-list+unsubscr...@googlegroups.com.
To post to this group, send email to everything-list@googlegroups.com.
Visit this group at https://groups.google.com/group/everything-list.
For more options, visit https://groups.google.com/d/optout.


Re: Consistency of Postulates of QM

2017-12-12 Thread agrayson2000


On Wednesday, December 13, 2017 at 5:55:59 AM UTC, Bruce wrote:
>
> On 13/12/2017 11:41 am, agrays...@gmail.com  wrote:
>
> On Tuesday, December 12, 2017 at 10:52:12 PM UTC, Bruce wrote: 
>>
>> On 13/12/2017 9:45 am, agrays...@gmail.com wrote:
>>
>> On Tuesday, December 12, 2017 at 10:14:01 PM UTC, Bruce wrote: 
>>>
>>>
>>> So Schrödinger's cat was once a coherent state of a cat in a box, and 
>>> the splitting occurs with the decay of a nucleus; 
>>
>>
>> *Unlike the double slit experiment, which can only be understood with 
>> interference and the model that the electron wave, say, goes through both 
>> slits, the nuclear decay is a BINARY event, very roughly like a coin toss 
>> where there is no interference. Generalizing interference to every quantum 
>> state is where Schrodinger went wrong. The cat, which shares or inherits 
>> the wf from the radioactive decay, is never in both Alive and Dead states 
>> simultaneously. AG*
>>
>>
>> In the double slit, the paths through the two slits remain coherent, so 
>> they can interfere when they come together. In the case of nuclear decay, 
>> the coherence is lost immediately, so the nucleus does not interfere with 
>> the decay products.
>>
>> Bruce
>>
>
> *So you agree or disagree with my conclusion; namely, the cat is never in 
> a superposition of states? That is, no situation where cat is Alive and 
> Dead simultaneously. I think you disagree and think the nuclear state is 
> superposed with interference existing. AG*
>
>
> The superposition of |live + dead> or |live - dead> does not exist in any 
> single world since such states are not stable against decoherence. But if 
> you take the pedantic view of the many worlds of MWI, the superposition of 
> live and dead cats, together with everything entangled with them, exists 
> for ever in the multiverse. What good that does anyone, I fail to 
> understand.
>
> Bruce
>

*I am not referring to the MWI. I am referring to whether in Schrodinger's 
cat experiment, the wf of the radioactive source, ( |decayed> + |undecayed> 
) , is a superposition of states without interference between its 
components. If that's the case, perhaps what you would call an "incoherent 
superposition", then the cat which shares or inherits this wf in 
Schrodinger's set up, is never in a state of Alive and Dead simultaneously. 
AG*

-- 
You received this message because you are subscribed to the Google Groups 
"Everything List" group.
To unsubscribe from this group and stop receiving emails from it, send an email 
to everything-list+unsubscr...@googlegroups.com.
To post to this group, send email to everything-list@googlegroups.com.
Visit this group at https://groups.google.com/group/everything-list.
For more options, visit https://groups.google.com/d/optout.


Re: Consistency of Postulates of QM

2017-12-12 Thread Bruce Kellett

On 13/12/2017 11:41 am, agrayson2...@gmail.com wrote:

On Tuesday, December 12, 2017 at 10:52:12 PM UTC, Bruce wrote:

On 13/12/2017 9:45 am, agrays...@gmail.com  wrote:

On Tuesday, December 12, 2017 at 10:14:01 PM UTC, Bruce wrote:


So Schrödinger's cat was once a coherent state of a cat in a
box, and
the splitting occurs with the decay of a nucleus; 



*Unlike the double slit experiment, which can only be understood
with interference and the model that the electron wave, say, goes
through both slits, the nuclear decay is a BINARY event, very
roughly like a coin toss where there is no interference.
Generalizing interference to every quantum state is where
Schrodinger went wrong. The cat, which shares or inherits the wf
from the radioactive decay, is never in both Alive and Dead
states simultaneously. AG*


In the double slit, the paths through the two slits remain
coherent, so they can interfere when they come together. In the
case of nuclear decay, the coherence is lost immediately, so the
nucleus does not interfere with the decay products.

Bruce


*So you agree or disagree with my conclusion; namely, the cat is never 
in a superposition of states? That is, no situation where cat is Alive 
and Dead simultaneously. I think you disagree and think the nuclear 
state is superposed with interference existing. AG*


The superposition of |live + dead> or |live - dead> does not exist in 
any single world since such states are not stable against decoherence. 
But if you take the pedantic view of the many worlds of MWI, the 
superposition of live and dead cats, together with everything entangled 
with them, exists for ever in the multiverse. What good that does 
anyone, I fail to understand.


Bruce

--
You received this message because you are subscribed to the Google Groups 
"Everything List" group.
To unsubscribe from this group and stop receiving emails from it, send an email 
to everything-list+unsubscr...@googlegroups.com.
To post to this group, send email to everything-list@googlegroups.com.
Visit this group at https://groups.google.com/group/everything-list.
For more options, visit https://groups.google.com/d/optout.


Re: Consistency of Postulates of QM

2017-12-12 Thread Brent Meeker



On 12/12/2017 6:21 PM, John Clark wrote:


​> ​
He seemed to claim it negated the main claim of MWI, that
everything that CAN happen, DOES happen.


​I don't see how.​


I pointed out that is inconsistent with SWE to say that anything 
possible actually happens.  "Possible" needs to be qualified.  For 
example the SWE in a Young's slit experiment tells you that the 
probability of a particle striking the detector is zero at some places.  
It's logically possible for a particle to strike there, but not 
nomologically possible.  Similary if you measure the value of a variable 
for a state you can only get values that are eigenvalues of the 
operator.  Others are logically possible, but not nomologically.


Brent

--
You received this message because you are subscribed to the Google Groups 
"Everything List" group.
To unsubscribe from this group and stop receiving emails from it, send an email 
to everything-list+unsubscr...@googlegroups.com.
To post to this group, send email to everything-list@googlegroups.com.
Visit this group at https://groups.google.com/group/everything-list.
For more options, visit https://groups.google.com/d/optout.


Re: Consistency of Postulates of QM

2017-12-12 Thread Brent Meeker



On 12/12/2017 6:21 PM, John Clark wrote:


​>​
I've been saying all along that a conscious observer is not needed
to create or destroy the interference


​ I know you have. And I've been asking all along exactly what is it 
that collapses the wave function. If its not an observer and its not a 
measurement and its not consciousness then what is it?


It is interaction with something with lots degrees of freedom (like an 
instrument or the environment) so that there are many instances of the 
result recorded.  A measurement is one such event, but it doesn't have 
to be a measurement; compare the Bucky Ball Young's slits experiment.  
The interference is washed out when the interaction makes the welcher 
weg available even if it is inaccessible FAPP.


Brent

--
You received this message because you are subscribed to the Google Groups 
"Everything List" group.
To unsubscribe from this group and stop receiving emails from it, send an email 
to everything-list+unsubscr...@googlegroups.com.
To post to this group, send email to everything-list@googlegroups.com.
Visit this group at https://groups.google.com/group/everything-list.
For more options, visit https://groups.google.com/d/optout.


Re: Consistency of Postulates of QM

2017-12-12 Thread agrayson2000


On Wednesday, December 13, 2017 at 2:21:24 AM UTC, John Clark wrote:
>
> On Mon, Dec 11, 2017 at 3:45 PM,  
> wrote:
>
> ​>
> ​>>​
> ​
> The fundamental unproven assumption, and IMO the core fallacy of the MWI, 
> is the belief that what CAN occur, necessarily MUST will occur.
>
>
> ​>> ​
> The
> ​ ​
> fundamental
> ​ ​
> assumption of the MWI is that the
> ​ ​
> Schrodinger 
> ​Wave
>  
> ​Equation
>  means what it says and says what it means. The ​
> ​
> fundamental
> ​ ​
> assumption
> ​ of Copenhagen is that ​
> Schrodinger
> ​ forgot to put a "except" and a "however" into his equation.​
>
>
> ​> ​
> Solutions of the SWE give the probabilities of getting possible 
> measurement outcomes
>
>
> ​It's not the SWE itself that gives the probabilities, 
>

I know. That's why I wrote "solutions" of the SWE. AG
 

> you've got to 
> square of the absolute value of the wave
> ​ 
> function
> ​ ​
> ​to find the probability ​
> of 
> ​finding ​
> a particle at 
> ​that​
>  point
> ​. 
>

You mean Born's rule? Never heard of it. Wasn't covered in my graduate 
courses in QM. AG
 

> I'm not splitting hairs this is important because 
> the SWE contains
> ​ 
> imaginary numbers (square root of -1) so 2 very different wave functions 
> can yield the exact same probability at a point when you square it. So even 
> if you know the probability you can't know the unique wave function that 
> produced it because there is no such unique function.
>

Maybe at discrete points, and even if not, why is this important? AG 

>
> ​> ​
> prior to the measurement. If you want to give the equation a life after 
> measurement
> ​ [...]
>
>
> ​If you want to say the equation has no life after measurement then you're 
> going to have to explain exactly what a measurement is and what term in the 
> SWE it interacts with causing it to self destruct.
>

The measurement process, whatever its details are, is the same in MWI as in 
Copenhagen regardless of your denials. I can speculate what happens to the 
SWE after measurement, but more important I don't see why MWI implies MW, 
and you have NOT made that case. AG

-- 
You received this message because you are subscribed to the Google Groups 
"Everything List" group.
To unsubscribe from this group and stop receiving emails from it, send an email 
to everything-list+unsubscr...@googlegroups.com.
To post to this group, send email to everything-list@googlegroups.com.
Visit this group at https://groups.google.com/group/everything-list.
For more options, visit https://groups.google.com/d/optout.


Re: Consistency of Postulates of QM

2017-12-12 Thread John Clark
On Mon, Dec 11, 2017 at 3:45 PM,  wrote:

​>
>>> ​>>​
>>> ​
>>> The fundamental unproven assumption, and IMO the core fallacy of the
>>> MWI, is the belief that what CAN occur, necessarily MUST will occur.
>>>
>>
>> ​>> ​
>> The
>> ​ ​
>> fundamental
>> ​ ​
>> assumption of the MWI is that the
>> ​ ​
>> Schrodinger
>> ​Wave
>>
>> ​Equation
>>  means what it says and says what it means. The ​
>> ​
>> fundamental
>> ​ ​
>> assumption
>> ​ of Copenhagen is that ​
>> Schrodinger
>> ​ forgot to put a "except" and a "however" into his equation.​
>>
>
> ​> ​
> Solutions of the SWE give the probabilities of getting possible
> measurement outcomes
>

​It's not the SWE itself that gives the probabilities, you've got to
square of the absolute value of the wave
​
function
​ ​
​to find the probability ​
of
​finding ​
a particle at
​that​
 point
​. I'm not splitting hairs this is important because
the SWE contains
​
imaginary numbers (square root of -1) so 2 very different wave functions
can yield the exact same probability at a point when you square it. So even
if you know the probability you can't know the unique wave function that
produced it because there is no such unique function.


​> ​
> prior to the measurement. If you want to give the equation a life after
> measurement
> ​ [...]
>

​If you want to say the equation has no life after measurement then you're
going to have to explain exactly what a measurement is and what term in the
SWE it interacts with causing it to self destruct.  ​



> ​> ​
> Do you know about Gleason's result which Brent mentioned.
>

​
Yes I mentioned it a few days ago.
​
Gleason's theorem
​ says
that in 3 spatial dimensions only the square of
​the absolute value of ​
Schrodinger's wave (the Born rule), and not the cube or anything else, can
yield a probability without inconsistencies
​.​

​S​
o the real question isn't why is the Born rule a square but why is it a
probability at all
​? ​
​ ​And
Gleason's theorem
​ is pure mathematics, Many Worlds give it a physical interpretation, the
best one I know of.




> ​> ​
> He seemed to claim it negated the main claim of MWI, that everything that
> CAN happen, DOES happen.
>

​I don't see how.​



​> ​
> The idea of general covariance as a principle for understanding the
> natural world is not in the least absurd. You seem to adopt a pov which
> reminds me of religious zealots, who defend poorly founded ideas by appeals
> to ignorance of God's behavior.
>

Both Bell's inequality and
​
the
​
Leggett–Garg inequality
​ are violated, that is not a theory that is a experimental fact, so
quantum mechanics, or any successful theory that comes after it, is going
to have to incorporate that fact, and there is no way to do that without
being weird;  or to be more precise, without being grossly non-intuitive,
radically different from everyday experience, and at odds with common sense
(although not at odds with logic). ​

​
>> ​>> ​
>> Show me how to measure something without anybody doing any measuring and
>> show me the new term you added to the Schrodinger Equation that causes it
>> to collapse when a measurement is taken.
>>
>
> ​> ​
> I've been saying all along that a conscious observer is not needed to
> create or destroy the interference
>

​I know you have. And I've been asking all along exactly what is it that
collapses the wave function. If its not an observer and its not a
measurement and its not consciousness then what is it?  ​



> ​> ​
> did MWI derive Born's rule, or did it simply argue for its plausibility?
>

​I
wish I could say MWI derived it but that would be going too far, but it did
a much better job at
​
arguing​
for its plausibility
​ than​ Copenhagen.

 John K Clark

-- 
You received this message because you are subscribed to the Google Groups 
"Everything List" group.
To unsubscribe from this group and stop receiving emails from it, send an email 
to everything-list+unsubscr...@googlegroups.com.
To post to this group, send email to everything-list@googlegroups.com.
Visit this group at https://groups.google.com/group/everything-list.
For more options, visit https://groups.google.com/d/optout.


Re: Consistency of Postulates of QM

2017-12-12 Thread agrayson2000


On Tuesday, December 12, 2017 at 10:52:12 PM UTC, Bruce wrote:
>
> On 13/12/2017 9:45 am, agrays...@gmail.com  wrote:
>
> On Tuesday, December 12, 2017 at 10:14:01 PM UTC, Bruce wrote: 
>>
>>
>> So Schrödinger's cat was once a coherent state of a cat in a box, and 
>> the splitting occurs with the decay of a nucleus; 
>
>
> *Unlike the double slit experiment, which can only be understood with 
> interference and the model that the electron wave, say, goes through both 
> slits, the nuclear decay is a BINARY event, very roughly like a coin toss 
> where there is no interference. Generalizing interference to every quantum 
> state is where Schrodinger went wrong. The cat, which shares or inherits 
> the wf from the radioactive decay, is never in both Alive and Dead states 
> simultaneously. AG*
>
>
> In the double slit, the paths through the two slits remain coherent, so 
> they can interfere when they come together. In the case of nuclear decay, 
> the coherence is lost immediately, so the nucleus does not interfere with 
> the decay products.
>
> Bruce
>

*So you agree or disagree with my conclusion; namely, the cat is never in a 
superposition of states? That is, no situation where cat is Alive and Dead 
simultaneously. I think you disagree and think the nuclear state is 
superposed with interference existing. AG *

 

> decoherent entanglement then leads to the splitting of worlds FAPP.
>


-- 
You received this message because you are subscribed to the Google Groups 
"Everything List" group.
To unsubscribe from this group and stop receiving emails from it, send an email 
to everything-list+unsubscr...@googlegroups.com.
To post to this group, send email to everything-list@googlegroups.com.
Visit this group at https://groups.google.com/group/everything-list.
For more options, visit https://groups.google.com/d/optout.


Re: Consistency of Postulates of QM

2017-12-12 Thread Bruce Kellett

On 13/12/2017 9:45 am, agrayson2...@gmail.com wrote:

On Tuesday, December 12, 2017 at 10:14:01 PM UTC, Bruce wrote:


So Schrödinger's cat was once a coherent state of a cat in a box, and
the splitting occurs with the decay of a nucleus; 



*Unlike the double slit experiment, which can only be understood with 
interference and the model that the electron wave, say, goes through 
both slits, the nuclear decay is a BINARY event, very roughly like a 
coin toss where there is no interference. Generalizing interference to 
every quantum state is where Schrodinger went wrong. The cat, which 
shares or inherits the wf from the radioactive decay, is never in both 
Alive and Dead states simultaneously. AG*


In the double slit, the paths through the two slits remain coherent, so 
they can interfere when they come together. In the case of nuclear 
decay, the coherence is lost immediately, so the nucleus does not 
interfere with the decay products.


Bruce




decoherent entanglement then leads to the splitting of worlds FAPP.




--
You received this message because you are subscribed to the Google Groups 
"Everything List" group.
To unsubscribe from this group and stop receiving emails from it, send an email 
to everything-list+unsubscr...@googlegroups.com.
To post to this group, send email to everything-list@googlegroups.com.
Visit this group at https://groups.google.com/group/everything-list.
For more options, visit https://groups.google.com/d/optout.


Re: Consistency of Postulates of QM

2017-12-12 Thread agrayson2000


On Tuesday, December 12, 2017 at 10:14:01 PM UTC, Bruce wrote:
>
> On 13/12/2017 2:12 am, smitra wrote: 
> > On 12-12-2017 12:33, Bruce Kellett wrote: 
> >> On 12/12/2017 9:46 pm, smitra wrote: 
> >>> 
> >>> Yes, it's only an estimation but it yields a good order of magnitude 
> >>> estimate for the center of mass.  What the calculation shows is that 
> >>> quantum superpositions do exists at the macroscopic level and these 
> >>> can then be amplified by chaotic dynamics. Of course,  it then 
> >>> becomes incoherent, but in the MWI that's besides the point. 
> >> 
> >> MWI splitting depends on coherence, so it is certainly not beside the 
> >> point for the coin toss. 
> > 
> > It doesn't depend on coherence. Why would it matter if the state of 
> > the coin gets entangled with a zillion other environmental degrees of 
> > freedom? The dynamics according to unitary time evolution leads toa   
> > superposition, no matter how many degrees of freedom are involved in 
> > the entanglement. 
>
> You are missing the point. Splitting according to the Schrödinger 
> equation does depend on coherence. The decoherence that entangles the 
> coin with a zillion other environmental degrees of freedom occurs after 
> the splitting. Given decoherence, the process is irreversible FAPP, 
> which means that there is no practical way, by design or chance, that a 
> decohered state can recohere. Sure, in the many worlds of MWI the 
> superposition, if it once existed, is still intact. But if no such 
> superposition ever existed, then it can't be created from non-coherent 
> interactions. 
>
> So Schrödinger's cat was once a coherent state of a cat in a box, and 
> the splitting occurs with the decay of a nucleus; 


*Unlike the double slit experiment, which can only be understood with 
interference and the model that the electron wave, say, goes through both 
slits, the nuclear decay is a BINARY event, very roughly like a coin toss 
where there is no interference. Generalizing interference to every quantum 
state is where Schrodinger went wrong. The cat, which shares or inherits 
the wf from the radioactive decay, is never in both Alive and Dead states 
simultaneously. AG*
 

> decoherent 
> entanglement then leads to the splitting of worlds FAPP. But given an 
> arbitrary coin, it is already non-coherently entangled with many 
> environmental degrees of freedom, but there is no state that can lead to 
> {heads>+|tails>} in a unitary manner, so there is no state that can then 
> evolve into a splitting and decoherence into worlds distinguished by 
> either |heads> or |tails>. If you think that there is, write out the 
> schematic sequence of states evolving under the SE that leads to this 
> result. 
>
> > The only relevant issue here is if you then become different due to 
> > such an entanglement such that even before you make a measurement, you 
> > will already feel subjectively different in the two sectors. 
>
> What you feel subjectively is irrelevant to the existence or 
> non-existence of splitting. In the Cat scenario, you don't know the 
> result until you open the box, but the split occurred long before that, 
> and the cat was dead in one world but still alive in the other long 
> before the observer became aware of it. 
>
> > Otherwise you have an exact copy as far as your subjective feelings 
> > are concerned, so before measurement you can end up in both sectors. 
>
> You are in both sectors even after the split -- it is just that the 
> "you" in one world exist alongside a live cat, and in the other world 
> you exist alongside a dead cat. However, in the case of the coin toss, 
> there is no splitting due to the toss -- so if there is splitting caused 
> by some other adjacent quantum event, then you exist in each of the 
> consequent worlds in the same state viv a vis the coin; i.e., it is 
> either heads in all the worlds, or tails in all the worlds. 
>
> MWI mumbo jumbo and appeals to "entanglement" magic cannot get you out 
> of the clear results of quantum evolution. 
>
> Bruce 
>

-- 
You received this message because you are subscribed to the Google Groups 
"Everything List" group.
To unsubscribe from this group and stop receiving emails from it, send an email 
to everything-list+unsubscr...@googlegroups.com.
To post to this group, send email to everything-list@googlegroups.com.
Visit this group at https://groups.google.com/group/everything-list.
For more options, visit https://groups.google.com/d/optout.


Re: Consistency of Postulates of QM

2017-12-12 Thread Bruce Kellett

On 13/12/2017 2:12 am, smitra wrote:

On 12-12-2017 12:33, Bruce Kellett wrote:

On 12/12/2017 9:46 pm, smitra wrote:


Yes, it's only an estimation but it yields a good order of magnitude 
estimate for the center of mass.  What the calculation shows is that 
quantum superpositions do exists at the macroscopic level and these 
can then be amplified by chaotic dynamics. Of course,  it then 
becomes incoherent, but in the MWI that's besides the point.


MWI splitting depends on coherence, so it is certainly not beside the
point for the coin toss.


It doesn't depend on coherence. Why would it matter if the state of 
the coin gets entangled with a zillion other environmental degrees of 
freedom? The dynamics according to unitary time evolution leads toa  
superposition, no matter how many degrees of freedom are involved in 
the entanglement.


You are missing the point. Splitting according to the Schrödinger 
equation does depend on coherence. The decoherence that entangles the 
coin with a zillion other environmental degrees of freedom occurs after 
the splitting. Given decoherence, the process is irreversible FAPP, 
which means that there is no practical way, by design or chance, that a 
decohered state can recohere. Sure, in the many worlds of MWI the 
superposition, if it once existed, is still intact. But if no such 
superposition ever existed, then it can't be created from non-coherent 
interactions.


So Schrödinger's cat was once a coherent state of a cat in a box, and 
the splitting occurs with the decay of a nucleus; decoherent 
entanglement then leads to the splitting of worlds FAPP. But given an 
arbitrary coin, it is already non-coherently entangled with many 
environmental degrees of freedom, but there is no state that can lead to 
{heads>+|tails>} in a unitary manner, so there is no state that can then 
evolve into a splitting and decoherence into worlds distinguished by 
either |heads> or |tails>. If you think that there is, write out the 
schematic sequence of states evolving under the SE that leads to this 
result.


The only relevant issue here is if you then become different due to 
such an entanglement such that even before you make a measurement, you 
will already feel subjectively different in the two sectors.


What you feel subjectively is irrelevant to the existence or 
non-existence of splitting. In the Cat scenario, you don't know the 
result until you open the box, but the split occurred long before that, 
and the cat was dead in one world but still alive in the other long 
before the observer became aware of it.


Otherwise you have an exact copy as far as your subjective feelings 
are concerned, so before measurement you can end up in both sectors.


You are in both sectors even after the split -- it is just that the 
"you" in one world exist alongside a live cat, and in the other world 
you exist alongside a dead cat. However, in the case of the coin toss, 
there is no splitting due to the toss -- so if there is splitting caused 
by some other adjacent quantum event, then you exist in each of the 
consequent worlds in the same state viv a vis the coin; i.e., it is 
either heads in all the worlds, or tails in all the worlds.


MWI mumbo jumbo and appeals to "entanglement" magic cannot get you out 
of the clear results of quantum evolution.


Bruce

--
You received this message because you are subscribed to the Google Groups 
"Everything List" group.
To unsubscribe from this group and stop receiving emails from it, send an email 
to everything-list+unsubscr...@googlegroups.com.
To post to this group, send email to everything-list@googlegroups.com.
Visit this group at https://groups.google.com/group/everything-list.
For more options, visit https://groups.google.com/d/optout.


Re: Consistency of Postulates of QM

2017-12-12 Thread smitra

On 12-12-2017 12:33, Bruce Kellett wrote:

On 12/12/2017 9:46 pm, smitra wrote:

On 12-12-2017 02:20, Bruce Kellett wrote:

On 12/12/2017 11:39 am, smitra wrote:

On 11-12-2017 23:11, Bruce Kellett wrote:

On 12/12/2017 1:51 am, smitra wrote:

On 11-12-2017 15:12, Bruno Marchal wrote:

On 10 Dec 2017, at 23:38, Bruce Kellett wrote:


On 11/12/2017 2:19 am, Bruno Marchal wrote:

On 09 Dec 2017, at 00:03, Bruce Kellett wrote:

On 9/12/2017 4:21 am, Bruno Marchal wrote:
Similarly, a shroedinger car, once alive + dead, will never  
become a pure alive, or dead cat. It will only seems so for  
anyone looking at the cat, in the {alive, dead} 
base/apparatus.  Superposition never disappear, and a coin 
moree or less with a precise position, is always a 
superposition of a coin with more  or less precise momenta. 
The relation is given by the Fourier  transforms, which gives 
the relative accessible states/worlds.


I pointed out that for a macroscopic object such as a coin, 
the  uncertainty relations give uncertainties in positions 
and/or  momentum far below any level of possible detection.


Of possible practical detection. That is good FAPP, but 
irrelevant  for theoretical consideration.


This is a purely rhetorical objection, Bruno. And when you trot 
this  out, as you do regularly, I know that your purpose is to 
obfuscate,  and hide the fact that you have no rational argument 
to offer.


You confuse physics and metaphysics. The difference is not 
rhetorical,

 but fundamental in this thread.



We actually do detect quantum uncertainties for macroscopic 
objects routinely when doing typical quantum experiments. 
Interference experiments involving photons is a good example. 
Suppose we have an interferometer that has mirrors in it, the 
photons bounce off the mirrors and at some spot the different 
possible paths come together and you can then detect or not detect 
photons there.


One can then ask why the momentum absorbed by the mirror when a 
photon bounces off it, does not destroy the interference pattern. 
One may consider here a thought experiment where the mirrors are 
freely floating in a magnetic field. But that's not actually 
necessary, if you could in principle detect the momentum from the 
recoil of the photons, then you won't get interference and in 
general the interference becomes weaker if you can in principle 
get partial information.


The answer to this question is that macroscopic objects such as 
the mirror in interferometers do not have sharply defined momenta. 
In fact, you could argue that unless the mirror surface is not 
located to well within the wavelength of light, you obviously 
wouldn't get interference, and applying the uncertainty relations 
then also gives you an uncertainty in the momentum. But this 
doesn't tell you what the uncertainty in the momentum typically 
is.


The uncertainty in the center of mass position can be estimated 
crudely as the thermal De-Broglie wavelength. A displacement well 
within this length scale will not lead to the environment 
interacting appreciably differently with it. So, the uncertainty 
in the position will be of the order of h/sqrt(m k T). The 
interpretation is then that a wavefunction spreading beyond this 
length will effectively collapse back to within this length scale 
due to the environment effectively having located the center of 
mass within this scale.


The uncertainty in the momentum is then of the order of sqrt(m k 
T), and this can actually be quite large for large objects. This 
large uncertainty in the momentum in absolute terms explains why 
you can actually do quantum experiments using macroscopic 
measurement devices.


There is a fairly serious error in your analysis. You use an
expression for the momentum, p = mv = sqrt(3mkT), which applies to
molecules in an ideal gas. Mirrors in quantum experiments are not
molecules in an ideal gas! What is more, molecules in an ideal gas 
are
not located within their de Broglie wavelengths. You forget that 
the

uncertainty principle applies to the uncertainty in measurement
results, and the molecules of the gas are not constrained such that
their position uncertainty is that small.

In other words, you are talking nonsense.



No, your arguments are totally wrong here.

 The thermal de Broglie wavelength is a measure for the coherence 
length of the molecules in a gas and this then gives the coherence 
length in momentum space via the uncertainty relation


No, the de Broglie wavelength is the wavelength, not the coherence
length. The coherence length is given by the size of the wave packet,
so for photons, the coherence length is often orders of magnitude
greater than the wavelength.

(if you want to invoke measurement here, you can say that the 
environment consisting of all other molecules effectively "measure" 
the position of the center of mass). To a good approximation this 
also applies to atoms in a solid, the fact that a solid is not an 
ideal gas doesn't actually 

Re: Consistency of Postulates of QM

2017-12-12 Thread Bruce Kellett

On 12/12/2017 10:47 pm, Bruno Marchal wrote:

On 12 Dec 2017, at 11:12, Bruce Kellett wrote:

On 12/12/2017 8:20 pm, Bruno Marchal wrote:

On 11 Dec 2017, at 23:15, Bruce Kellett wrote:
On 12/12/2017 1:12 am, Bruno Marchal wrote:

On 10 Dec 2017, at 23:38, Bruce Kellett wrote:

On 11/12/2017 2:19 am, Bruno Marchal wrote:

On 09 Dec 2017, at 00:03, Bruce Kellett wrote:

On 9/12/2017 4:21 am, Bruno Marchal wrote:
Similarly, a shroedinger car, once alive + dead, will never 
become a pure alive, or dead cat. It will only seems so for 
anyone looking at the cat, in the {alive, dead} 
base/apparatus. Superposition never disappear, and a coin 
moree or less with a precise position, is always a 
superposition of a coin with more or less precise momenta. The 
relation is given by the Fourier transforms, which gives the 
relative accessible states/worlds.


I pointed out that for a macroscopic object such as a coin, the 
uncertainty relations give uncertainties in positions and/or 
momentum far below any level of possible detection.


Of possible practical detection. That is good FAPP, but 
irrelevant for theoretical consideration.


This is a purely rhetorical objection, Bruno. And when you trot 
this out, as you do regularly, I know that your purpose is to 
obfuscate, and hide the fact that you have no rational argument 
to offer.


You confuse physics and metaphysics. The difference is not 
rhetorical, but fundamental in this thread.


Rubbish. The central point of contention on this thread is whether 
a coin toss can be regarded as a classical event, with 
probabilities given by ignorance of the initial conditions, or as a 
quantum event with probabilities given by purely quantum 
uncertainties.


This is a straightforward question of physics, and has nothing to 
do with metaphysics. As usual, you introduce the term 'metaphysics' 
merely to obfuscate, because you have no intelligent response to 
the clear physics of the situation.



I was using "metaphysics" in opposition to your instrumentalist FAPP 
distraction. Metaphysics and/or theology involves the question of 
the existence of the universe(s), parallel or not.


Then you make ad hominem insult in place of argument. I guess your 
goal is not to enlighten me.


You are the one making the personal attacks. You can do metaphysics 
in the privacy of your own home, but here in public, I talk about 
verifiable physics.


My attack was on your use of the "FAPP" to change the subject of the 
discussion which was on metaphysics at the start: the existence of the 
(obviously undetectable) other term of some macroscopic superposition. 
(It was not ad hominem at all).


Neither were any of my comments personal attacks. The original 
discussion was about whether or not the world split on the coin toss. 
Very little to do with metaphysics, and your continued introduction of 
irrelevant considerations has been nothing more than a distraction from 
the main discussion, adding nothing.


Bruce

--
You received this message because you are subscribed to the Google Groups 
"Everything List" group.
To unsubscribe from this group and stop receiving emails from it, send an email 
to everything-list+unsubscr...@googlegroups.com.
To post to this group, send email to everything-list@googlegroups.com.
Visit this group at https://groups.google.com/group/everything-list.
For more options, visit https://groups.google.com/d/optout.


Re: Consistency of Postulates of QM

2017-12-12 Thread Bruce Kellett

On 12/12/2017 10:55 pm, Bruno Marchal wrote:

On 12 Dec 2017, at 11:14, Bruce Kellett wrote:

On 12/12/2017 8:26 pm, Bruno Marchal wrote:

On 12 Dec 2017, at 02:02, Bruce Kellett wrote:

On 12/12/2017 11:44 am, smitra wrote:

On 11-12-2017 23:15, Bruce Kellett wrote:

On 12/12/2017 1:12 am, Bruno Marchal wrote:

On 10 Dec 2017, at 23:38, Bruce Kellett wrote:

On 11/12/2017 2:19 am, Bruno Marchal wrote:

On 09 Dec 2017, at 00:03, Bruce Kellett wrote:

On 9/12/2017 4:21 am, Bruno Marchal wrote:

Similarly, a shroedinger car, once alive + dead, will never
become a pure alive, or dead cat. It will only seems so for
anyone looking at the cat, in the {alive, dead} base/apparatus.
Superposition never disappear, and a coin moree or less with a
precise position, is always a superposition of a coin with more
or less precise momenta. The relation is given by the Fourier
transforms, which gives the relative accessible states/worlds.


I pointed out that for a macroscopic object such as a coin, the
uncertainty relations give uncertainties in positions and/or
momentum far below any level of possible detection.


Of possible practical detection. That is good FAPP, but 
irrelevant

for theoretical consideration.


This is a purely rhetorical objection, Bruno. And when you trot
this out, as you do regularly, I know that your purpose is to
obfuscate, and hide the fact that you have no rational argument to
offer.


You confuse physics and metaphysics. The difference is not
rhetorical, but fundamental in this thread.


Rubbish. The central point of contention on this thread is whether a
coin toss can be regarded as a classical event, with probabilities
given by ignorance of the initial conditions, or as a quantum event
with probabilities given by purely quantum uncertainties.

This is a straightforward question of physics, and has nothing to do
with metaphysics. As usual, you introduce the term 'metaphysics'
merely to obfuscate, because you have no intelligent response to the
clear physics of the situation.



That the probabilities are given by classical physics does not imply
that there is no branching due to the coin toss.


It does, because there is no superposition of head/tails -- no 
possibility of interference between heads and tails.


You are begging the question. The point was that without collapse, 
the shaking of the dice or coin can make the superposition of the 
positions (inherent in the Heisenberg uncertainty) can add up to 
make the coin behaving sufficiently differently to obtain a 
superposition of the head+tail or 1+2+...+6 superposition.


No, shaking the coin cannot make non-coherent uncertainties add up to 
anything. The physics is against you here, Bruno. There is no 
superposition, and no splitting of worlds on the toss. If you think 
different, prove it.



Just to be sure, do you agree that without collapse, the schroedinger 
cat remains in the state alive+dead, even after observation, and we 
see it alive OR dead, just by the first-person mechanist indeterminacy 
(or something akin to it) ?


The superposition exists in the original quantum state -- essentially 
the radioactive nucleus which is a quantum system in a superposition of 
decayed/not-decayed. Because of the experimental set up, this 
superposition is amplified so that it becomes entangled with the 
environment, including the cat and us. So:


   |nucleus>|box>|cat>|observer>|rest of the world> --> (by unitary 
evolution)
   {|decayed>|poison spilt>|cat dead>|see dead cat>|rest of world 
confirms dead cat> +
   |not decayed>|poison bottle intact>|cat alive>|see live cat>|rest of 
world confirms live cat>}


And the decoherence of the quantum phases into the |rest of the world> 
environmental states diagonalizes the density matrix FAPP. If you 
insist, the superposition is intact in the bird view, but such a 
superposition can never recohere, and there are NO consequences of the 
existence of other branches, practical or otherwise. Maintaining their 
existence might satisfy your existential angst, but it has nothing to do 
with physics or experience.



Also, how could the quantum uncertainties becoming non-coherent? 
Decoherence, without collapse is something relative to the environment 
when its get itself entangled with the superposition of the object 
observed. The splitting of worlds is not due to the toss, but to the 
fact that the position of the coin diffuse, which means get different 
in the multiverse. Instead of shaking the box, waiting long enough 
would work as well.


The coin case is different in an essential way -- it does not start from 
a single quantum state that can be expanded into {|heads> + |tails>} by 
any single quantum event. The system starts off decohered and 
non-coherent. So just as you can never bring the two branches of the cat 
back together, you can never take an initially decohered state and 
reconstruct some imagined coherent superposition, unitary evolution and 
the laws of thermodynamics forbid it. 

Re: Consistency of Postulates of QM

2017-12-12 Thread Bruno Marchal


On 12 Dec 2017, at 11:14, Bruce Kellett wrote:


On 12/12/2017 8:26 pm, Bruno Marchal wrote:

On 12 Dec 2017, at 02:02, Bruce Kellett wrote:

On 12/12/2017 11:44 am, smitra wrote:

On 11-12-2017 23:15, Bruce Kellett wrote:

On 12/12/2017 1:12 am, Bruno Marchal wrote:

On 10 Dec 2017, at 23:38, Bruce Kellett wrote:

On 11/12/2017 2:19 am, Bruno Marchal wrote:

On 09 Dec 2017, at 00:03, Bruce Kellett wrote:

On 9/12/2017 4:21 am, Bruno Marchal wrote:

Similarly, a shroedinger car, once alive + dead, will never
become a pure alive, or dead cat. It will only seems so for
anyone looking at the cat, in the {alive, dead} base/ 
apparatus.
Superposition never disappear, and a coin moree or less  
with a
precise position, is always a superposition of a coin with  
more

or less precise momenta. The relation is given by the Fourier
transforms, which gives the relative accessible states/ 
worlds.


I pointed out that for a macroscopic object such as a coin,  
the

uncertainty relations give uncertainties in positions and/or
momentum far below any level of possible detection.


Of possible practical detection. That is good FAPP, but  
irrelevant

for theoretical consideration.


This is a purely rhetorical objection, Bruno. And when you trot
this out, as you do regularly, I know that your purpose is to
obfuscate, and hide the fact that you have no rational  
argument to

offer.


You confuse physics and metaphysics. The difference is not
rhetorical, but fundamental in this thread.


Rubbish. The central point of contention on this thread is  
whether a

coin toss can be regarded as a classical event, with probabilities
given by ignorance of the initial conditions, or as a quantum  
event

with probabilities given by purely quantum uncertainties.

This is a straightforward question of physics, and has nothing  
to do

with metaphysics. As usual, you introduce the term 'metaphysics'
merely to obfuscate, because you have no intelligent response to  
the

clear physics of the situation.



That the probabilities are given by classical physics does not  
imply

that there is no branching due to the coin toss.


It does, because there is no superposition of head/tails -- no  
possibility of interference between heads and tails.


You are begging the question. The point was that without collapse,  
the shaking of the dice or coin can make the superposition of the  
positions (inherent in the Heisenberg uncertainty) can add up to  
make the coin behaving sufficiently differently to obtain a  
superposition of the head+tail or 1+2+...+6 superposition.


No, shaking the coin cannot make non-coherent uncertainties add up  
to anything. The physics is against you here, Bruno. There is no  
superposition, and no splitting of worlds on the toss. If you think  
different, prove it.



Just to be sure, do you agree that without collapse, the schroedinger  
cat remains in the state alive+dead, even after observation, and we  
see it alive OR dead, just by the first-person mechanist indeterminacy  
(or something akin to it) ?


Also, how could the quantum uncertainties becoming non-coherent?  
Decoherence, without collapse is something relative to the environment  
when its get itself entangled with the superposition of the object  
observed. The splitting of worlds is not due to the toss, but to the  
fact that the position of the coin diffuse, which means get different  
in the multiverse. Instead of shaking the box, waiting long enough  
would work as well.


Bruno




Bruce

--
You received this message because you are subscribed to the Google  
Groups "Everything List" group.
To unsubscribe from this group and stop receiving emails from it,  
send an email to everything-list+unsubscr...@googlegroups.com.

To post to this group, send email to everything-list@googlegroups.com.
Visit this group at https://groups.google.com/group/everything-list.
For more options, visit https://groups.google.com/d/optout.


http://iridia.ulb.ac.be/~marchal/



--
You received this message because you are subscribed to the Google Groups 
"Everything List" group.
To unsubscribe from this group and stop receiving emails from it, send an email 
to everything-list+unsubscr...@googlegroups.com.
To post to this group, send email to everything-list@googlegroups.com.
Visit this group at https://groups.google.com/group/everything-list.
For more options, visit https://groups.google.com/d/optout.


  1   2   3   4   5   6   7   >